Download as pdf or txt
Download as pdf or txt
You are on page 1of 273

1

TABLE OF CONTENTS

(EKLAVYA UNIT - I TO X)

UNIT - I
Indefinite and Definite Integration
Part - A .................................................................................................................................................. 5
Part - B .................................................................................................................................................. 11
Answer Key ............................................................................................................................................. 132
Solutions .............................................................................................................................................. 136

UNIT - II
Area Under the curves, Differential Equation
Part - A .................................................................................................................................................. 16
Part - B .................................................................................................................................................. 19
Answer Key ............................................................................................................................................. 132
Solutions .............................................................................................................................................. 151

Probability
Part - A .................................................................................................................................................. 21
Part - B .................................................................................................................................................. 24
Answer Key ............................................................................................................................................. 132
Solutions .............................................................................................................................................. 156

UNIT - III
Vectors & 3D Coordinate Geometry
Part - A .................................................................................................................................................. 29
Part - B .................................................................................................................................................. 36
Answer Key ............................................................................................................................................. 132
Solutions .............................................................................................................................................. 163
2

UNIT - IV
Limit Continuity and Differentiability(LCD)
Part - A .................................................................................................................................................. 42
Part - B .................................................................................................................................................. 45
Answer Key ............................................................................................................................................. 133
Solutions .............................................................................................................................................. 174

Functions, Inverse Trigonometric Functions(ITF)


Part - A .................................................................................................................................................. 49
Part - B .................................................................................................................................................. 52
Answer Key ............................................................................................................................................. 133
Solutions .............................................................................................................................................. 181

UNIT - V
Differentiation, Applications of Derivatives(AOD)
Part - A .................................................................................................................................................. 55
Part - B .................................................................................................................................................. 62
Answer Key ............................................................................................................................................. 133
Solutions .............................................................................................................................................. 189

UNIT - VI
Quadratic Equations
Part - A .................................................................................................................................................. 69
Part - B .................................................................................................................................................. 72
Answer Key ............................................................................................................................................. 133
Solutions .............................................................................................................................................. 202

Sequence and series


Part - A .................................................................................................................................................. 74
Part - B .................................................................................................................................................. 77
Answer Key ............................................................................................................................................. 134
Solutions .............................................................................................................................................. 207
3

UNIT - VII
Matrices and Determinants
Part - A .................................................................................................................................................. 80
Part - B .................................................................................................................................................. 82
Answer Key ............................................................................................................................................. 134
Solutions .............................................................................................................................................. 214

Permutations and Combinations(PnC), Binomial Theorem


Part - A .................................................................................................................................................. 84
Part - B .................................................................................................................................................. 88
Answer Key ............................................................................................................................................. 134
Solutions .............................................................................................................................................. 217

UNIT - VIII
Solution of Triangles (SOT)
Part - A .................................................................................................................................................. 91
Part - B .................................................................................................................................................. 94
Answer Key ............................................................................................................................................. 134
Solutions .............................................................................................................................................. 224

Trigonometry, Trigonometric Equations


Part - A .................................................................................................................................................. 98
Part - B .................................................................................................................................................. 100
Answer Key ............................................................................................................................................. 134
Solutions .............................................................................................................................................. 233

UNIT - IX
Straight lines
Part - A .................................................................................................................................................. 103
Part - B .................................................................................................................................................. 105
Answer Key ............................................................................................................................................. 135
Solutions .............................................................................................................................................. 239
4

Circles
Part - A .................................................................................................................................................. 107
Part - B .................................................................................................................................................. 110
Answer Key ............................................................................................................................................. 135
Solutions .............................................................................................................................................. 244

UNIT - X
Complex Numbers
Part - A .................................................................................................................................................. 115
Part - B .................................................................................................................................................. 118
Answer Key ............................................................................................................................................. 135
Solutions .............................................................................................................................................. 254

Conic Sections
Part - A .................................................................................................................................................. 121
Part - B .................................................................................................................................................. 128
Answer Key ............................................................................................................................................. 135
Solutions .............................................................................................................................................. 261
INDEFINITE AND DEFINITE INTEGRATION 5

Indefinite and Definite Integration

Part-A 4. If I   x
x2  1
dx , then I equals _____.
x2  1
1. If e is the eccentricity of a hyperbola and
1 1 4
f  e is the eccentricity of its conjugate (a) x4  1  x 1  c
2 2
2
1 1
hyperbola, then   f  e  f  f  e   de
1
is (b)
2 2

x 4  1  ln x 2  x 4  1  c 
equal to _____. (c) x 4  1  sin 1  x 2   c
(a) 3 (b) 1
(d) x 4  1  2sin 1  x 2   c
3
(c) (d) 2 5. If f  x  is a differentiable function such that
2
2. Consider the equation 2
x
f (t )dt 
 f  x for all x  and
lim 
x
tdt 1
 . Which

0 2
19
 x x  4  x  a3  2t  104
x0
0
f (2)  0 , then value of f  2 is _____.
of the following is/are correct? (a) 3 (b) 1
(a) a  771 (c) 2 (d) –1
(b) a  312 6. For any integer n , the integral

(c) L’Hospital’s Rule is used to solve above cos 2 x
e cos3   2n  1 x  dx, n  has the
equation. 0
(d) none of these value _____.
3. Statement-1 : If f :  0,    is continuous (a)  (b) 1
and non-constant function such that 
 (c) 0 (d)
2
 f  x  sin xdx  0 , then f  x   0 has at least
0
7. Let a , b, c be non-zero real numbers such
1
one root in  0,   .
 e  e x  ax 2  bx  c  dx
x
that
Statement-2 : If g  x  is continuous and 0
2
g   g     0,    , then g  x   0 has at    e x  e  x  ax 2  bx  c  dx .
least one root in  ,   . 0

(a) Statement-1 is True, Statement-2 is True; Then the quadratic equation ax 2  bx  c  0


Statement-2 is a correct explanation for has _____.
Statement-1 (a) no root in (0, 1)
(b) Statement-1 is True, Statement-2 is True; (b) at least one root in (1, 2)
Statement-2 is not a correct explanation (c) both roots in (0, 1)
for Statement-1 (d) none of these
(c) Statement-1 is True, Statement-2 is False
(d) Statement-1 is False, Statement-2 is True
INDEFINITE AND DEFINTE INTEGRATION 6

8. The value of 13. Statement-1 : If


 /2 4 x
  
 2sin x dx   sin 1  log 2  x  2   dx is equal   x     3sin t  4cos t  dt , x   ,  ,
 / 2 5/2 0 6 3
to _____. 
  x  attain its maximum value at x  .
5 3
(a) (b) 
4 because
x
3 
(c) (d) Statement-2 :   x     3sin t  4cos t  dt is
4 4 0
9. The values of a and b which satisfy  
2 sin 2 x  cos x an increasing function in  ,  .
 4  cos2 x  4sin x dx  a log sin x  1 6 3
(a) Statement-1 is True, Statement-2 is True;
b log sin x  3  c , are _____.
Statement-2 is a correct explanation for
3 11 3 11 Statement-1
(a)  , (b) ,
2 2 2 2 (b) Statement-1 is True, Statement-2 is True;
3 11 3 11 Statement-2 is NOT a correct explanation
(c) ,  (d)  , 
2 2 2 2 for Statement-1
10. The value of (c) Statement-1 is True, Statement-2 is False
4
  sin x   cos x  (d) Statement-1 is False, Statement-2 is True
  sin x      dx , where [ ] 14. If f  x   x3  x , then
0  2   2 
2 5
denotes the GIF, is _____.
 f  x  dx  2  f 1  2 x  dx is equal to _____.
(a) 8 (b) 6 1 1
(c) 4 (d) none of these
(a) 21 (b) 9
11. If f  x   ae2x  be x  cx satisfies the
(c) 8 (d) 18
condition f  0  1, f   log 2  31 and 15. The angle between the tangents of
log 4 x
39
  f  x   cx  dx  , then _____. f  x     2t  5  dt at points where it cuts
0 2 2

(a) a = 5, b = 6, c = 3 the X-axis is ____.


(b) a = 5, b = –6, c = 3 (a) 30o (b) 45o
(c) a = –5, b = 6, c =3 (c) 90o (d) 60o

(d) none of these sin x
4
16. If I   1 x 2
dx , then I is _____.
tx
12. Let f  x    e dt , 0  x  4 . If the range of 

0 
(a) (b) 
f  x  is  a, b then b  a is _____. 2
(c) 2 (d) 0
(a) e 2 (b) e 2  1
e2
(c) e 2  1 (d)
2
INDEFINITE AND DEFINITE INTEGRATION 7

g  x   1  x  ln x  5  g  x 21. The line y  intersects the curve


17. Let  dx  c,
x  ln x 
6
 ln x 
5
y  g  x  , at least two distinct points, where
x 2
g 1  e and g :  0, e    0,   . If g n  x  x2
 g (t ) dt   t 2 g (t ) dt  x  R then set of
2 x
denotes the n th derivative of g  x  , then the 2

possible value of  .
value of g 2012 1 is _____.  1 1  1 1
(a)   ,  (b)   , 
(a) 2011g 1 (b) 2012 g 1  2 2  2 2
(c) 2013g 1 (d) 2014 g 1  1 1 1 1 
(c)   ,   0 (d)  ,0, 
 /2  2 2 2 2
sin x sin  cosec
18. 0 dx  _____. tdt dt
1  cos x  sin x 22. A   2
and B   2
then the
1 1 t 1 t (1  t )
 
(a) (b)  log 2 A A2 B
4 4 A B
  value of e e B2 1 is _____.
(c)  log 2 (d)  log 2 1 A 2  B2 1
4 4
x
(a) sin  (b) cosec

19. Let f  x    t 2  2t  2 dt , where x is the (c) A + B (d) 1
0
x
set of real numbers satisfying the in-equation 23. Consider  t 2  8t  13 dt  x sin  ax  and
 
log 2 1  6 x  x 2  8  0 . If f  x    ,   ,
 a, x 
0

 0  x takes the values for which


then maximum value of    is _____.
the equation has a solution, then the number
103 104 of values of a   0,100 is _____.
(a) (b)
3 3
105 106 (a) 1 (b) 2
(c) (d)
3 3
(c) 3 (d) 4
x dx
20. If  2012 1012 3012
2  tan 2 z
 1  x2   2  x2  24. Let I1   xf  x  3  x   dx and let
 sec 2 z



1  f  x   2  k then which is true 2  tan 2 z
I2   f  x  3  x   dx where f is a
1
(a)   503;   500, f 2 
 
  sec 2 z
continuous function and z is any real
1 I
(b)   503;   250, f 2 
 
  number, then 1  _____.
I2
1 3 1
(c)   503;   500, f 1  (a) (b)
  2 2
1 2
(d)   503;   225, f 3 
 
  (c) 1 (d)
3
INDEFINITE AND DEFINTE INTEGRATION 8

sec2 x  2010 P x 27. If


25. If  dx   C, then
sin 2010 x sin 2010 x dx 1  1 x  sin x 
  5  4cos x 2  A tan  tan   B 
3 2
C
 5  4cos x 
 
value of P   is _____.
3 then  A, B  _____.
(a) 0  10  4   10 4 
1 (a)  ,  (b)  , 
(b)  27 9   27 9 
3  10 4   10  4 
(c)  ,  (d)  , 
(c) 3  27 9   27 9 
(d) None of these 2 17
28. The value of  x 26  x  1  5 x  3 dx
f  x 0
26. If g  x   where f  x  equals to _____.
 x  a  x  b  x  c 
(a) 0 (b) 1
is a polynomial of degree  3 , then _____. 27
2
1 a f  a  log x  a (c) (d) 325
9
(a)  g  x  dx  1 b f  b  log x  b
29. Let  x  denote the greatest integer not
1 c f  c  log x  c
exceeding x for x  .
2
1 a a 1000

 1 b b2  k Statement-1: 2 sec 1 x  dx  998


1 c c2

2
Statement-2: 0  sec 1 x  for x  1 .
1 a f  a  log  x  a  2
d 2 (a) Statement-1 is True, Statement-2 is True;
(b) g  x  1 b f  b  log  x  b 
dx Statement-2 is a correct Explanation for
2
1 c f  c  log  x  c  Statement-1
(b) Statement-1 is True, Statement-2 is True;
1 a a2
Statement-2 is NOT a correct explanation
 1 b b2
for Statement-1
1 c c2 (c) Statement-1 is True, Statement-2 is False
1 a f  a  log x  a (d) Statement-1 is False, Statement-2 is True
g  x  dx  1 b f  b  log x  b 30. If f  x  is integrable over 1, 2 , then
(c)  2
1 c f  c  log x  c
2
 f  x  dx is equal to _____.
a a 1 1

1 n r
 b2 b 1 k (a) lim
n  n
 f 
r 1 n
c2 c 1
1 2n  r 
(d) None of these (b) lim  f  
n  n
r  n 1 n
n
1 rn
(c) lim  f  
n n
r 1  n 
1 2n  r 
(d) lim  f  
n  n
r 1 n
INDEFINITE AND DEFINITE INTEGRATION 9

x 1
31. The tangents to the curve y  f  x    2 t dt 36. If f  x   x    x y 2  x 2 y  f  y  dy and
0 0
which are parallel to the bisector of the A x  Bx 2

positive coordinate axes are _____. f  x  , where A, B, C are co-


C
1 1 prime, then _____.
(a) y  x  (b) y  x 
4 4 (a) A  80 (b) B  180
3 3 (c) C  199 (d) All are wrong
(c) y  x  (d) y  x 
2 2 37. If A i is the area bounded by
x
 4t  1  | x  a i |  | y | bi , i  where
32. If f  x     t  dt , then which of the
0
2 1
3 b
following is/are true. ai 1  ai  bi and bi 1  i , a1  0 , b1  32 ,
2 2
(a) f(x) has a local maxima at 0 then _____.
(b) f(x) has a local minima at 0 (a) A 3  128
(c) f(x) has a local maxima value 0
(b) A3  256
(d) f(x) has a minimum value 0
n
33. Area of the region bounded by the curve 8 2
(c) lim  Ai   32 
y  tan x and the lines x  1 and y  0 is n
i 1 3
_____. n
4 2
1 (d) lim  Ai  16
n
i 1 3
(a)  tan 1  x  dx
0 38. Let f :  is a function such that
tan1
(b) tan1  1
xdx
f  2  x   f  2  x  and
 tan
0
f  4  x   f  4  x  and given that
tan1
(c) 1
ydy 2 50
 tan
0 0 f  x  dx  5 then the value of 0 f  x  dx is
1
1
(d)  tan xdx equal to _____.
0 46

3 (a) 125 (b)  f  x  dx
34. If x 2

 e  cos x ln x  1  x dx 

 2
, then  4
51 52
can be _____. (c) 1 f  x  dx (d) 2 f  x  dx
(a) 1 (b) 2
(c) 3 (d) 4 39. Consider the integral
10 
35. The value of the definite integral cos 6 x cos 7 x cos8 x cos9 x
1
I  3 dx
dx 0 1  e 2 sin 4 x
 equals _____.  /2
3
0  x  1  3x  1 If I  k cos8x cos9x dx then
 cos 6 x cos 7 x
 0
(a) 2 1 (b) tan k is _____.
2
 5 (a) 1 (b) 5
(c) tan (d) tan (c) 10 (d) 20
8 12
INDEFINITE AND DEFINTE INTEGRATION 10

40. Consider the integral satisfying f   x   f  x  with


10 
cos 6 x cos 7 x cos8 x cos9 x
I  3 dx f  0  1 and g  x  be a
0 1  e 2 sin 4 x
 /4
function satisfying
If I    cos 6 x cos8 x cos 2 x dx then  is f  x   g  x   x2 and the value
0
1
_____.
(a) 5 (b) 10
of the integral  f  x  g  x  dx
0
is

5 1
(c) 20 (d) equal to a  e  e 2 , then a is
2 2
41. Consider the integral (a) A → p; B → q; C → p; D → r
10 
cos 6 x cos 7 x cos8 x cos9 x (b) A → r; B → q; C → p; D → r
I  dx .
1  e 2 sin
3
4x (c) A → p; B → q; C → q; D → r
0
(d) A → p; B → r; C → p; D → r
The value of I is _____.
43. Match the following:
5 5
(a) (b) Column – I Column – II
4 16
K (p) 0
5 5 (a) Let f  K   and
(c) (d) 2009
32 8 4
42.
g K  
 f  K 
4 4
Column – I Column – II 1  f  K     f  K  
(p) 1
(a) If  f  x  sin x cos xdx 2009

1
and S   g  K  then sum of
K 0
 ln f  x   c and
2 b  a2 
2
digits of S is _____.
  2
(b) f : D  such that (q) 1
a 2 f    1 , then b f  0  is
2 f  x   sin  cos x 
(b) If (q) – 4  log e  2 cos 2 x  3cos x  1 then
2 k
Ik   sin x sin x  dx  k  x2
 1
2 k 
 cos x  2  dx is equal to,
1 4 x1
, then  I k is (where [.]
10 k 1 x1 , x2  D and [.] denotes greatest
denotes the greatest integer integer function.
function) (c) The area of region, (r) 7
(c) The value of the integral 3 represented by x  y  2 and
2
(r)
1 2
x 2  y is S square units then 3S
   2sin x dx is (where [.]
0 = _____.
denotes the greatest integer
function)
(d) Let f  x  be a function (s) 0
INDEFINITE AND DEFINITE INTEGRATION 11

y (s) 6 dy y (s) Family of curves


1 (D) 
  tan x  dx
1
dx x orthogonal to the line
(d) lim y
 _____. joining origin and the point
y 
 1
1 1  x  dx  x, y  .
([.] denotes greatest integer
function) (a) A → s; B → r; C → s; D → p
(a) A → s; B → p; C → s; D → q (b) A → s; B → p; C → q; D → p
(b) A → p; B → s; C → r; D → q (c) A → r; B → r; C → q; D → p
(c) A → s; B → p; C → r; D → q (d) A → s; B → r; C → q; D → p
(d) A → s; B → p; C → r; D → s 46. If k be the value of x at which the function
x
44. Match the following : 3
f ( x)   t  et  1  t  1 t  2   t  3 dt has a
5

Column – I Column – II 1
1
1   local maximum value and sin x  cosecx  k ,
(A)  dx lies in (p) 1, 
0 1 x 4
 2 then, for n  , sin n x  cos ec n x  _____.
1 47. The value of the definite integral
1  
(B)  dx lies in (q)  ln 2,  5 /2 tan 1  sin x 
2
4 x  x 3
 2 1 e
0
 dx is _____.
1  tan 1  sin x  tan 1  cos x 
1     /2 e e
(C)  dx lies in (r)  ,  e
0 1  x6 6 4 2  1 ln x 
48. 1  x ln x
   dx  L then the value of

   x 
2
sin x (s)  , 1
(D)  dx lies in 4  3L
0 x is _____.
e
(a) A → q, s; B → r; C → q, s; D → p, q
(b) A → q, s; B → r; C → s; D → p, q Part-B
(c) A → q, s; B → q, r; C → q, s; D → p, q dx
49. If I   , then I equals _____.
(d) A → q; B → r; C → q, s; D → p, q x a2  x2 4

45. Match the following : 1 1 1


(a) 4  a 2  x 2  3 a 2  x 2   c
Column – I Column – II a x 3x 
(A) (p) Family of straight lines 3
1 1 2 1 2 2 2
dy

x passing through origin (b) 
a4  x
a  x 2

3 x3
 a  x  c
dx y 
3
(B) (q) Family of straight lines 1 1 2 1 2 2
(c) 
a4  x
a  x 2

2 x
 a 2
 x  c
dy y having slope -l 

dx x 1 1 2 1 
(d) 4 
a  x2  3 a2  x2   c
(C) (r) Family of curves having a x 3x 
dy slope at any point  x, y 
 1
dx equal to the negative slope
of the line joining origin and
the point  x, y  .
INDEFINITE AND DEFINTE INTEGRATION 12

x 2  20 3
sin x  x cos x  sin x 
50. If I   dx , then I equals
 x sin x  5cos x 
2 54.   cos2 x  dx  .
e 

_____. (a) esin x  x  sec x   c


x
(a)  tan x  C (b) esin x 1  sec x   c
cos x  x sin x  5cos x 
x (c) esin x 1  sec x   c
(b)  cot x  C
sin x  x sin x  5cos x  (d) esin x  x  sec x   c
1 
(c)  x sin x  5cos x  sin x  7 x  C cos 
55. 0 e cos  sin   d  _____.
(d) none of these
1
(a) 2  (b) 
51. If f :  0, 1   0,   ,  f ( x) dx  1 ,
0

(c) (d) none of these
1 1 2
2 2
 xf  x  dx   and  x f  x  dx   , then  /2
dx
0 0 56. 0 2
 _____.
_____.  4 cos 2 2
x  9sin x 
(a) f(x) = 5
11 13
(b) f(2) = 8 (a) (b)
864 864
(c) f(x) is not possible
17 97
(d) f(x) = (2x – 1) (c) (d)
864 864
52. Let g  x  be a real valued function defined on  2
  57. I n  0 cos n x cos  n x  dx , n  then
the interval   ,  such that
 2 2
sin x I 2001 : I 2002 can be the eccentricity of
et dt   
g ( x)  e2 x    x , 
2
cos x  2t sin x  t 2
 2 2 (a) parabola (b) ellipse
0
(c) circle (d) hyperbola
. The value of g  0  g  0  g  0 is _____. 1
(a) 6 (b) 7 58. I n   x n tan 1 x dx . If
(c) 8 (d) 9 0
1
x a  xb an In  2  bn I n  cn  n  , n  1 then
53. If 1  a  b , then the value of 0 log e x dx is _____.
_____. (a) a1, a2 , a3... are in A.P.
a 1 (b) b1, b2 , b3... are in G.P.
(a) log e
b 1 (c) c1, c2 , c3... are in H.P.
(b) log e  a  1 b  1 (d) a1, a2 , a3... are in H.P.
b 1
(c) log e
a 1
(d) log e  a  1 b  1
INDEFINITE AND DEFINITE INTEGRATION 13

59. Let f  x  be continuous function on  0,1 and Statement 2: If f  x  is continuous on  0,a


if a a
1 1 then, 0 f  x  dx   f  a  x  dx
1
 f  x  dx  1,  xf  x  dx  2 and  x 2 f  x  dx  3. 0
0
0 0 (a) Statement-1 is True, Statement-2 is True;
Then the number of roots of f  x   0 in Statement-2 is a correct Explanation for
Statement-1
 0,1 is _____
(b) Statement-1 is True, Statement-2 is True;
(a) exactly one (b) atleast one Statement-2 is NOT a correct explanation
(c) atmost one (d) zero for Statement-1
 x (c) Statement-1 is True, Statement-2 is False
60. If y  2cot 1    x , then
 2 (d) Statement-1 is False, Statement-2 is True
x 2 dx 1

 e 
x3 3

  63.  e x dx is less than _____.



x 2  4 sin x  4 x cos x 1

2
(a) log cos ec 2 y  cot y  c (a) 2 (b) 2e 
e
(b) log cos ec 2 y  cot 2 y  c 1
(c) e   2 (d) 2e
(c) log cos ec y  cot y  c e
64. Let f :  be continuous function and
(d) log cos ec y  cot y  c
1
bijective, defined such that f    0   0 .
x2  1 
61. Statement-1:  4 2
dx  The area bounded by
0
x  x 1 2 3
y  f  x  , x   , x    t is equal to area
Statement-2: sum of the infinite series
1 1 1 1  bounded by y  f  x  , x   , x    t ,
1      ... 
5 7 11 13 2 3 t  then
(a) Statement-1 is True Statement-2 is True y  f  x  is symmetrical about the point
and Statement-2 is correct explanation of
Statement-1 (a)  0,0 (b)  0, 
(b) Statement-1 is True Statement-2 is True (c)  ,0 (d)  , 
and Statement-2 is not a correct
65. Let f :  be continuous function and
explanation of Statement-1
(c) Statement-1 is True and Statement-2 is bijective, defined such that f    0   0 .
False The area bounded by
(d) Statement-1 is False and Statement-2 is y  f  x  , x   , x    t is equal to area
True
62. Statement 1: bounded by y  f  x  , x   , x    t ,
n 1 m 1
kn km t  then
  1 Ck .    1 Ck 
k0 k  m 1 k  0 k  n 1 f  2  equal to _____.
(where m, n, k are distinct positive integers)
(a) f   (b)  f  
(c) f  0 (d)  f  0 
INDEFINITE AND DEFINTE INTEGRATION 14

66. Let f :  be continuous function and 68. Match the following:


Column-I Column-II
bijective, defined such that f    0   0 . 
(A) n 1
The area bounded by  2 2
 (p)
n 1  2n  1  2n  1 4
y  f  x  , x   , x    t is equal to area 1

n  n2  1 (q)
bounded by y  f  x  , x   , x    t ,
(B) 
n 1 n  n  1
 8
t  then 
2n  1 (r) 1
 (C)  2

1 n 1  n  n  1 
Value of  f t  dt  _____.


1 1
(D)   (s)
(a) 0 (b) 2 n 1  n  n 1  n  n  1 2
(c)  (d) none (t) 
67. Match the following: (a) A → q; B → r; C → r; D → q
(b) A → q; B → r; C → r; D → r
 x  greatest integer less than or equal to x
(c) A → s; B → r; C → r; D → q
Column-I Column- (d) A → s; B → r; C → q; D → q
II 69. Match the following :
(A) (p) 4 Column-I Column-II
 (a) The number of elements in the (p) 1
4
range set of f  x   3  2cos x ,

25 tan 6  x   x   tan 4  x   x  dx  
0   
x    ,  and [.] denotes,
2
x 6  3x5  7 x 4 (q) 2  2 2
(B) I1   x4  2 dx greatest integer function
2
2
(b) Let f :  be a function (q) 2
1
2  x  1  11 x  1  14 satisfying the condition
I2   4
dx
3  x  1 2 f  x  y 3   f  x    f  y   x ,
3

then I 1  I 2  y and f   0  k ,  k  0
(C) 100  tan x 
(r) then lim 
3
2  1 x x 1 2
3   , [.] denotes,
 tan 1    
x 0 f x
  tan 2  dx 
 1  x 1 x  greatest integer function.
(D) (s) 5 10  1
x x   (r) 3
 2
(c)  xe dx  _____. [.]
2 x 1
f  x  cos   x  3  2 sin   x  3  10
 
denotes greatest integer function.
where 0  x  4 . The number of (d) The shortest distance of the (s) 0
points of local extrema are
point  0, 0 from the curve
102
(t) 1 x x
3 y
2
 e  e  is
(a) A → s; B → r; C → q; D → q (a) A → r; B → s; C → s; D → p
(b) A → s; B → s; C → q; D → r (b) A → s; B → p; C → s; D → p
(c) A → s; B → r; C → s; D → q (c) A → r; B → p; C → r; D → p
(d) A → s; B → r; C → r; D → q (d) A → r; B → p; C → s; D → p
INDEFINITE AND DEFINITE INTEGRATION 15

1 1
70. Match the following: 1 1
72. If I1   1  x 2  3 dx and I 2   1  x 3  2 dx ,
Column-I Column- II
0 0
(A) The shortest distance between (p) 6
I1
origin and the curve then  _____.
I2
x 2  y 2  xy  60 is 1
 n  n 
(B) The value of (q) 0 73. If I n     x k 1    2k  1 x k 1  dx, n 
2011 0  k 1  k 1 
dx
  2011 is
 2011 1  x 9  1  x18 , then the value of I9 is ________
1
(C) On  0, 2 the maximum value (r) 3   x    
74. The value of   x 1  cos 
  1 dx ,
2   
of f  x   max  x 2 , x  1, 3 x is 2

where [.] denotes the greater integer function,


(D) Let f :  be given by (s) 40 is _____.
 x   x  when  x  is odd 1
f  x   75. Let I n   x n 1  x 2 dx where n is an integer
 x   x   1 when  x  is even 0
4
In
. Then the value of  f  x  dx  3 is  2 . Then lim  _____.
nI
2 n2

(a) A → s; B → q; C → p; D → q 
1 2 1
ln x 2
(b) A → s; B → q; C → q; D → p
76. Given 
n 1 n
2

6
and 0 1  x 2 dx = 

(c) A → r; B → r; C → q; D → p
then value of  is _____.
(d) A → s; B → r; C → q; D → p  /2 n
n
1 77. Let I n    sin x  cos x  dx  n  2  . Then
71. If I n   x  a  x  dx and 2
0

3 the value of nIn  2  n  1 In  2 is _____.


 4a
I2   x   a  x  2  I1 , then evaluate
7 7  /2 8  7cos x  dx
78. Let I   2
then 56 I equals
   . 0
 7  8cos x 
to _____.
 
2 4
79. If 
3
sin 2 x sin xdx  k  3 cos 2 x cos x dx then
0 0
2
the value of k is _____.
AREA UNDER THE CURVES, DIFFERENTIAL EQUATION 16

Area Under the curves, Differential Equation

Part-A
1. If 1  x  dy
2

dx
4
 x 1  y  , y  0   , then
3
the
6. Let Sn be the area of the figure enclosed by a
n
curve y  x 3 1  x 2  , 0  x  1 and the x-
value of y  8   89 is n
axis. The value of lim  S K is
n 
(a) 4 (b) 2 k 1

(c) 3 (d) 5 1 1
(a) (b)
2. If A is the area of the figure bounded by the 2 3
straight line x = 0 and x = 2 and the curves 1
(c) (d) 1
y  2 x and y  2 x  x 2 , then the value of 4
 3  3 yx 2
6  A  is 7. Solution of the differential y  is
 n2  x3  2 y 4
(a) 4 (b) 8 2 3
(a) x3 y 1  y c
(c) 9 (d) 12 3
3. Let f ( x)  2 x and g ( x)  2 1  x be two 2
(b) x 2 y 1  y 3  c
functions then the area bounded by y = f(x), 3
y = g(x) and x axis is 2
(c) xy 1  y 3  c
1 2 3
(a) sq unit (b) sq unit
3 2 3 2 (d) None of these
1 4 8. If ydx  dy  e x y 4 dy and y = 1 at x   ln 3 ,
(c) sq unit (d) sq unit
2 3 2 then at y = 3, x  ln k , k is
4. The area bounded by the lines y = 2, x = 1, x = (a) 21 (b) 24
a and the curve y = f(x), which cuts the last (c) 27 (d) 81
two lines above the first line for all a  1, is 9. Find the area of the region enclosed between
2 the curve y = 2x4 – x2, the x-axis and the
equal to
3
 32

 2a   3a  3  2 2 then f(x) = ordinates of the points where the curve has
local minima
(a) 2 2 x ; x  1 (b) 2x; x  1
7 7
(a) (b)
(c) 2 x ; x  1 (d) None of these 60 80
5. A variable circle C touches y = 0 and passes 7 7
(c) (d)
through the point (0,1). Let the locus of the 120 160
centre of C is the curve P. The area enclosed
by the curve P and the line x + y = 2 is
14 16
(a) (b)
3 3
15 11
(c) (d)
3 3
AREA UNDER THE CURVES, DIFFERENTIAL EQUATION 17

10. The area bounded by the curves y  2  x  1 , 15. Let f  x   cos 1  sin x   sin 1  cos x  and
y  sin x ; x  0 and x  2 is _____. 3
 x  2
(a) 1  2cos 1 2 2
(b) 2  sin 1 2
 The function f  x   _____.
(c) (d) 1  log 2
2 (a) 4   2 x (b) 2 x  4 
11. The solution of the differential equation (c) 2 x (d) x  
 x cot y  ln  cos x   dy   ln  sin y   y tan x  dx  0 16. Statement-1 : The solution of the equation
is _____. dy 4
1
y x x  6 y  3xy 3 is y  x   3
.
(a)  sin x   cos y  c dx  x  Cx2 
x y
(b)  sin y   cos x   c because
y x Statement-2 : The solution of a linear
(c)  sin x   sin y   c
equation is obtained by multiplying with its
y x
(d)  cot x   cot y  c integrating factor.
x (a) Statement-1 is True, Statement-2 is True;
 
12. Let f  x    t 2  2t  2 dt , where x is the Statement-2 is a correct explanation
0
for Statement-1
set of real numbers satisfying the in-equation
(b) Statement-1 is True, Statement-2 is True;
 
log 2 1  6 x  x 2  8  0 . If f  x    ,   , Statement-2 is NOT a correct explanation
for Statement-1
then maximum value of    is _____. (c) Statement-1 is True, Statement-2 is False
103 104 (d) Statement-1 is False, Statement-2 is True
(a) (b)
3 3 17. Statement 1: If P is a differentiable function of
105 106 dP
(c) (d) x and  3P  6  x  0 and P  0  4
3 3 dx
13. Let f  x  be a continuous function which then P  6 e3 x  2  x  0
takes positive values for x  0 and Statement 2:
x dP d
1  3P  6  e3 x  P  2  e  3 x 
0 f  t  dt  x f  x  with f 1  . Then the
2 dx dx
(a) Statement-1 is True, Statement-2 is True;
value of f  
2  1 is _____. Statement-2 is a correct Explanation for
(a) 1 (b) 2 Statement-1
1 (b) Statement-1 is True, Statement-2 is True;
(c) 4 (d)
4 Statement-2 is NOT a correct explanation
14. The ratio in which Y–axis divides the area of for Statement-1
the region bounded by the curves y  x2  x (c) Statement-1 is True, Statement-2 is False
(d) Statement-1 is False, Statement-2 is True
and y  x 2  1 is _____.
(a) 5 : 13 (b) 7 : 20
(c) 8 : 11 (d) 4 : 5
AREA UNDER THE CURVES, DIFFERENTIAL EQUATION 18

18. Let a solution y  y  x  of the differential 21. Match the following:


Column I Column II
equation x x 2  1 dy  y y 2  1 dx  0 satisfy (A) Solution of the 1
(p) x  e2 y  c1 y  c2
2 differential equation 4
y  2  .
3 y   e 2 y  y   0 is
3

 
Statement 1: y  x   sec  sec1 x   (B) Solution of the (q) y 2  2 xy  3 x3
 6 differential equation
Statement 2: y  x  is given by 2 x  x  y  y  3 y 2  4 xy
1 2 3 1 at y 1  1 is
  1 2
y x x (C) Equation of (r)
(a) Statement-1 is True, Statement-2 is True; orthogonal trajectories 2 x  4 y  sin 2 x  c
Statement-2 is a correct Explanation for of the curve
Statement-1 y  tan x  k is
(b) Statement-1 is True, Statement-2 is True; (D) Solution of the 1
1
 2

Statement-2 is NOT a correct explanation (s) y 2   ce 4 x  


differential equation  4
for Statement-1 y   4 xy  xy 3  0 is
(c) Statement-1 is True, Statement-2 is False (a) A-p; B-q; C-r; D-s
(d) Statement-1 is False, Statement-2 is True (b) A-s; B-p; C-r; D-q
19. The area bounded by the curve y  f 1  x  , x- (c) A-p; B-q; C-s; D-r
axis, x = 0 and x = π, where f(x) is given by (d) A-p; B-s; C-q; D-r
2 a 22. If ye y dx   y 3  2 xe y  dy , y  0   1, then the
f(x) = x + sin x is then the correct
b value of  x  when y  2 is
statement are
(a) a = 2b where  denotes greatest integer function.
(b) a + 2b = 8 23. The segment of the tangent to the curve
(c) 3a =b 2 2

 2 a 
x 3  y 3  16 , contained between x and y
(d)  ,  lies on f(x)
 b 4  axes, has length equal to  2 then the value of
2  is _____.
 dy   1  dy
20. The solution of    2  x    1  0 24. The area bounded by the curves
 dx   4 x  dx
y   4  x 2 , x 2   y 2 and x  y can be
(a) y  x 2  c

1 expressed in the form of   , (,  being
(b) y  ln  x   c, x  0 
2
x relatively prime positive integers) then   
(c) y   c equals to _____.
2
x2
(d) y  c
2
AREA UNDER THE CURVES, DIFFERENTIAL EQUATION 19

dy
1
30. Let f:  , such that
25. If  y   y  x  dx, y  0   1, then y = f(x)
dx 0 f   x   2 f   x   f  x   2 e x and f   x   0 ,
has  x  , then which of the following can be
(a) a maxima correct
(b) minima
(a) f  x    f  x  ,  x 
(c) both maxima and minima
(d) neither maxima nor minima (b) f  x   f  x  ,  x 
1
dy
26. If  y   y  x  dx, y  0   1, then (c) f  3   5
dx 0
(d) f  3  7
The subnormal at x = 0 on curve y = f(x) is
2 2e 31. Solution of the differential equation
(a) (b) y  xy  2 x 2 y 2  dx  x  xy  x 2 y 2  dy  0 is
2e 2
2 3e given by
(c) (d) 1
3e 2 (a) 2log | x |  log | y |  C
xy
Part-B
2 1
27. The graph of y + 2xy + 40|x| = 400 divides (b) 2log | y |  log | x |  = C
xy
the plane into regions. The area of the
bounded region is 1
(c) 2log | x |  log | y |   C
(a) 400 sq. units xy
(b) 600 sq. units 1
(d) 2log | y |  log | x |   C
(c) 800 sq. units xy
(d) 10000 sq. units 32. The area bound by the curve y  f  x  , the
ln  x   x  ; x  I x–axis and the line y 1, where
28. Let f  x   Area x
 0 ; xI 1
f  x  1  f  t  dt ,  x  0 is _____.
enclosed by curves y = f(x), y = f(|x|), x 1
5  x  5 and y = 0 is  1  1
(a) 2 1   (b) 2 1  
(a) 5(1 – ln2) (b) 0  e  e
(c) 5ln2 (d) none of these  1  1
(c) 1   (d) 1  
29. The solution of the differential equation  e  e
 2 x 2  3 y 2  7  x dx   3 x 2  2 y 2  8 y dy  0 33. The solution of the differential equation
dy 1
is _____.  is, (‘c’ being an
4 3 dx xy ( x sin y 2  1)
2


(a) x 2  y 2  3   c  x2  y 2  1 arbitrary constant).
(b)  x 2  y 2  3  c  x 2  y 2  1
5

(a) x 2 cos y 2  sin y 2  2ce  y
2

2
(b) x  cos y   4c
2
5 2 2
(c)  x 2  y 2  3  c  x 2  y 2  1  sin y 2  2e  y

(c) y  cos x 2


2
2 2
(d) None of These  sin x 2  2ce  y

(d) y  cos x   4c
2
2 2
 sin x 2  ce  y
AREA UNDER THE CURVES, DIFFERENTIAL EQUATION 20

34. If particular solution of 40. Given f ( x)  x3  x  2, f : R  R, area of


p  cos px sin y  sin px cos y where region bounded by y  f  x  , y  f 1  x  and
 dy  2 1 x2  c 5
 p   is y  a x  b  sin then xy = 0 in 1st quadrant is A, where A  3t  ,
 dx  x 2
bc 1b
where t  a and 3(a + b) = n; a, b, n  N and
evaluate
a a is prime. Number of ways of putting n alike
35. The solution of apples and n alike oranges in six baskets if
2 2 2
x dy  y dx  xy  x  y  dy  0 is each basket can hold atmost five fruits is
(a) 20C5 – 614C5 + 158C5
1 1 y2
ln    C , then the value of k is (b) (20C5 - 6 14C5 + 15 8C5)2
x y k
(c) (20C5 - 6 14C5 + 15 8C5)2 - 56
______. (d) none of these
36. Area enclosed by the curve
4  x  y  2  x  y  is (in sq. units)
2 2

37. f be a real valued differential function in


 0,   such that f 1  0 . If y intercept of the
tangent at any point p  x, y  on the curve

 
y  f  x  is equal to  x 2  y 2 , then value

of f  1 is equal to _____.


38. Suppose a solution of the differential equation

 xy3  x2 y 7  dy
dx
1 satisfies the initial

1
condition y    1 . Then the value of
4
5 dy
  when y  1 , is _____.
4 dx
39. Given f ( x)  x3  x  2, f : R  R, area of
region bounded by y  f  x  , y  f 1  x  and
5
xy = 0 in 1st quadrant is A, where A  3t  ,
2
1b
where t  a and 3(a + b) = n; a, b, n  N and
a is prime. Difference of last two digits of
11
a a 36 is
(a) 0 (b) 1
(c) 2 (d) 3
PROBABILITY 21

Probability
Part-A 15 17
(a) (b)
64 64
1. The curve y  f  x  passes through the origin 21 19
(c) (d)
dy 3 64 64
and satisfies  ydx  11 . If a and b are
dx 0
5. A natural number is chosen at random from
the first 100 natural numbers. The probability
chosen randomly from the set S  1, 2, ..., 10 100
that x   50 is _____.
with replacement, the probability that x
y  f  x  passes through  a, b  is equal to (a) 1 (b) 11
10 50
_____.
1 19 (c) 1 1 (d) none of these
(a) (b) 20
20 20
6. Urn A has 5 black balls and one white ball.
1 12
(c) (d) Urn B has 6 black balls. If two balls are
13 13 randomly picked up from urn A and
2. Five numbers out of 1, 2, 3, ..., 9 are written transferred to urn B and then two balls are
randomly on four vertices and the centroid of randomly picked up from urn B and
a regular tetrahedron. The probability that transferred to urn A, then the probability that
centroid of the tetrahedron is 6 is _____. the white ball is now in urn A is _____.
5 4
(a) (b) (a) 5 (b) 1
9 9 6 3
8 1 2 3
(c) (d) (c) (d)
9 9 3 4
3. ‘A’ tosses a fair coin. If it shows a tail ' A ' is 7. Mr. RKT travel from Delhi to Muscat with
asked to roll a fair die and A ' s score is the change of airplanes in Chennai, Dubai and
number that die shows. If the coin shows a Sharjah respectively. He has one piece of
head, ‘A’ is asked to toss five more coins and luggage. At each stop, the luggage is
A ' s score is total number of heads shown transferred from one airplane to another. At
(including the first coin). If ' A' tells you that the airport in Delhi there is a probability of 5
his score is only 3 then the probability that ‘A’ % that the luggage is not placed in the right
rolls a die is plane. This probability is 10 % at the airport
23 1 in Chennai, 20 % at the airport in Dubai and
(a) (b) 10 % at the airport in Sharjah. If Mr. RKT’s
96 6
8 7 luggage does not reach Muscat with him, then
(c) (d) the probability that it was lost at the airport of
23 44
4. Five cards are drawn randomly one by one Dubai is _____.
with replacement from a well shuffled pack of (a) 855 (b) 779
1922 1922
52 playing cards. The probability that these
365
cards will contain cards of each suit is _____. (c) (d) none of these
1922
PROBABILITY 22

8. Statement 1: If A,B are two events such that called as 2nd, 3rd, ... generations. If it is known
3 5 that the amoeba population died in 2 seconds,
P  A   and P  B   then the maximum
4 8 the probability that mother amoeba died after
1 the 1st second is _____.

value of P A  B is 8 11 21
(a) (b)
Statement 2: If A,B are any two events of a 32 32
sample space then the maximum value of 8
(c) (d) none of these
P  A  B  min P  A , P  B  11
11. A fair die is rolled four times. Find the
(a) Statement-1 is True, Statement-2 is True;
probability that each number is no smaller
Statement-2 is a correct Explanation for
than the preceding number.
Statement-1
7 1
(b) Statement-1 is True, Statement-2 is True; (a) (b) 
72 3
Statement-2 is NOT a correct explanation for
1 11
Statement-1 (c)  (d)
(c) Statement-1 is True, Statement-2 is False 3 72
12. If three numbers are chosen randomly from
(d) Statement-1 is False, Statement-2 is True
9. Let S be the set of all functions f : A  B , A the set 1, 3, 3 ,...., 3  without
2 n
replacement,
and B being finite sets having the same then the probability that they form an
number of elements. Let P be the probability increasing geometric progression is
that the function is one-one. 3
(a) if n is odd
Statement-1: The probability that the function 2n
is on to is P.
Statement-2: A function f : A  B is one-one 3
(b) if n is even
and n  A  n  B is also on to ( n  x  denotes 2n
3n
cardinal number of the set) (c) if n is even
2  n 2  1
(a) Statement-1 is True, Statement-2 is True;
Statement-2 is a correct Explanation for 3n
(d) if n is odd
Statement-1 2  n 2  1
(b) Statement-1 is True, Statement-2 is True; 13. Let ‘A’ and ‘B’ be independent events such
Statement-2 is NOT a correct explanation for 1 3
that P  A  , P  A  B  and
Statement-1 2 25
(c) Statement-1 is True, Statement-2 is False 8
P  A  B  . Then which of the following
(d) Statement-1 is False, Statement-2 is True 25
10. An amoeba either splits into two, or remains statements is/are correct ?
the same or eventually dies out after every (a) (P (A))2 + (P(B))2 = 1
1 1 (b) P(A) satisfies 25x2 – 20 x + 3 = 0
second with probabilities respectively ,
2 4 (c) P(A) satisfies 5x2 – 20 x + 3 = 0
1 (d) (P (A))2 + (P(B))2 = 2
and . The initial amoeba let be called as
4
mother amoeba and after every second the
amoeba, if is distinct with previous one be
PROBABILITY 23

1 4 p 1 p 1 2 p which exactly one trio is of the same family is


14. If , and are the
4 3 2 _____.
probabilities of there mutually exclusive 125 25
(a) (b)
events then p may be: 1547 221
1 1 30 60
(a) (b) (c) (d)
8 2 1547 1547
3 5 18. There are six families each consisting of a
(c) (d)
8 8 husband, a wife and a child. A group of three
15. A bag contains n (white and black) balls. It is consisting of a man, a woman and a child is
given that the probability that the bag contains said to form a trio. If 3 persons are selected,
exactly r white balls is directly proportional to the probability that there will be exactly 2
r  0  r  n  . A ball is drawn at random and is persons of the same family will be _____.
45 45
found to be white. The probability that there is (a) (b)
68 136
only one white ball in the bag is p, then:
27 18
1 (c) (d)
(a) p  if n  5 91 119
55
19. In a game of chess between two players A
6
(b) p  if n  5 and B, either A wins or B wins or game ends
55
in a drawn. Probability of winning the game
1
(c) p  if n  6 1 1
91 by A and B are and respectively. They
2 3
13
(d) p  if n  6 agree to play a chess tournament consisting of
93
3 games. The probability that 2 games end in
16. Let X be a set containing n elements. If two
a draw equals to
subsets A and B of X are picked at random,
5 5
the probability that A and B have the same (a) (b)
36 72
number of elements is: [A may or may not
25 25
have same element(s)] (c) (d)
2n
72 36
Cn
(a) 20. In a game of chess between two players A
22 n and B, either A wins or B wins or game ends
1 in a drawn. Probability of winning the game
(b) 2 n
Cn 1 1
by A and B are and respectively. They
1.3.5.....  2n  1 2 3
(c)
2n.  n ! agree to play a chess tournament consisting of
3 games. The probability that A and B wins
3n
(d) alternately equals to
4n
5 5
17. There are six families each consisting of a (a) (b)
36 72
husband, a wife and a child. A group of three
25 25
consisting of a man, a woman and a child is (c) (d)
72 36
said to form a trio. If 6 persons are selected,
the probability that there will be two trios in
PROBABILITY 24

21. In a game of chess between two players A and two faces marked with 2 and one face marked
B, either A wins or B wins or game ends in a with 3. Die B has one face marked with 1,
drawn. Probability of winning the game by A two faces marked with 2 and three faces
1 1 marked with 3. Both the dice are thrown
and B are and respectively. They agree to
2 3 randomly once. If E be the event of getting
play a chess tournament consisting of 3 games. sum of the numbers appearing on top faces
The probability that B wins at least 1 game, equal to x and if P(E) be the probability of
equals to event E, then find the value of x when P(E) is
5 19 maximum.
(a) (b)
9 27 24. 3 coins are thrown at one time and we remove
11 25 those coins which show tails. Then we throw
(c) (d)
36 72 the remaining coins at one time and we
22. Match the following remove those coins which show tails. This is
Column-I Column-II done repeatedly until all of coins are
(A) Number of ways in which (p) 720 removed. If the probability that the trials are
all the letters of word ended in the 2nd round is P, then the value of
“RESONANCE” can be 10P is _____ (where [.] denotes the greatest
arranged such that vowels and integer function).
consonants occurs alternately
25. Suppose n   4 people are seated at a round
is
(B) If probability of scoring a (q) 210 table. If three people are selected at random,
total of 20 in 4 throws of a die and p n is the probability that atleast two of
is p then 65 p is them are sitting next to each other, then
(C) If probability that product (r) 370 a a
p7  (when is expressed in lowest
of four whole numbers ends in b b
‘5’ is q then 104 q is less than terms) where a  b 
(D) The variance of a binomial (s) 360 Part-B
random variable is 180. The 26. There are four seats numbered 1, 2, 3, 4 in a
minimum numbers of trials room and four persons having tickets
possible for the experiment are corresponding to these seats (one person
(E) If sum of roots of equation having one ticket). Now the person having the
ticket number 1, enters into the room and sits
12 x 4  56 x 3  89 x 2  56 x  12  0 on any of the seat at random. Then the person
9  having the ticket number 2, enters in room. If
be A , then value of  A ! is
7  his seat is empty then he sits on his seat
otherwise he sits on any of the empty seat at
(a) A-p; B-q; C-p, r; D-p; E-p random. Similarly the other persons sit.
Probability that the person having ticket
(b) A-r; B-p; C-s; D-r
numbered 4 gets the seat number 4 is
(c) A-s; B-q; C-p; D-r 1 1
(d) A-r; B-p; C-q; D-r (a) (b)
2 4
23. There are two dice A and B both having six 1 1
faces. Die A has three faces marked with 1, (c) (d)
8 16
PROBABILITY 25

27. Statement 1: Rahul and Grover are weak (c) Statement-1 is True, Statement-2 is False
students in mathematics and their chances of (d) Statement-1 is False, Statement-2 is True
1 1 29. Players P1 , P2 , P3 ,..., Pm , of equal skill, play a
solving a problem correctly are and
8 12 game consecutively in pairs as
respectively. They are given a question and PP
1 2 , P2 P3 , P3 P4 ,... Pm1 Pm , Pm P1 ,.... , and any
they obtain the same answer, if the probability
player who wins two consecutive games (i.e. i
1
of a common mistake is , then the and (i + 1)th game) wins the match. If the
1001
chance that the match is won at the rth game
13
probability that the answer was correct is . is k then:
14
1
Statement 2: If E1 and E 2 are mutually (a) k  , if r  5
8
exclusive and exhaustive events with non zero 5
(b) k  , if r  5
probabilities of a random experiment and E is 32
any other event of the same experiment, then 3
(c) k  , if r  6
 E  32
 E1 
p   p E1
 E1    5
  (d) k  , if r  6
p   64
E
   E   E 
 
p   p E1  p 
 E1   E 2   
p E2 30. A bag contains 6 different balls of three
    colours white, green and red (at least one ball
of each colour. The probability that the bag
(a) Statement-1 is True, Statement-2 is True; contains 2 balls of each colour is
Statement-2 is a correct Explanation for 1 1
(a) (b)
Statement-1 3 7
(b) Statement-1 is True, Statement-2 is True; 1 1
(c) (d)
Statement-2 is NOT a correct explanation for 9 10
Statement-1 31. A bag contains 6 different balls of three
(c) Statement-1 is True, Statement-2 is False colours white, green and red (at least one ball
(d) Statement-1 is False, Statement-2 is True of each colour. Three balls are picked up at
2

28. If P B   P A  B  then (A and B are two random and found to be one of each colour.
The probability that the bag contained 4 red
non empty events of a random experiment) balls
5 1 1 1
Statement-1: P  A  B   (a) (b)
2 10 14
Statement-2: For any three non-empty sets 1 7
(c) (d)
 
A,B,C n A B C  n A B  n AC 7 25
(a) Statement-1 is True, Statement-2 is True;
Statement-2 is a correct Explanation for
Statement-1
(b) Statement-1 is True, Statement-2 is True;
Statement-2 is NOT a correct explanation for
Statement-1
PROBABILITY 26

32. A bag contains 6 different balls of three 2


(c) P100 , P101  (d) None of these
colours white, green and red (at least one ball 3
of each colour. Three balls are picked up at 36. Match the following
random and found to be one of each colour. Column-I Column-II
The probability that the bag contained equal (A) A and B play a game with a pair 3
(p)
of dice each. A has a “good throw” 8
number of white and green balls is
if the sum on A’s dice is 7, while B
3 1 has a “good throw” if the sum on B’s
(a) (b)
14 10 dice is 4. In each round they throw
2 7 simultaneously. If one of them has a
(c) (d)
7 25 “good throw” and the other does not,
33. A player tosses a coin and scores one point for the player having a “good throw” is
declared a winner, otherwise they
every head and two points for every tail that
throw again. The probabilitythat A
turns up. He plays on until his score reaches wins is
' n ' or passes n. Pn denotes the probability of (B) Suppose A and B shoot 2
(q)
getting a score of exactly n. The value of Pn is independently until each hits his 7
equal to 3
target. They have probabilities
5
1
(a)  Pn 1  Pn  2  5
2 and respectively of hitting the
7
1
(b)  2 Pn 1  Pn  2  targets at each shot. The probability
2
that B will require more tries than A
1
(c)  Pn 1  2 Pn  2  to hit the target is
2
(C) The probability of a missile 6
(d) None of these (s)
being destroyed before hitting the 31
34. A player tosses a coin and scores one point for
1
every head and two points for every tail that target is . If the missile is not
turns up. He plays on until his score reaches 3
' n ' or passes n. Pn denotes the probability of destroyed, the probability of hitting
getting a score of exactly n. The value of 3
the target is .
1 4
Pn  Pn 1 is equal to (D) The probability of a bomb 3
2 (t)
1 2 1 16
(a) (b) hitting a bridge is . At least two
2
2 3
direct hits are needed to destroy it. If
1 n is the least number of bombs
(c) 1 (d)
4 required so that the probability of the
35. A player tosses a coin and scores one point for bridge getting destroyed is greater
every head and two points for every tail that 2
turns up. He plays on until his score reaches than 0.9, then the value of is
n
' n ' or passes n. Pn denotes the probability of (a) A→s; B→s; C→p; D→q
getting a score of exactly n. Which of the (b) A→p; B→s; C→p; D→q
following is not true? (c) A→t; B→s; C→p; D→q
2 2 (d) A→r; B→s; C→p; D→q
(a) P100  (b) P101 
3 3
PROBABILITY 27

37. n whole numbers are randomly chosen and adjacent boxes so that
multiplied. each box contain one
Column-I Column-II card, is ‘ k ’, then
(A) The probability 8n  4 n (d) A box contains 4 1 2
(p) (s)  k 
that the last digit is 1, 10n balls which are either 7 3
3, 7, or 9 is red or black, 2 balls
(B) The probability 5n  4 n are drawn and found
(q)
that the last digit 2, 4, 10n to be red if these are
6 or 8 is replaced, then the
(C) The probability 4n probability that next
(r)
that last digit is 5 is 10 n draw will result in a
(D) The probability 10n  8n  5n  4n red ball is ‘ k ’, then
(s)
that the last digit is 10n 1 1
(t) k
zero is 6 2
(t) none of these (a) (a) p, q (b) s, t; (c) q, s, t; (d) (r)
(a) A→r; B→p; C→q; D→s (b) (a) p, s (b) s, t; (c) q, s, t; (d) (r)
(b) A→s; B→p; C→q; D→r (c) (a) p, r (b) s, t; (c) q, s, t; (d) (r)
(c) A→r; B→q; C→p; D→s (d) (a) p, s (b) s, t; (c) q, s, r; (d) (t)
(d) A→q; B→p; C→r; D→s 39. A blind man rolls a fair die marked 1, 2, 3, 4,
38. Match the following 5 and 6 on its six faces. After the first rolling,
Column-I Column-II he erases the number on one of the six faces
(a) An urn contains 2 3 turning up and marks it with 1. He rolls the
(p)  k 
five balls, two balls 5 4 die again and erases the number on the face
are drawn and are turning up and marks it with 2. He repeats
found to be white. If this exercise four more times and marks 3, 4,
probability of all the 5 and 6 on faces turning up in subsequent
balls in urn are white trials. If probability that the face initially
is ‘ k ’, then marked with 4 still has 4 marked on it, after
(b) Out of 15 1 1 66 p
(q)  k  the sixth trial is p then find value of
consecutive integers 6 3 4205
three are selected at 40. Five persons play with their car keys such
random, then the that after the game none leaves with his own
probability of the sum key and everybody has exactly one key. It is
is divisible by 3 is ‘ k not possible for all five of them to meet at one
’, then place again. If the probability that they can
(c) If 3 cards are 2 still have their right key back assuming each
(r)  k  1
placed at random and 3 one of them meets one or more friends at
independently in 4 most once is P, then 11P is equal to _____.
boxes lying in a
straight line. Then the
probability of the
cards going into 3
PROBABILITY 28

41. Six faces of a cube are numbered randomly 43. Two friends decide to meet at a spot between
1, 2, 3, 4, 5, 6 . The probability that 2 p.m. and 3 p.m. whosoever arrives first
faces 1 and 6, 6 and 3, 3 and 1 will share an agrees to wait for 15 minutes for the other.
m p
edge is (in its lowest form). m  n  The probability that they meet is (where
n q
_____. H.C.F ( p , q  1 ) then q  p  ___ )
42. Two natural numbers x and y are chosen at
random. If P is the probability that x2  y 2 is
divisible by 5, then the value of 25P is _____.
VECTORS & 3D COORDINATE GEOMETRY 29

Vectors & 3D Coordinate Geometry

Part-A 1
(c) pq r
3
1. If the orthocentre of the triangle whose 1
vertices are A(a, 0, 0), B(0, b, 0) and C(0, 0, c) (d)  2 p  q  r 
3
k k k 4. A ray of light emanating from the point
is  , ,  , then k is equal to _____.
a b c
1
 
source P iˆ  3 ˆj  2kˆ and travelling parallel
 1 1 1 x  2 y z 1
(a)  2  2  2  to the line   is incident on the
a b c  1 3 2
1
1 1 1 plane x  3 y  3 z  0 at the point Q. After
(b)     reflection from the plane the ray travels along
a b c
the line QR. It is also known that the incident
1 1 1
(c) 2  2  2 ray, reflected ray and the normal to the plane
a b c at the point of incident are in the same plane.
1 1 1 The position vector of Q is _____.
(d)  
a b c (a) 3iˆ  15 ˆj  6kˆ
2. The equation of the line passing through A(1,
x y 1 z  2 (b) 3iˆ  6 ˆj  3kˆ
0, 3), intersecting the line  
2 3 1 (c) 3iˆ  6 ˆj  3kˆ
and which is parallel to the plane x + y + z = 2  9iˆ  15 ˆj  18kˆ 
is _____. (d)  
 2
3x  1 2 y  3 2 z  5  
(a)   5. A ray of light emanating from the point
2 3 1
(b)
x 1 y  0 z  3
 
 
source P iˆ  3 ˆj  2kˆ and travelling parallel
2 3 1 x  2 y z 1
x 1 y 1 z  3 to the line   is incident on the
(c)   1 3 2
2 / 3 3 / 2 1 / 2 plane x  3 y  3 z  0 at the point Q. After
3x  1 2 y  3 6 z  13 reflection from the plane the ray travels along
(d)  
2 3 5 the line QR. It is also known that the incident
3. If p , q , r are three mutually perpendicular ray, reflected ray and the normal to the plane
vectors of the same magnitude and if a vector at the point of incident are in the same plane.
x satisfies the equation The vector equation of line containing QR is
b  c  x   c  a  y   a  b  z  0 _____.
 c  a  x   a  b y  b  c  z  0  9 15 ˆ 
(a) r   iˆ 
 2 2 

j  9kˆ    11iˆ  33 ˆj  62kˆ 
 a  b x  b  c y   c  a  z  0 , then vector
is equal to _____.   
(b) r  3iˆ  15 ˆj  6kˆ   3iˆ  7 ˆj  2kˆ 
1
(a)  p  q  2r  (c) r   3iˆ  6 ˆj  3kˆ    15iˆ  37 ˆj  10kˆ 
2
1
(b)  p  q  r  (d) r   iˆ  ˆj  kˆ     iˆ  ˆj  kˆ 
2
VECTORS & 3D COORDINATE GEOMETRY 30

6. A ray of light emanating from the point source 10. Let a and b be two vectors of equal
 
P iˆ  3 ˆj  2kˆ and travelling parallel to the magnitude of 5 units each. Let p and q be

x  2 y z 1 vectors such that p  a  b and q  a  b . If


line   is incident on the plane 1
1 3 2
x  3 y  3 z  0 at the point Q. After reflection 
p  q  2   a . b   , then value of  is
2 2

from the plane the ray travels along the line _____.
QR. It is also known that the incident ray, (a) 25
reflected ray and the normal to the plane at the (b) 125
point of incident are in the same plane. (c) 625
The equation of the plane in Cartesian form is (d) none of these
_____. 11. Equation of plane containing the line
(a) 5 x  2 y  z  3  0 x y
  1, z  0 and parallel to the line
(b) 3 x  y  6 a b
x z
(c) 5 x  y  z  6   1, y  0 is _____.
c d
(d) x  y  z  6 a c
(a)  x  a   y  z  0
7. A If a  iˆ  ˆj  2kˆ , b  iˆ  2 ˆj  2 kˆ and c is b d
a vector such that  a  b b c c  a  has a c
(b)   x  a   y  z  0
b d
maximum value, where c  1 then the value a c
(c)  x  a   y  z  0
2 b d
 
of a  b  c is _____.
a c
(d)   x  a   y  z  0
(a) 0 (b) 1 b d
4 12. The position vector 2iˆ  ˆj  kˆ is rotated
(c) (d) none of these
3 about the origin (O) by an angle of 90 such
8. Let A, B, C, D be distinct points on a circle that it crosses the X-axis. The new position
centred at O. If there exists non-zero real vector after rotation is _____.
numbers x and y such that x OA  y OB = 
(a) 2 iˆ  ˆj  kˆ 
x OB  y OC = x OC  y OD = x OD  y OA , then (b) 2  iˆ  ˆj  kˆ 
_____. (c)  2  iˆ  ˆj  kˆ 
(a) ABCD is a rectangle
(d)  2  iˆ  ˆj  kˆ 
(b) ABCD is a square
(c) ABCD is a rhombus 13. Let P denote the plane consisting of all points
(d) nothing can be said that are equidistant from A   4, 2, 1 and
9. Let O be an interior point of such that . Then B  2,  4, 3 . Let Q denote the plane
the ratio of area of to the area of is _____. x  y  cz  1 , where c  . If the angle
3 between the planes P and Q is 45 , then the
(a) 2 (b)
2 product of all possible values of c is _____.
5 (a) –17 (b) –2
(c) 3 (d) 24
3 (c) 17 (d)
17
VECTORS & 3D COORDINATE GEOMETRY 31

14. Let P denote the plane consisting of all points 18. The point of intersection of the plane

that are equidistant from A   4, 2, 1 and r. 3 iˆ  5 j  2 k  6 with the straight line
 
B  2,  4, 3 . Let Q denote the plane passing through the origin and perpendicular
to the plane 2 x  y  z  4 is
x  y  cz  1 , where c  . If the line L with
(a) 1,  1,  1
x 1 y  2 z  7
equation   intersects the
1 3 1 (b)  1,  1, 2 
plane P at a point R  x0 , y0 , z0  , then (c)  4, 2, 2 
x0  y0  z0 is equal to _____.  4 2 2 
(d)  , , 
(a) 12 (b) –15 3 3 3 
(c) 13 (d) –11 19. A line is drawn from the point P 1,1,1 and
15. Let a , b and c be three unit vectors such that perpendicular to a line with direction ratios
1,1,1 to intersect the plane x  2 y  3 z  4
a b c  3 . If  a  b .b  c  + at Q. The locus of point Q is
 b  c  .  c  a  +  c  a  .  a  b    , then the (a) 
x y 5 z 2

maximum value of  is _____. 1 2 1
(a) 0 (b) 1 x y 5 z  2
(b)  
(c) 3 (d) 2 2 1 1
(c) x  y  z
16. Let r be the position vector of a variable
x y z
point in the Cartesian OXY plane such that (d)  
2 3 5
 
r . 10 ˆj  8iˆ  r  40 and     
20. Given | a || b | 1 and | a  b | 3 . If c is a
    

P1  max r  2iˆ  3 ˆj ,2
 
vector such that c  a  2b  3 a  b , then

P  min  r  2iˆ  3 ˆj  .
2  
c .b is equal to
2
1 1
(a)  (b)
P2 is equal to _____. 2 2
3 5
(a) 9 (b) 2 2  1 (c) (d)
2 2
(c) 6 2  3 (d) 9  4 2 21. Let L1 and L2 be the lines
17. Let r be the position vector of a variable 
r  2 i  j  k   i  2 k
    and
point in the Cartesian OXY plane such that

 
r . 10 ˆj  8iˆ  r  40 and r  3 i  j   i  j  k . If the plane 
   
which contains L1 and parallel to L 2 meets

P1  max r  2iˆ  3 ˆj ,2

the coordinate axes at A, B and C


P  min  r  2iˆ  3 ˆj  .
2
respectively, then the volume of the
2
tetrahedron OABC is
P1  P2 is equal to _____. 4 4
(a) (b)
(a) 2 (b) 10 9 3
2 2
(c) 18 (d) 5 (c) (d)
9 3
VECTORS & 3D COORDINATE GEOMETRY 32

22. A The equations of three planes are  2 


(d) cos 1  
b  c  x   c  a  y   a  b z  0  186 
 c  a  x   a  b y  b  c  z  0 26. If in a triangle ABC, BC 
u v
 and
 a  b x  b  c  y   c  a  z  0 u v
when a, b, c are distinct real numbers. Then 2u
AC  , where u  v , then _____.
(a) The planes are parallel to each other but u
not identical (a) 1  cos 2A  cos 2B  cos 2C  0
(b) The planes meet exactly at one point.
A B C
(c) The planes intersect in a line. (b) 8sin   .sin   .sin    1
2 2 2
(d) The planes are identical.
(c)  ABC is right angled
23. Two lines whose equations are
(d)  ABC is equilateral
x  3 y  2 z 1 x2 y 3 z 2
  and   27. Let a and c be unit vectors such that
2 3  3 2 3
lies in the same plane, then a  b  2 a  c , where b  4 . The angle
1
The value of sin  sin    1
between a and c cos 1   . If
is
(a) 3 (b)   3 4
(c) 4 (d)   4 b  2c  ka , then k is equal to _____.
24. Two lines whose equations are 1 1
(a) (b)
x  3 y  2 z 1 x2 y 3 z 2 3 4
  and  
2 3  3 2 3 (c) –4 (d) 3
lies in the same plane, then 28. Let a  2iˆ  ˆj  kˆ , b  iˆ  2 ˆj  kˆ ,
Point of intersection of the lines lies on
(a) 3 x  y  z  20 c  iˆ  ˆj  2 kˆ be three vectors. A vector in

(b) 2 x  y  z  25 the plane of b and c whose projection on


(c) 3 x  2 y  z  24 2
a is of magnitude can be _____.
(d) x  y  z 3
25. Two lines whose equations are (a) 2iˆ  3 ˆj  3kˆ (b) 2iˆ  3 ˆj  3kˆ
x  3 y  2 z 1
and
x2 y 3 z 2 (c) 2iˆ  ˆj  5kˆ (d) 2iˆ  ˆj  5kˆ
   
2 3  3 2 3 29. A ray of light emanating from the point A(1,
lies in the same plane, then 0, 1) reflects from the plane mirror
The angle between the plane containing both 2 x  2 y  z  1 at the point B(0, 0, 1). The
the lines and the plane 4 x  y  2 z  0 is equation of the reflected ray is _____.
equal to (a) 8  72 x  9 y  8  18 z  10
 (b) 8 x  y  2 z  2
(a)
3
(c) 8 x  y  2 z  2

(b) (d) 8  72 x  9 y  8  2 z  10
2

(c)
6
VECTORS & 3D COORDINATE GEOMETRY 33

30. If the line


x y z
intersects line 35. If d is a unit vector and a , b , c are three
 
1 2 3 non-coplanar vectors, then the value of
2
3 x  3 1  2  y  z  3 =  a . d  b  c    b . d   c  a    c . d  a  b 
1 a b c 

2
 6 2 x  3 1  2   y  2 z  , then the point
is equal to _____.
 ,  ,1 lies on the plane _____. 36. If points (0, 0, 9), (1, 1, 8), (1, 0, 7) and
(a) 2 x  y  z  4 (b) x  y  z  2  2, 2,   are coplanar, then   _____.
(c) x  2 y  0 (d) 2 x  y  0 37. Let  pˆ  q   pˆ   pˆ . q  q =
31. If r is a vector satisfying r  i  j  2 k  i  j
   x 2  y 2  q  14  4 x  6 y  pˆ , where p̂ and

then r can be q are two non-collinear vectors (and p̂ is a


unit vector) and x, y are scalars. Find the
(a)  (b) e
1 1 value of  x  y  .
(c) (d)
3 5 38. If the length of shortest distance between the
32. A Three planes x  y  z 1  0 , 1 1
two lines  x  1   y  3  z  2 and
 x 3 y  2 z 3  0 , 3x  y  z 2  0 2 4
s
form a triangular prism if 3 x  y  2 z  4  0  2 x  y  z  1 is ,
(a)    4 (b)    3 14
then the value of s is ______.
(c)   4 (d)   4
39. The perpendicular distance of the point
33. Consider the planes P1 : 2 x  y  z  4  0,
1,  2,3 to plane containing the straight line
P2 : y  z  4  0 and P3 : 3 x  2 y  z  8  0.
x y z
Let L1 , L 2 , L 3 be the lines of intersection of   and perpendicular to the plane
2 3 4
the planes P2 and P3 , P3 and P1 , and P1 x y z
containing the lines   and
and P2 respectively. Then, 3 4 2
x y z
(a) Atleast two of the lines L1 , L 2 and L 3 are   is
4 2 3
non–parallel 40. If the length of the shortest distance between
(b) Atleast two of the lines L1 , L 2 and L 3 are x2 y 1 z
the lines   ;
parallel 2 3 4
(c) The three planes intersect in a line 2 x  3 y  5 z  6  0  3 x  2 y  z  3 is K , then
(d) The three planes form a triangular prism  3 K  is equal to (where [.] denotes greatest
34. The coordinates of a point on the line integer function)
x y 1 z 1 41. aˆ , bˆ, cˆ are unit vectors, not all collinear such
  at a distance 11 from the
2 3 1 that aˆ  cˆ  cˆ  bˆ and bˆ  aˆ  aˆ  cˆ , ,  are
point (1, –1, 1) are
angles between aˆ and bˆ , bˆ and cˆ , cˆ and aˆ
(a) (2, –4, 2) (b) (1, –2, 4)
respectively. if
 10  2   1 2 3 
(c)  , ,  (d)  , ,  4  cos   cos   cos    t  0 then value of
 7 7 7 7 7 7
t is
VECTORS & 3D COORDINATE GEOMETRY 34


42. A plane P contains the line r  2iˆ  t ( ˆj  kˆ) (c) Statement – 1 is True, Statement – 2 is
 False
and is perpendicular to the plane r .(iˆ  kˆ) =
(d) Statement – 1 is False, Statement – 2 is
3. The position vector of the point where the True

plane P meets the line r  t (2iˆ  3 ˆj  kˆ) is - 47. Statement 1: If x, y , z  R and
 2iˆ   ˆj  kˆ  then  is 3 x  4 y  5 z  10 2 then the least value of
43. Let G1, G2 and G3 be the centroids of the x 2  y 2  z 2 is 4.
triangular faces OBC, OCA and OAB of a Statement 2: If  is a given plane and ' P ' is
tetrahedron OABC. If V1 denotes the volume a given point then the point on plane which is
of tetrahedron OABC and V2 that of the nearest to ' P ' is the foot of the perpendicular
parallelepiped with OG1, OG2 and OG3 as from ' P ' to the plane.
three concurrent edges, then the value of (a) Statement-1 is True, Statement-2 is True;
4V1/V2 is (where O is the origin) Statement-2 is a correct Explanation for
  
44. Let a  2iˆ  ˆj  2kˆ, b  iˆ  ˆj , c is a vector Statement-1
    (b) Statement-1 is True, Statement-2 is True;
such that a.c  c , c  a  2 2 and angle
Statement-2 is NOT a correct explanation
      
between a  b and c is . If a  b  c is
3
  for Statement-1
(c) Statement-1 is True, Statement-2 is False
6 3 (d) Statement-1 is False, Statement-2 is True
, then the value of ‘λ’ is
 48. The shortest distances between an interior
45. A line drawn through the point A  1,1,1 diagonal of a rectangular parallelepiped (box)
drawn parallel to a line which makes equal P, and the edges it does not meet are
angles with the coordinate axes meets the 2 6 3
, and .
plane x  y  3 z  2  0 at B. The value of 5 13 10
AB 2 is Statement 1: Volume of ' P ' is 36 cubic
46. Let â and ĉ be unit vectors inclined at an units.
 Statement 2: If x, y , z are the lengths of the
angle of with each other. If
3 coterminous edges of ' P ', then the shortest
distances between a diagonal and an edge not
 aˆ   b  cˆ  .  aˆ  cˆ   5 , then intersecting it are
Statement – 1:  aˆ b cˆ   10 because xy yz zx
, and .
Statement – 2:  x y z   0 if x  y or y  z or x2  y2 y2  z2 z 2  x2
zx. (a) Statement-1 is True, Statement-2 is True;
(a) Statement – 1 is True, Statement – 2 is Statement-2 is a correct Explanation for
True; Statement – 2 is a correct Statement-1
explanation for Statement – 1 (b) Statement-1 is True, Statement-2 is True;
(b) Statement – 1 is True, Statement – 2 is Statement-2 is NOT a correct explanation
True; Statement – 2 is NOT a correct for Statement-1
explanation for Statement – 1 (c) Statement-1 is True, Statement-2 is False
(d) Statement-1 is False, Statement-2 is True
VECTORS & 3D COORDINATE GEOMETRY 35

49. Let ABC be a triangle with A(2, 3, 5), B(–1, 3 2) and C(h, 5, k). If the median through A is
perpendicular to the plane x  2 y  5 z  7 , then _____.
Column – I Column – II
(A) h 2  k 2 is equal to (p) 5
(B) The area of  ABC in square units is equal to 216
(q)
2
(C) Distance between C and plane x  2 y  5 z  7 is equal to 2
(r)
15
(D) The perpendicular distance of the plane ABC from origin is  9 
equal to (s)  
 6
(E) Distance of plane 2 x  y  z  11  0 from C is equal to

(a) A–p; B–q; C–r; D–s; E–s


(b) A–q; B–p; C–r; D–s; E–s
(c) A–p; B–q; C–s; D–p; E–s
(d) A–r; B–p; C–q; D–s; E–q
50. Match the following :
Column-I Column-II

(A) The vector equation of the plane perpendicular (p) r . iˆ  3 ˆj  2kˆ  3
x 1 y  2 z  1

to the line   and passing through
1 1 2
the point (3, 6, 2) is
 
(B) The vector equation of the plane through the (q) r . iˆ  ˆj  kˆ  2
point (5, –2, 4) and parallel to the plane
4 x  12 y  8 z  7
 
(C) The vector equation of the plane containing the (r) r . iˆ  ˆj  kˆ  0
 
line r  2iˆ   ˆj  kˆ and perpendicular to the

 
plane r . iˆ  kˆ  3 is

 
(D) The vector equation of the plane containing the (s) r . iˆ  ˆj  2kˆ  1
x 1 y 1 z x 1 y 1 z
lines   ;   is
1 2 1 1 1 2
 
(t) r . iˆ  ˆj  kˆ  1

(a) A → (p); B → (s); C → (q); D → (r)


(b) A → (s); B → (p); C → (q); D → (r)
(c) A → (s); B → (q); C → (p); D → (r)
(d) A → (s); B → (p); C → (r); D → (q)
VECTORS & 3D COORDINATE GEOMETRY 36

Part-B (b) coplanar lines


(c) co-incident lines
51. A line segment joining (1, 0, 1) and the origin
(d) concurrent lines
(0, 0, 0) is revolved about the X – axis to form
55. P, Q, R, S are four coplanar points on the
a right circular cone. If (x, y, z) is any point on
sides AB, BC, CD, DA of a quadrilateral
the cone, other than the origin, then it satisfies
AP BQ CR DS
the equation _____. respectively. The product   
PB QC RD SA
(a) x 2  2 y 2  z 2  0
equals _____.
(b) x 2  y 2  z 2  0 (a) –2 (b) –1
(c) 2 x 2  y 2  2 z 2  0 (c) 2 (d) 1
(d) x 2  2 y 2  2 z 2  0 x  6 y  10 z  14
56. The line   is the
52. L1 and L 2 are two lines whose vector 5 3 8
hypotenuse of an isosceles right angled
equations are given below.
triangle whose opposite vertex is (7, 2, 4).

L1 : r   cos   3 iˆ      
2 sin  ˆj  cos   3 kˆ Then which of the following is not the side of
the triangle?

L 2 : r   aiˆ  bjˆ  ckˆ  x7 y2 z 4
Here,  and  are scalars. If the angle  is (a)  
2 3 6
the acute angle between the two lines and is x7 y 2 z 4
independent of  , then a possible value of  (b)  
3 6 2
is _____. x7 y2 z4
  (c)  
(a) (b) 3 5 1
6 4 (d) None of these
  57. The lengths of two opposite edges of a
(c) (d)
3 2 tetrahedron are 3 and 4 units, the shortest
53. A rigid body rotates about an axis through the distance between them is equal to 6 unit and
origin with angular velocity 10 3 radians per angle between them is 30 . Then the volume
of tetrahedron in cubic units is _____.
second. If  points in the direction iˆ  ˆj  k ,
(a) 4
then the equation of the locus of the points (b) 6
having tangential speed 20 metres per second (c) 8
is _____. (d) none of these
(a) x 2  y 2  z 2  xy  yz  1  0 58. Let P denote the plane consisting of all points
(b) x 2  y 2  z 2  2 xy  2 yz  2 zx  1  0 that are equidistant from A   4, 2, 1 and
(c) x 2  y 2  z 2  xy  yz  zx  2  0 B  2,  4, 3 . Let Q denote the plane
2 2 2
(d) x  y  z  2 xy  2 yz  2 zx  2  0 x  y  cz  1 , where c  . The plane P is
54. Let P1  r . r1  d , P2  r . r2  d 2 , parallel to plane Q _____.
P3  r . r3  d 3 be three planes where r1 , r2 , r3 (a) for no value of c
are three non-coplanar vectors. Then the lines (b) if c = 3
P1  0  P2 , P2  0  P3 and P3  0  P1 are 1
(c) if c 
_____. 3
(d) if c = 1
(a) parallel lines
VECTORS & 3D COORDINATE GEOMETRY 37

59. A line L1 with direction ratios –3, 2, 4 passes (a) 1 (b) 5


through the point A(7, 6, 2) and a line L 2 with (c) 3 (d) 6

direction ratios 2, 1, 3 passes through the point 63. a  b  c  a  b  1 , a .c  0 and


B(5, 3, 4). A line L3 with direction ratios 2, – iˆ ˆj
a  , where a , kˆ and b are linearly
2, –1 intersects L1 and L3 at C and D. 2 2
The length CD is equal to _____. dependent vectors.
(a) 4 (b) 6 If for some c, b c  a c and
(c) 9 (d) 11
c  piˆ  qjˆ  rkˆ, then 16  p 4  q 4  r 4  is
60. A line L1 with direction ratios –3, 2, 4 passes
_____.
through the point A(7, 6, 2) and a line L 2 with
(a) 8 (b) 21
direction ratios 2, 1, 3 passes through the point 21
B(5, 3, 4). A line L3 with direction ratios 2, – (c) (d) 4
3
2, –1 intersects L1 and L3 at C and D. 64. a  b  c  a  b  1 , a .c  0 and
The equation of the plane parallel to the line
iˆ ˆj
L1 and containing the line L 2 is equal to a  , where a , kˆ and b are linearly
2 2
_____.
dependent vectors.
(a) x  3 y  4 z  30
If b  siˆ  tjˆ  ukˆ , then 8  s 4  t 4  u 4  is
(b) x  2 y  z  15
_____.
(c) 2 x  y  z  11
(a) 4 (b) 5
(d) 2 x  17 y  7 z  33
25 19
61. A line L1 with direction ratios –3, 2, 4 passes (c) (d)
21 4
through the point A(7, 6, 2) and a line L 2 with 65. The length of the perpendicular from the
direction ratios 2, 1, 3 passes through the point origin to the plane passing through three non–
  
B(5, 3, 4). A line L3 with direction ratios 2, – collinear points a, b, c is
  
2, –1 intersects L1 and L3 at C and D. a b c 
 
The volume of the parallelepiped formed by (a)      
ab  ca  bc
AB, AC and AD is equal to _____.   
(a) 140 (b) 138 2  a b c 
(b)      
(c) 134 (d) 130 ab  bc  c a
62. a  b  c  a  b  1 , a .c  0 and   
(c)  a b c 
iˆ ˆj
a  , where a , kˆ and b are linearly   
2 2 a b c 
 
dependent vectors. (d)      
2 ab  c a  bc
If the maximum and minimum values of b . c
are M and m respectively, then 4  M 2  m 2  is
equal to _____.
VECTORS & 3D COORDINATE GEOMETRY 38

66. Let the equations of two straight lines L1 , L2 69. Let DABC be a tetrahedron such that AD is
y  3 z  15 perpendicular to the base ABC and
be respectively be x  5   and ABC  30 . The volume of the tetrahedron
5 2
x y 1 z  6 is 18 cubic units. If the value of
  . A,B are two distinct points AB  BC  AD is minimum, then the length
2 5 3
on the x – axis such that two straight lines of AC is _____.
l1 , l2 both perpendicular to the x – axis ( l1 (a) 6 2  3 (b) 3  6 2 
through A, l2 through B) are drawn so as to
(c) 6 2  3 6(d) 3  2
intersect both L1 , L2 .      
Direction ratios of one of the lines l1 , l2 are     
70. If a  b  c  d . a  d  0, then which of
the following may be true?
(a)  0,3,1 (b)  0, 4,  3    
(a) a , b , c and d are necessarily coplanar
(c)  0,5,  2  (d)  0, 2,3   
(b) a lies in the plane of c and d
67. Let A,B,C represent the vertices of a triangle,   
where A is the origin and B and C have (c) b lies in the plane of c and d
  
position vectors b & c respectively. Points M, (d) b lies in the plane of a and d
N and P are taken on sides AB, BC and CA 71. A vector r has components r1 , r2 , r3 in a right
respectively such that handed rectangular Cartesian coordinate
AM BN CP system o x y z. The coordinate system is
    ,   R  0 now
AB BC CA rotated about y–axis through an angle of 60
If  represents the area enclosed by the three in anti–clock wise direction [As seen from
  
vectors AN , BP, CM then   ___ above XZ plane]. It has components 4, 3,  2,
(a) 0 in the new system O X Y Z then
b c 1  3
(b)
2
  2    1 (a) Direction cosines of OX are
2
, 0,
2
b c (b) r1  2  3
(c)
2
 2    1
(c) r3   1  2 3
(d) b  c  2    1 3ik
(d) Unit vector along OZ is
68. Let A,B,C represent the vertices of a triangle, 2
where A is the origin and B and C have [ iˆ, ˆj & kˆ denotes unit vector in oxyz ]
position vectors b & c respectively. Points M, 72. A Let
N and P are taken on sides AB, BC and CA r   a  b  sin x   b  c  cos y   c  a  ,
respectively such that where a , b and c are non–zero non–
AM BN CP
    ,   R  0 now coplanar vectors. If r is orthogonal to
AB BC CA
3a  5b  2c , then the value of
(a) 0 (b) 1
1 1 sec 2 y  cosec 2 x  sec y cosec x is _______.
(c) (d)
2 4
VECTORS & 3D COORDINATE GEOMETRY 40

81. Match the following.


Column – I Column – II
(A) If D, E and F are the mid points of the sides BC, CA and AB (p) 0
respectively of a triangle ABC and  is a scalar such that
2 1
AD  BE  CF   AC , then  is equal to
3 3
(B) If A , B and C are vectors such that B  C , then the value of 1
(q)
3
 A  B   A  C   B  C . B  C is
(C) In a  ABC, points D, E and F are taken on the sides BC, CA and 1
(r)
BD CE AF 2
AB respectively such that    2 . If
DC EA FB
area   DEF    area   ABC  , then  is equal to
(D) The corner P of the square OPQR is folded up so that the plane 3
(s)
OPQ is perpendicular to the plane OQR. If  is angle between OP and 5
QR, then cos  is equal to
3
(t)
4
(a) A–p; B–r; C–q; D–s (b) A–r; B–p; C–s; D–r
(c) A–s; B–q; C–p; D–r (d) A–r; B–p; C–q; D–r
82. Match the following.
Column – I Column – II
(A) The equation of the plane through the points (p) 4 x  y  2 z  6  0
1, 0,  1 ,  3, 2, 2 and parallel to the line
x 1 1  y z  2
  is
1 2 3
(B) A variable plane is at a constant distance 7 (q) 3 x 2  3 y 2  3 z 2  4 xy  4 yz  48
from the origin and meets the coordinate axes in
A, B, C. The locus of the centroid of tetrahedron
OABC is
(C) The locus of a point which moves such that 16
sum of squares of its distances from the plane (r) x 2  y  2  z  2 
49
x  y  z  0 , x  2 y  z  0 and x  z  0 is 16 is
(D) The locus of a point which moves in such a (s) x 2  y 2  z 2  5 xy  3 yz  3 zx  0
x y z
way that its distance from the line   is
1 1 1
twice its distance from x  y  z  0 is
(t) 3 y  2 z  2  0
(a) A–r; B–p; C–q; D–s
(b) A–p; B–r; C–s; D–q
(c) A–p; B–r; C–q; D–s
(d) A–s; B–r; C–q; D–p
VECTORS & 3D COORDINATE GEOMETRY 41

83. Match the following:


Column-I Column-II
x y z 3y  5 (p) 2
(A) A line drawn parallel to   intersect the lines x  z
2 1 2 3
3x  5
and  2 y  2 z at ‘P’ and ‘Q’ respectively then PQ 
3
(B) Let ‘M’ be the number of distinct solutions of the equation (q) 7
M 1
sin 2 x  sin 3x in  7 , 7  then 
10
(C) If a  1, b  2, c  3, a , b are perpendicular to c , a .b  1 then (r) 5

a b a b c 
 b c  
 5 3 
1
2 100
(s) 3
 1  x 
201 0
dx
(D)  
100 1 2 99
 1  x 
0
dx

(a) A → (p); B → (q); C → (s); D → (r)


(b) A → (r); B → (s); C → (q); D → (p)
(c) A → (q); B → (q); C → (s); D → (p)
(d) A → (r); B → (q); C → (s); D → (p)
LIMIT CONTINUITY AND DIFFERENTIABILITY(LCD) 42

Limit Continuity and Differentiability(LCD)

Part-A
1 5. Let f  x   1  x 2  sgn x and g  x   f 1  x  .
1. If f  x  , then the points of
2 x The correct statements of the following is
discontinuity of the function _____.
 
y  f f  f  x   are _____. (a) g is continuous at x = –1, 6, 1
(b) g is discontinuous at x = 0 only
3
(a) 2, (b) 1, 2 (c) g is discontinuous at x = –1, 1 only
4 (d) g(x) is continuous everywhere in its
3 4 domain
(c) 2, 3 (d) 2, ,
2 3  
2. If  x  is the greatest integer less than or equal 
n

 1 
6. The value of lim  is
 x2  n
r 1
   n  r   n 
to x, then lim    _____.  sin   
x  0 sin x tan x
    4n  
(a) –1 (b) 2 equal to _____.
(c) 0 (d) 1
et f  x   e x f  t  1
(a) 2 ln  2 1  (b) 4 ln  2 1 
3. If lim 2 and f  0  ,
tx
t  x   f  x 
2
2 (c) 4 ln  2  1 (d) ln 2

then find the value of f '  0  . 7. If f  x   sgn  sin 2 x  sin x  1 has exactly
(a) 4 (b) 2 four points of discontinuity for x   0, n 
(c) 0 (d) 1 n  then n can be
4. If g  x is a polynomial satisfying (a) only 4 (b) 4 or 5
g  x  g  y   g  x   g  y   g  xy   2 for all (c) only 5 (d) 5 or 6
 n r
real x and y and g  2   5 , then lim g  x  is 8. If f  x     ar x  1  , where ar are
x 3
 r0 
_____.
(a) 9 (b) 25 positive real constants, then f  x  is _____.
(c) 10 (d) none of these (a) not continuous at x  1
(b) continuous everywhere but not
differentiable at x  0
(c) continuous everywhere but not
differentiable at x  1
(d) differentiable everywhere
LIMIT CONTINUITY AND DIFFERENTIABILITY(LCD) 43

9. Let f be a real valued function defined on 12. Consider the function


some neighbourhood c ,where c  . x
 4  t  2 dt , x  3
Statement 1: If f is differentiable at x  c , f  x   0 . If f  x is
f c  h  f c  h  ax2  bx x3
then lim exists and 
h0 2h differentiable at x = 3, then _____.
equals f   c  . (a) a  b  83 (b) a  b  85
18 18
f c  h  f c  h
Statement 2: If lim 7 b
h0 h (c) ab  (d)  84
27 a
exists, then f is differentiable at x  c .
13. f  x is a differentiable function such that
(a) Statement-1 is True, Statement-2 is True;
Statement-2 is a correct Explanation for 1
f    0  x then which of the
Statement-1  x
(b) Statement-1 is True, Statement-2 is True; following is/are true _____.
Statement-2 is NOT a correct explanation (a) f  0  0 (b) f   0  0
for Statement-1
(c) Statement-1 is True, Statement-2 is False (c) f 1  0 (d) f  1  0
(d) Statement-1 is False, Statement-2 is True 14. If f  x   x 2  4 x  3 then _____.
10. Consider a differentiable function f : 
(a) f  x is non-differentiable at 5 points
for which f  0  ln2 and
(b) f  x is non-differentiable at 4 points
f  x  y   2x f  y   4y f  x   x, y  .
Which of the following is (are) correct? (c) f  x has local maxima at x = 0

(a) f  4  240 (d) f  x has local minima at x = –1

(b) f   2  24ln2 sin  1  sin x  x


15. If A  lim and B  lim   ,
1 x0 cos  1  cos x  x 0 x
(c) the minimum value of y  f  x  is 
4 where   represents GIF, then _____.
(d) the number of solutions of f(x) = 2 is 1
(a) A = 1 (b) A does not exist
11. Suppose that f(x) is a differentiable invertible
x (c) B = 0 (d) B = 1
function with f   x   0 and h  x    f  t  dt  sin 3  x 
1  a  x0
 x
. Given that f 1  f  1  1 and g(x) is 16. If f  x   3, x0 and
inverse of f(x). Let  sin x  x 
2b   x0
G  x  x g  x  xh g  x  x 
2
. Which of 

 x
3 

the following are correct? f  x  is continuous at x = 0, then _____.
(a) G1  2 (b) G1  3 [.] denotes greatest integer function.
(c) G 1  2 (d) G 1  3 (a) a = 2 (b) a = 3
(c) b = 2 (d) b = 3
LIMIT CONTINUITY AND DIFFERENTIABILITY(LCD) 44

 1
17. lim  mx sin  is equal to (where [.] denotes
x 
 x 22. Let K  0 and   lim

K 1  4 K 2  x2 
x0
x2 K 2  x2
the greatest integer function and m  I )
(a) m if m < 0 (b) m – 1 if m > 0 is finite then the value of  K is _____.
(c) m – 1 if m < 0 (d) m if m > 0 23. Let f  x  , f:  be a non-constant
18. Let f:  be defined by continuous function such that
 2 1 f  2 x    e  1 f  x  . The value of f   0  is
x
 x  2 x sin for x0
f  x   x then
 _____.
0 for x0
f h f h
(a) f  x  is differentiable for all x but f   x  (a) lim (b) lim
h  0 eh  1 h 0 eh  1

is not continuous at x  0 f h f h


(c) lim h (d) lim  h
(b) f   0   1 h0 e  h 1 h0 e 1
(c) f  x  is increasing at x  0 24. Let f  x , f:  be a non-constant
continuous function such that
(d) Both f  x  and f   x  are differentiable
f  2 x    e x  1 f  x  . If f   0   1 , then
for all x
1
19. Suppose f is a function that satisfies the  f  x x
equation lim   equal to _____.
x0
 x 
f  x  y   f  x   f  y   x 2 y  x y 2 for all
1
f  x (a) f  1 (b) f   
real numbers x and y . If lim 1, 2
x0 x
(c) f   0  (d) f   1
Then _____.
(a) f  x   0 for x  0 and f  x   0 for 1  1 
25. Consider   1 and f :  ,     ,   be
x0
   
a bijective function. Suppose that
(b) f   0   1
1 1 
(c) f   0   1 f 1  x   , x   ,   . Then f 1 
f  x  
(d) f   0   6 _____.
sin x  ae x  be x  c ln 1  x  (a) 1
20. If L  lim (b) 0
x 0 x3
(c) –1
exists, then find the value of a  b  c .
(d) doesn’t attain a unique value
21. The number of points where
 1
f  x    x    x  is discontinuous (where
 2
[.] represents greatest integer function) for
2  x  2 is _____
LIMIT CONTINUITY AND DIFFERENTIABILITY(LCD) 45

1  1  30. Match the following:


26. Consider   1 and f :  ,     ,   be
   
a bijective function. Suppose that Column – I Column – II
1 (A) The equation (p)  ,  11,  
1 
f 1  x   , x   ,   . Which of the x  cos x  a has exactly a
f  x  
positive root, complete set
following statements can be concluded about of value of a is
f  x ? (B) If (q) 1, 2
2
1  f  x   cos x  a x  b is
(a) f  x  is discontinuous in  ,  
  an increasing function for
1  all values of x , then the
(b) f  x  is increasing in  ,  
  complete set of values of a
1  is
(c) f  x  is decreasing in  ,   (C) If the function
  (r)  20,  16 
3 2
(d) none of these f  x   x  9 x  24 x  a
27. Let f  x   cos 1  sin x   sin 1  cos x  and has 3 real and distinct roots
 ,  and  then the
3
 x  2  . The function f  x   _____. possible values of a are
2
(a) 4   2 x (b) 2 x  4  (s) 1,  
(c) 2 x (d) x  
(a) A → (p); B → (q); C → (r)
28. Let f  x   cos 1  sin x   sin 1  cos x  and
(b) A → (s); B → (r); C → (p)
3 sin x  tan x (c) A → (p); B → (s); C → (r)
 x  2 lim 3
 _____.
2 x  2 (d) A → (s); B → (p); C → (r)
 f  x  
1 1 Part-B
(a) (b)
16 32 31. If f  x  be a non-constant thrice differentiable
1 1 function defined on such that
(c) (d)
8 16 f  x  f 6  x and
29. Let f  x   cos 1  sin x   sin 1  cos x  and
f   0   f   2   f   5  0 , then minimum
3
 x  2  . The area bounded by and x– number of zeroes of
2 2
axis is _____. g  x    f   x    f   x  f   x  in the
 2 interval (0, 6) is equal to _____.
(a) (b) (a) 10 (b) 11
2 2
(c) 12 (d) 13
2 2
(c) (d)
4 24
LIMIT CONTINUITY AND DIFFERENTIABILITY(LCD) 46

32. The value of  2 


1 1  x cos when x0
37. f  x    x
lim
1  3x  2 x   1  3x  2 x 
2 x 2 x

is _____. 
 0 when x0
x0 x
(a) e3 (b) 3e 2 (a) f  x  is not differentiable at x  0
1 (b) f  x  is differentiable at x  0
(c) e2 (d) 4e3
3 (c) f  x  is not differentiable at
tan x
33. If f  x   then 2
x x :n
1 2n 1
x 0 
 
lim  f  x    x 2  f  x   (where [.] denotes
(d) f  x  is differentiable at x 
2
:n
the greatest integer function and {.} denotes 2n 1
fractional part). xm f  x   h  x   3
38. g  x   lim when x 1
(a) 3 (b) log 3 m 2 xm  4 x  1
(c) e3 (d) Does not exist and g 1  e3 such that f  x  , g  x  and
34. A A hexagon is inscribed in a circle of radius
h  x  are continuous function at x = 1 and
r . Two of its sides have length 1, two have
length 2 and the last two have length 3, then r f 1  h 1  a  b  g 1  , a, b  Q then a + b
is a root of the equation. is _____.
(a) 2 r 3  7 r  3  0 (b) 2 r 3  7 r  3  0 39. A function f:  satisfies
3 3
(c) 2 r  7 r  3  0 (d) 2 r  7 r  3  0 sin x cos y  f  2 x  2 y   f  2 x  2 y  
t . If
f  x   lim
 a  sin  x   1 , x  0,6  cos x sin y  f  2 x  2 y   f  2 x  2 y  
35. t   then
t 
 a  sin  x   1 1
f  0 , then the value of 4 f   x   f  x 
_____. 2
(a) If a = 1, then f(x) has 5 points of is ______
discontinuity 1
(b) If a = 3, then f(x) has no point of 40. Let I n   x n 1  x 2 dx where n is an integer
0
discontinuity
(c) If a = 0.5, then f(x) has 6 points of In
 2 . Then lim  _____.
nI
discontinuity n2

(d) If a = 0, then f(x) has 3 points of sgn  x  2    log e x  , 1  x  3


discontinuity 41. If f  x   2
 x  , 3  x  3.5
36. If f  x  1  x  2 , 0  x  4 and
where [.] denotes the greatest integer function
g  x   2  x ,  1  x  3 then  fog  x  is and {.} represents the fractional part function,
_____. then the number of points of discontinuity is
(a) Discontinuous at x  0 _____.
(b) Continuous at x  0
(c) Not differentiable at x  0
(d) Differentiable at x  0
LIMIT CONTINUITY AND DIFFERENTIABILITY(LCD) 47

n
42. Let f  x  be a differentiable function,  x
46. Let f  x   lim  cos  ,
f 1  0 , f  1  2 then the value of n
 n 
x2 n

lim 1
x 2 sin f  t  dt
is _____. n 

g  x   lim 1  x  x n e  . Now, consider the
x 1
 x  12 function y  h x , where
43. Let A r  r   be the area of the bounded
ln  f  x  
region whose boundary is defined by h  x   tan 1  g 1 f 1  x   lim is
x0 ln  g  x  
 6 3 r y 2  x  6 e2 y  x   0 then
equal to _____.
A1 , A 2 , A 3 ,... are in 1 1
(a) (b) 
(a) A.P. (b) G.P. 2 2
(c) H.P. (d) A.G.P. (c) 0 (d) 1
n
44. Let A r  r   be the area of the bounded  x
47. Let f  x   lim  cos  ,
n   n 
region whose boundary is defined by 
 6 3 2
 2

r y  x 6 e y  x  0 then lim
n
[

g  x   lim 1  x  x n e
n
 . Now, consider the
n 
A 1 A 2 A 3  A 2 A 3 A 4  A 3 A 4 A 5 …. n y  h x ,
function where
terms] is _____.
9 9
h  x   tan 1  g 1 f 1  x   . Domain of the
e 1 e 
(a)   (b)   function y  h  x  is _____.
  2 
1 e 
9
1 e 
9 (a)  0,  (b) R
(c)   (d)  
3   4  (c) (0, 1) (d) [0, 1]
n
45. Let A r  r   be the area of the bounded  x
48. Let f  x   lim  cos  ,
n   n 
region whose boundary is defined by 
 6 3 r y 2  x  6 e2 y  x   0 then lim
n 
g  x   lim 1  x  x n e
n
 . Now, consider the
n 
1 1 1
A1 A2 A3 function y  h x , where
[e e e  ….n terms] is _____.
1 h  x   tan 1  g 1 f 1  x   . Range of the
 3  3 
3
(a) e e3 (b)  e e  1 function y  h  x  is _____.
 
 
    
2 3  3 
2 (a)  0,  (b)   ,0 
3  2  2 
(c) e e3 (d)  e e  1
    
  (c) R (d)   , 
 2 2
LIMIT CONTINUITY AND DIFFERENTIABILITY(LCD) 48

49. Match the following

Column – 1 Column – 2
 1 (p) continuous
(A) f  x   lim cos 2 n  2 x    x   , where {.} denotes the fractional part
n 
 2
1
function, at x  , n  N .
2
1
(q) discontinuous
(B) f  x    log e x  x  1 5 at x =1

4 x (r) differentiable
(C) f  x    cos 2 x 
sin   , where [.] and {.} denote the greatest
 2
integer and the fractional part function, respectively, at x = 1.
cos 2 x, x    (s) non differentiable
(D) f  x    at x 
 sin x, x   6
(E) f  x   min 1, cos x, 1  sin x ,   x   at x = 0.

(a) A–p; B–r; C–q; D–s; E-p


(b) A–r; B–p; C–s; D–r; E-p, s
(c) A–q, s; B–p, r; C–p, r; D–p, s; E–p, s
(d) A–r; B–p; C–q; D–r; E-s
FUNCTIONS, INVERSE TRIGONOMETRIC FUNCTIONS (ITF) 49

Functions, Inverse Trigonometric Functions (ITF)

Part-A
6. Total number of values of x, of the form
1. If cos 1 x    cot 1 x   0 , where [.] denotes
1 1 1
, n  N in the interval x   ,  , which
the greatest integer function, then the n  25 10 
complete set of values of x is satisfy the equation
(a) (cos 1, 1] (b) (cos 1, cot 1)
 x  2 x  ....  12 x  78 x , (where  
(c) (cot 1, 1] (d) [0, cot 1)
2. If a polynomial represents fractional part function)
(a) 12 (b) 13
f  x   4 x 4  ax 3  bx 2  cx  5  a, b, c  R 
(c) 14 (d) 15
has four positive real roots r1 , r2 , r3 , r4 such 7. A continuous function y  f  x  is defined in
r r r r
that 1  2  3  4  1 . Then value of a will a closed interval  7, 5 . A  7,  4  ,
2 4 5 8
be B  2, 6  , C  0, 0  , D 1, 6  , E  5,  6  are
(a) 20 (b) 21 consecutive points on the graph of ‘ f ’ and
(c) 19 (d) 22 AB, BC, CD, DE, DE are line segments. The
3. Consider the function f  x  : A  A which minimum number of real roots of the
satisfy the condition f  f  x    x , then equation f  f  x   6 is _____.
number of such functions for A = {1, 2, 3, 4, (a) 6 (b) 4
5} are
(c) 2 (d) 0
(a) 54 (b) 52
(c) 21 (d) 26 8. The greatest value of
2 3
 3   2sin   3cos  4   6  2sin   3cos 
4. f : 0,       1,1 and
 2  as  ranges over all real numbers is _____.
 x    . If f (x) is onto, then (a) 3456 (b) 2345
f  x    sin x  (c) 4567 (d) 1234
 x   2
Set of equations x  bn x  111....1
9.   n , n = 2,
set of exhaustive values of  is n  times
(a)  0,   (b)  0,     / 2 3, 4, ….., 9 and bn  N , have integer roots.
  3  Which of the following are CORRECT?
(c)  0,   (d)  ,  (a) Common root of these equations may be 9.
2 2 
5. A function f is defined by (b) No three of these equations have a
m n common root.
f  x   x x  1 x  R . Then local maxima (c) A root of an equation with n = 4 may be
value of the function is  m, n  N  123.
(a) 1 (b) nm (d) A root of an equation with n = 4 is 1234.
mn

(c)
mm nn
(d)
 mn 
m n m n
m  n m  n
FUNCTIONS, INVERSE TRIGONOMETRIC FUNCTIONS (ITF) 50

10. Statement 1: Let f :  , 13. If complete solution set of inequality


f  x   x  sin x if f 1  x  is inverse function m 
x 2  2 x  4  x, is  ,   , m, n being co–
 n 
2
of f  x  then 0 f 1  x  dx  2 prime numbers, then value of m + n is
2
14. Number of ordered triplets (x, y, z) for which
Statement 2: The functions f  x and x3  y 3  z 3  3 xyz  4, where x, y, z  N , are
f 1  x  are mirror images w.r.t the line y  x 15. N is the set of all natural number and R is the
(a) Statement-1 is True, Statement-2 is True; set of all real numbers. A function f : N  R
Statement-2 is a correct Explanation for is given by:
Statement 1 4n  4n 2  1
f n  , then the value of
(b) Statement-1 is True, Statement-2 is True; 2n  1  2n  1
Statement-2 is NOT a correct explanation  40 
for Statement-1  f r 
(c) Statement-1 is True, Statement-2 is False  r 1  is equal to _________ (where 
 40 
(d) Statement-1 is False, Statement-2 is True  
1
11. For the function f  x   x sin , x  1 . represents greatest integer function)
x
16. If f is a function such that f(0) = 2 and f(1) =
(a) lim f   x   0
x 3 and f(x + 2) = 2f(x) – f(x + 1) for every real
(b) For atleast one x in the interval 1,   , x, and f  5   a 2  b 2 , a  b then the value of

f  x  1  f  x  b 
 a  , where  denotes the greatest integer
(c) For all x in the interval 1,   , function and a, b  N ) is ______.
f  x  1  f  x  17. The number of solutions of equation
(d) f   x  is strictly decreasing in the interval 8  x 2  x   4  x   13  12 sin x  is (here [.]
1,   represents greatest integer less than or equal
to ‘ x ’)
12. Let f  x   a1 cos 1  x   a2 cos  2  x  
18. p  x   x 4  ax3  bx2  cx  d , where
.....  an cos  n  x  . If f  x  vanishes
p 1  10 , p  2   20 , p  3  30 then value
for x  0 and x  x1 (where x1  k  , k  ),
p (12)  p( 8)
then _____. of  1980 is _____.
10
(a) a1 cos1  a2 cos 2  ...  an cos n  0
19. Let x , y , z , t be real numbers such that
(b) a1 sin 1  a2 sin  2  ...  an sin  n  0
x2  y 2  9 , z 2  t 2  4 , and xt  yz  6 .
(c) f  x   0 has only two solutions 0, x1 Then greatest value of P  xz is _____.
(d) f  x  is identically zero  x
FUNCTIONS, INVERSE TRIGONOMETRIC FUNCTIONS (ITF) 51

 2 x  a x  1 24. For x  0, 1 define


20. Let f  x    2 and
bx  3, x  1 1
f1  x   x, f 2  x   , f3  x   1  x,
 x  4, 0 x8 x
g  x   1 x 1
3x  2, 2  x  0 f4  x   , f5  x  
1 x x
g  f  x   is not defined if
Let H be a function such that f3o H o f 2  f 4
(a) a  10,   , b   5,      , 5
then H is equal to
(b) a   4,10  , b   5,   (a) f5 (b) f 4
(c) a  10,   , b   0,1 (c) f3 (d) f 2
(d) a   4,10  , b  1,5  25. Let f :  be a continuous function such
 2 x  a x  1  x x
21. Let f  x    2 and that f  x   2 f    f    x 2 , then f  3
bx  3, x  1 2 4
is equal to _____.
 x  4, 0 x8
g  x   (a) f  0  (b) 4  f  0 
3x  2, 2  x  0
If the domain of g  f  x   is  1, 4  , then (c) 9  f  0  (d) 16  f  0 

(a) a  1, b  5 26. f :  be a continuous function such that


(b) a  2, b  7 x x
f  x   2 f    f    x2
(c) a  2, b  10 2 4
The equation f  x  – x – f  0  0 has
(d) a  0, b  R \  5,5
_____.
22. For x  0, 1 define
(a) no solution
1 (b) one solution
f1  x   x, f 2  x   , f3  x   1  x,
x (c) two solution
1 x 1 (d) infinitely many solution
f4  x   , f5  x  
1 x x 27. Let f :  be a continuous function such
Let F be a function such that f1o F  f 4 then F
x x
is equal to that f  x   2 f    f    x 2
2 4
(a) f1 (b) f 2
f   0  is equal to _____.
(c) f 3 (d) f 4
23. For x  0, 1 define (a) 0 (b) 1
1 (c) f  0  (d)  f  0 
f1  x   x, f 2  x   , f3  x   1  x,
x
1 x 1
f4  x   , f5  x  
1 x x
Let g be a function such that f 4 oG  f 5 then G
is equal to
(a) f1 (b) f 2
(c) f 3 (d) f 4
FUNCTIONS, INVERSE TRIGONOMETRIC FUNCTIONS (ITF) 52

28. Match the orbital given in Column - I with the z  z 2  1  10 x  z are


properties given in Column - II (a) 4 (b) 3
Column-I Column-II (c) 2 (d) 1
(A) f(x) = x sin x (p) even
31. If the equation 2log  x  3  log  ax  has
(B) f(x) = x + tan–1 x (q) bounded
only one root then _____.
(C) f(x) = {cos x} (r) periodic
(where {.} represents (a)    0,  
fractional part (b)    , 0   12
function)
(c)   12,    1
(D) f(x) = {tan x} (s) invertible in R 
(where {.} represents R (d)    0, 12   24
fractional part
32. If f  x   cos  sin x   sin  cos x  , then
functions)
(t) odd the range of f  x  .
(a) A–p; B–t, s; C–p, q, r; D–q, r
(a)  cos1, sin1 
(b) A–r; B–p; C–s; D–r
(c) A–s; B–q; C–p; D–r (b)  cos1, 1  sin1 
(d) A–r; B–p; C–q; D–r
(c)  1  cos1, sin1 
Part-B
29. Statement – 1: In a quadrilateral ABCD with (d)  cos1,1
a2  b2  c2 33. If f : R  N  0 , where f (area of triangle
sides a, b, c, d the range of is
d2 joining points P(5, 0) , Q(8, 4) and R(x, y)
1  such that the angle PRQ is a right angled) =
 , .
3  number of triangle. Then, which of the
Statement – 2: For sides of quadrilateral a, b, following is TRUE?
2
c and d always  a  b  c   d 2 . (a) f(5) = 4
(b) f(7) = 0
(a) Statement – 1 is True, Statement – 2 is
(c) f(6.25) = 2
True; Statement – 2 is a correct explanation
(d) f(x) is into function
for Statement – 1
(b) Statement – 1 is True, Statement – 2 is 34. Let f :  be a function defined by
True; Statement – 2 is NOT a correct f  x  5
f  x  1  x . Then which of
explanation for Statement – 1 f  x  3
(c) Statement – 1 is True, Statement–2 is False the following statement(s) is/are true
(d) Statement – 1 is False, Statement – 2 is _____.
True (a) f  x  2   f  x 
30. The number of solutions of the system of
equations (b) f  x  4   f  x 

x  x 2  1  10 y  x (c) f  x  6   f  x 
y  y 2  1  10 z  y (d) f  x  8   f  x 
FUNCTIONS, INVERSE TRIGONOMETRIC FUNCTIONS (ITF) 53

35. A function f(x) satisfies the relation 7


(a) (b) 9 7
f  x  y   f  x   f  y   xy  x  y  x, y  R 3
. If f '  0   1 then f '  3  equals to _______. 7
(c) 3 7 (d)
36. If ‘ x ’ is positive and x   x  ,  x  and ‘ x ’ are 9
42. Consider the system of equations in real
in G.P., (where [.] denotes greatest integer
numbers x, y, z satisfying
2
function), then  2 x  2  x   1 is equal to x

y

z
and
_____. 4 x  1 5 y  1 6 z2  1
2 2

  x  1 , x  0 x  y  z  xyz .
37. If f  x   2 , then the A value of z satisfying the system is _____.
   x  1 , x  1
3 7 2 7
number of solutions of the equation (a) (b)
2 3
f  x   f 1  x   0 is/are _____. 7
(c) 3 7 (d)
1 1 3
38. f  x    . Let x1, x2 are points
1 x 1 x 1 43. Let am  m  1,2,3,.... p  be the possible
where f  x  attains local minimum and global integral values of a for which the graphs of
f  x   ax2  2bx  b and
maximum respectively. Let
k  f  x1   f  x2  then 6 k  9 . g  x   5 x 2  3bx  a meets at some point for
p
39. The greatest value of all real values of b . Let Tr    r  am  and
m 1
2 3


f  x  3  4  x 2
  1  4 x 2
 is A, n
Sn   Tr  n   minimum possible non-
r 1
A
then  _____. integral value of a is _____.
7 5 5
40. Consider the system of equations in real (a) (b) 
26 26
numbers x, y, z satisfying 3 3
(c) (d)
x y z 26 26
  and
4 x 12 2
5 y 1 6 z2  1 44. Let am  m  1,2,3,.... p  be the possible
x  y  z  xyz . integral values of a for which the graphs of
The number of ordered triplet, satisfying the f  x   ax2  2bx  b and
system is
g  x   5 x 2  3bx  a meets at some point for
(a) exactly two (b) exactly three
p
(c) exactly five (d) exactly six all real values of b . Let Tr    r  am  and
m 1
41. Consider the system of equations in real
n
numbers x, y, z satisfying Sn   Tr  n  
r 1
x y z
  and Sum of all the possible values of n for which
4 x2  1 5 y2  1 6 z2  1
Tn vanishes, is ______
x  y  z  xyz .
(a) 10 (b) 15
A value of x satisfying the system is _____.
(c) 21 (d) 20
FUNCTIONS, INVERSE TRIGONOMETRIC FUNCTIONS (ITF) 54

45. Let am  m  1, 2,3,.... p  be the possible integral


values of a for which the graphs of
2
f  x   ax  2bx  b and
g  x   5 x 2  3bx  a meets at some point for
p
all real values of b . Let Tr    r  am  and
m 1
n
Sn   Tr  n  
r 1

1
Values of  r  r  1 r  2  r  3  ____.
r 1

1
(a) 1 (b)
2
1
(c) (d) 0
18
46. Match the following
Column-I Column-II
 x   2x  (p) 2
(A) The number of real solutions of       x , where [.] is greatest
2  3 
integer less than or equal to x
(B) Natural number ‘a’ for which x  ax 2  2 for all x   0,   is (q) 6
(C) f :  a, a  4  b, b  32; f  x   x 2 is an invertible function, then (r) 19
maximum value of a is
(D) Let  ,  be real roots of the equation x 2   k  2  x +  k 2  3k  5   0 . (s) 4
Then the maximum value of  2   2 is
(t) 1
(a) A–p; B–p, q, r, s; C–q; D–s
(b) A–q; B–p; C–q; D–r
(c) A–s; B–q; C–p; D–r
(d) A–r; B–p; C–q; D–r
DIFFERENTIATION, APPLICATIONS OF DERIVATIVES (AOD) 55

Differentiation, Applications of Derivatives (AOD)

Part-A 5. Let f 1  2 and f   x  4.2 for 1  x  6 .


1. Water is drained from a vertical cylindrical The smallest possible value of f(6) is
tank by opening a valve at the base of the (a) 9 (b) 12
tank. It is known that the rate at which the (c) 15 (d) 19
water level drops is proportional to the square 6. Let a, b be real numbers. If slope of the
root of water depth y, where the constant of 2
tangent to y  eabx at point P (1, 1) is 2,
proportionality k > 0 depends on the
acceleration due to gravity and the geometry then  a 2  b2  is equals to
of the hole. If t is measured in minutes and (a) 1 (b) 2
1 (c) 4 (d) 8
k  then the time to drain the tank if the
15 7. Let f : R  {0}  R defined by
water is 4m deep to start with is 1
(a) 30 minutes (b) 45 minutes f x  x  , then
x
(c) 60 minutes (d) 90 minutes
(a) f (x) is odd and injective x  R  0
2. Let f(x) be an increasing function defined on
 0,   . If f  2a 2  a  1  f  3a 2  4a  1 , (b) in the interval  0,  f  x  is odd and
then the number of integral values of a is/are strictly increasing
(a) 5 (b) 4 (c) in the interval  ,0 f  x  is odd and
(c) 3 (d) 6 strictly increasing
3. A composite function
(d) in the interval  ,0   0,  f  x  is
 f1o f2o f3o..... o f21  X  is an increasing
odd and strictly increasing
function. If number of increasing functions in
8. If the equation
the set  f1, f2 ,....., f21 is r and remaining are n n1
an x  an1x  ...  a1x  0(a1  0, n  2) has
decreasing functions, then maximum value of
a positive root x  , then the equation
r  21 r  is n1 n2
nan x  (n 1)an1x  ....  a1  0 has a
441
(a) 110 (b) positive root, which is:
4
(a) equal to  (b)  
(c) 105 (d) None of these
(c)   (d)  
 x 3  x 2  10 x  6, x 1 9. The chord of the parabola
4. Let f  x    then
 2 x  log 2  b  17  , x  1
2
y  a 2 x 2  5ax  4 touches the curve
the sum of all integral values of all integral 1
at the point x = 2 and is bisected by
y
values of ‘b’ such that f(x) has the greatest 1 x
value at x = 1, is that point. Then ‘a’ can be.
(a) 23 (b) 9 (a) 0 (b) 1
(c) 0 (d) 35 (c) 2 (d) 3
DIFFERENTIATION, APPLICATIONS OF DERIVATIVES (AOD) 56

10. The set of all values of a for which the 15. A particle is moving along the parabola
 a4  5 y 2  12 x at the uniform rate of 10 cm/sec,
function f  x   1 x  3x  log5 then the component velocity parallel to x-
 1 a 
axis when the particle is at the point  3,6 is
decreases for all real x is
(a)  ,   (a) 2 2 (b) 4 2
(c) 5 2 (d) 3 2
 3  21 
(b) 4,   1,  16. If the graphs of the functions y  ln x & y  ax
 2  intersect at exactly two points, then
 27   1
(c)  3,5     2,  (a) a  0, e (b) a   0, 
 2   e
(c) a  e,1 (d) a 1, e
(d) 1,
11. Let a, b, c, d, e, f, g, h be distinct elements in 17. A function f such that f (a) = f (a) = .... = f 2n
the set {–7, –5, –3, –2, 2, 4, 6, 13}. The (a) = 0 and f has a local maximum value b at
x = a, if f(x) is
minimum value of (a + b + c + d)2 + (e + f +
(a) (x – a)2n+2
g + h)2 is (b) (b – 1) – (x + 1 – a)2n+1
(a) 30 (b) 32 (c) b – (x – a)2n+2
(c) 34 (d) 40 (d) (x – a)2n+2 – b
p2 1 3 x
cos t
12. If f ( x)  x  3x  log 2 is a decreasing 18. Let f  x    dt ; and f '  x   0 for
p2  1 0
t
function for all x  R then the set of possible 
values of p is x  n  , n  Z then f (x) has
2
(a)  1,1 (a) maxima when n   2,  4,  6,........
(b) 1, (b) maxima when n   1,  3,  5,........
(c) minima when n  0, 2, 4,........
(c)  , 1
(d) maxima when n  1, 3, 5,........
(d) None of these
19. The total number of maxima and minima of
13. If ab  2 a  3b , a  0, b  0 then the minimum
 x 4  2 x2 , 3  x  0
value of ab is  23
(a) 12 (b) 24  3x 2 x , 0 x2
f  x   is
1  1 , x2
(c) (d) None of these
4  x 3 , 2 x4

14. A drop of liquid evaporates at a rate
(a) 2 (b) 3
proportional to its area of surface. If the radius
(c) 4 (d) 5
initially is 4 mm and 5 min later, the radius is
reduced to 2 mm, then the radius of the drop
(in mm) as a function of time t (in min) is
(a) 4 + 0.4t (b) 4 – 0.4t
(c) 4 + 0.6t (d) 4 – 0.6t
DIFFERENTIATION, APPLICATIONS OF DERIVATIVES (AOD) 57

20. Statement-1: The range of the function 24. If 2 f  sin x   2 f   cos x   tan x for all
2
f  x   sin x   sin x   where   2 will
  1
2
x  R   2n  1  , n  Z , then f   is
  2 2
be real numbers between  and
4 equal to
    1. 3 2 3 2
2 (a) (b)
Statement-2: The function g  t   t   t  1 6 6
where t  1, 1 and   2 will attain 2 3 3 2
(c) (d)
minimum and maximum values at t = -1 and 6 6
t = 1 respectively 25. Angle between the curves
(a) Statement-1 is True, Statement-2 is True; y   sin x  cos x  where [.] denotes the
Statement-2 is a correct explanation for
greatest integer function and x2  y2  5 is
Statement-1
(b) Statement-1 is True, Statement-2 is True; tan1  k  then k  _____.
Statement-2 is NOT a correct explanation 1
(a) (b) 3
for Statement-1 2
(c) Statement-1 is True, Statement-2 is False 1
(c) (d) 2
(d) Statement-1 is False, Statement-2 is True 3
(e) Both Statement are false. 26. The number of real solutions of the equation
21. Let P(x) be a fourth degree polynomial with 1  cos2 x  2 sin 1  sin x  ,    x   is
derivative P' x . Such that P(1) = P(2) = P(3) _____.
 P '  7  0 . Let k is the real number k  1, 2, 3 (a) 0 (b) 1
(c) 2 (d) Infinite
such that P(k) = 0, then k is equal to
317 319 27. If x2  4y2  4  0 and minimum and
(a) (b)
37 37
maximum values of x2  y2  xy are a and b
321 15
(c) (d) respectively, then
37 37
22. Range of the function f  x   x 1 x is 5  13
(a) a 
3  9 1 2
(where   represents fractional part function) 5  13
(b) b 
(a) [1, 2] 2
(b) [1, 2)
(c) (1, 2] (d) none of these 5  13
(c) b 
23. If g(x) is the inverse function of f(x) and 2
f '  x   cos 2x, then g '  x is equal to 5 13
(d) b 

(a) sec 2 g  x   2
(b) sec g  x
(c) cosec g  x 
(d) none of these
DIFFERENTIATION, APPLICATIONS OF DERIVATIVES (AOD) 59

35. Consider the function (d) The number of critical point(s) of ‘f ’ is 2


  38. Let a curve be given parametrically
f  x   sin 5 x  cos5 x  1, x  0,  . Which
 2 x  t   3t 2 , y  t   2t 3 , t  R
of the following is/are correct? Suppose a line L is tangent at one point and
  normal at another point of the curve. Then
(a) ' f ' is strictly decreasing in  0, 
 4 which of the following statements is/are
correct?
  
(b) ' f ' is strictly increasing in  ,  (a) There are two possible situation for the
 4 2 line L
  (b) A possible equation of line L is
(c) There exists a number ' c ' in  0,  such
 2 2x  y  2 2  0
that f '  c   0 (c) A possible equation of line L is
(d) The equation f  x   0 has only two 2x  y  2 2  0
  (d) A possible equation of line L is
roots in  0, 
 2 x  2y  2 2  0
36. Let ' f ' be a real valued function defined on 39. Let f : R  R defined by
the interval  0, by x 2  2 k x  16
f  x  ,  2  k  0 and
x x 2  2 k x  16
f  x   ln x   1  sin t dt . Then which of x2
0
g  x  0 f ' t  dt, then
the following statement(s) is/are true?
(a) f ' x exists for all x   0,   and f ' is (a) g  x  increases  x   2 and has local
continuous on  0,   , but not differentiable maximum at x   2
on  0, (b) g  x  increases  x  2 and has local
(b) f " x  exists for all x   0,   minimum at x  2
(c) There exists   1 such that (c) g  x  decreases  x  2 and has local
f '  x   f  x  for all x ,  maximum at x  2

(d) There exists   0 such that (d) g  x  decreases  x   0,2 and has local
maximum at x  0
f  x   f '  x    for all x   0,  
x 1
2 3
40. If f  x  then f  x is

37. Let f  x   3  4  x 2
  1  4 x 2
, x2
x   2, 2  then which of the following (a) One - One in  2,  
statements is/are correct? (b) One - One in (0, 1)
(a) The least value of ‘f ’ is 256 (c) One - One in  ,0
27
(d) One - One in (1, 2)
(b) The greatest value of ‘f ’ is 28
(c) The number of critical point(s) of ‘f ’ is 3
DIFFERENTIATION, APPLICATIONS OF DERIVATIVES (AOD) 60

x3 45. If the minimum value of


41. If the function f  x   3 is decreasing,
1 x  x 1 x  2 x  3 x  4 is M, then M + 4
then x  a,   . Find the value of is ________

1 46. f  x is a polynomial of 6th degree and


 a  a   . (where [.] denotes greatest
 2 f  x  f  2  x  x  R. If f  x   0 has
integer function). 4 distinct real roots and two real and equal
42. If composite function f1  f 2  f 3 ....... f n  x    is roots then sum of roots of f  x   0
an increasing function and the number of
   
decreasing functions of all fi 1  i  n  47. Let f :  1,1    tan1,  tan1
 4 4 
functions are r and rest are increasing. The defined by f  x   tan x  tan1 x and the
2
n 1
maximum value of r  n  r  is when n is derivative of f  x at x  0 is ' k ' then the

multiple of 4, maximum value of r  n  r  is value of 4 is
k
2 2
n 1
when n is odd and
n 
when n is 48. Let f  x   x  tan3 x, g  x  is inverse
 4
 
even but not multiple of 4. Evaluate      . function of f(x), find 28 g    1
4 
43. If equation of normal to the curve
y
49. If f : X  Y be a function defined by y = f (x)
y  1  x   sin 1  sin 2 x  at x  0 is given
such that f is both one-one and onto then there
by ax  by  1 then value of 3a  2b is exists a unique function g : Y  X such that
44. Suppose a room, containing 12000 m3 of air is for each y  Y , g  y   x iff y  f  x  . The
originally free of carbon monoxide. Beginning function g so defined is called the inverse of f
at t = 0, smoke containing 4% carbon 1
and denoted as f  x  g  x .
monoxide is introduced into the room at a rate
3 If f : R  R be a invertible function such that
of 1.0m / min and the well circulated mixture
is allowed to leave the room at the same rate. f 1  x  g  x and x1, x2 are two distinct
Extended exposure to carbon monoxide roots of the equation f  x   g  x  . Then
concentration as much as 0.00012 is harmful
to the human body. If T (in min) is the g  x2   g  x1 
value of
approximate time at which this concentration x2  x1
T  (a) must be 1
is reached, then   (in minutes) (where [.] (b) must be 1
4
denotes the greatest integer function) is (c) may be 3
3 (d) cannot determine
_______ ( ln  0.997  3.00510 )
DIFFERENTIATION, APPLICATIONS OF DERIVATIVES (AOD) 62

55. Match the following: 2


58. The curves C1 : y  x  3; C2 : y  kx , k  1
2

intersect each other at two different points.


Column-I Column-II
The tangent drawn to C2, at one of the points
A. If x  y  1, the minimum p. 3
2 2

of intersection A   a, y1  a  0 meets C1
value of x  y is
B. If maximum value of q.  2 again at B 1, y2   y1  y2  . Then the value of
1 a= ?
y  a cos x  cos 3 x occurs at
3 (a) 4 (b) 3
 (c) 2 (d) 1
x  , then value of ‘ a ’ is
6 59. Tangent at P1 other than origin on the curve
C. If r. 3 y = x3 meets the curve again at P2. The
f  x   x  2sin x,0  x  2 is tangent at P2 meets the curve again at P3 and
increasing in the interval area of PP
1 2 P3
so on, then equals
 a , b  then a  b is area of P2 P3 P4
D. If equation of tangent to the s. 3 (a) 1 : 20 (b) 1 : 16
curve y  e  x /2 where it crosses (c) 1 : 18 (d) 1 : 2
x y 60. The portion of the tangent to the curve
the y-axis is   1 , then
p q 2 a a  a2  y2
2
x  a  y  log between
p  q is 2 a  a2  y2
t. 2 the point of contact and the x-axis is of length
(a) a2 (b) 2a
(a) A–p; B–r; C–q; D–s (c) a (d) 3a
(b) A–q; B–s; C–s; D–r 61. A sector subtends an angle 2α at the centre
(c) A–s; B–q; C–p; D–r
then the greatest area of the rectangle
(d) A–r; B–p; C–q; D–r inscribed in the sector is (R is radius of the
 xe ax ; x0 circle)
56. f  x    2 3
; a  0.
 x  ax  x ; x0
R2 

(a) R 2 tan (b) tan
Find interval in which f  x increases 2 2 2
Part-B R2
(c) R 2 tan  tan 
(d)
57. Let f : R  R be a differentiable function, 2
2 62. Let f  x  be a polynomial of degree 3
such that  f '  x    f  x  f   x  for all x. If
satisfying f  3  5, f  1  9, f  x  has
 4
f(0) = 1 and f  0   9 (i.e. 4th derivative at x
minimum at x  0 and f   x  has maximum
= 0). Then possible value of f ' 0 is equal to at x  1. The distance between the points
where local maximum and local minimum of
(a) 3 (b) 2 f  x  occur, is
(c) 1 (d) 1
(a) 3 2 (b) 15
(c) 2 5 (d) 4 3
DIFFERENTIATION, APPLICATIONS OF DERIVATIVES (AOD) 63

63. Statement 1: In  ABC , (a) If f   0  exists, then f   0  also exists


3 3 but
sin A  sin B  sin C 
2 not equal
Statement 2: Let y  f  x be a twice (b) If f   0  exists, then f   0  does not
differentiable function such that f " x   0 in exists
 a, b  then (c) If f   0  exists, then it is equal to f   0 

f  a1   f  a2   f  a3  (d) none of these


a a a 
 f  1 2 3  for 67. Set of values of a for which one negative and
3  3 
two positive real roots of the equation
a1, a2 , a3   a, b
x3  3x  a  0 are possible, is _____.
(a) Statement-1 is True, Statement-2 is True;
(a)  0, 2  (b)  0, 4 
Statement-2 is a correct Explanation for
Statement1 (c)  2, 4  (d)  0, 10 
(b) Statement-1 is True, Statement-2 is True; 68. The number of values of a  such that the
Statement-2 is NOT a correct explanation 3 2
equation x  x  x  a  0 has three integer
for Statement-1 roots (not necessarily distinct) is _____.
(c) Statement-1 is True, Statement-2 is False (a) 1 (b) 2
(d) Statement-1 is False, Statement-2 is True (c) 3 (d) 4
64. Let f  x   2 x  2  x  , 0  x  2 . The number 69. The maximum value of x2  y 2 such that the
x real numbers x and y satisfy

of solution of f f  f  x     2
is
x 2  y 2  4 1  x  is _____.
(a) 2 (b) 4
(c) 8 (d) 12 (a) 24  16 2 (b) 24  16 2
  3  (c) 12  8 2 (d) 12  8 2
65. If f :  ,    1,1 ; f  x   sin x and
2 2  70. f  x    x    x here [.] is integral part
g :  0,     1,1 ; g  x   cos x also of x and {.} fractional part of x
f 1 1
 x  g  x   , f 1 1
 x  g  x   functions then f  x  is _____.
x   1,1 then (a) Continuous in  2,2 
(a) max  2     3 (b) Non differentiable at 3 points in  2,2 
7 (c) Monotonically increasing in  2,2 
(b) max   2   
2 (d) Discontinuous at 2 points in  2,2 
(c) min  2     3 71. 1  sin 1 x   cos 1 x  if (  x denotes
7 greatest integer function)
(d) min   2  
2 (a) x   cos1, sin1
66. If f  x  y   f  x  g  y   f  y  g  x  x , (b) x  sin1, 1
y ,(where f  x  is not identically (c) x   cos 2, 0 
(zero)), then _____. (d) x   cos1, 1
DIFFERENTIATION, APPLICATIONS OF DERIVATIVES (AOD) 64

72. Let x , y , z be real numbers with values of x, then number of integral values of
  ‘a’ is
x yz such that x  y  z  and let 78. The number of points on
12 2
P  cos x. sin y. cos z then y  tan 1 x,  x   0,   , whose image in
1 y  x is the centre of the circle with radius
(a) Minimum value of P is
8 
units and which is at a minimum
1 2 2
(b) Minimum value of P is
4 
distance of units from the circle.
2 3 2 2
(c) Maximum value of P is
4 79. Let f  x  be thrice differentiable function in
2 3
(d) Maximum value of P is
8
 a, b , a  1  2  3  4  5  b and
73. The line y  mx  1 touches the curves f  a   f  b    2, f  1   f  3   3,
y   x4  2x 2  x at two distinct points f   2   f   4    3, f  5    1, then the
P  x1 , y1  and Q  x2 , y2  . The value of minimum number of roots of the equation
x12  x22  y12  y22 is ____ f  x  f "'  x   f '  x  f " x   0 in  a, b  is
74. If f : N  N and  e x  2  e 2 , x   2

x2  x1  f  x2   f  x1  x1 , x2  N and 80. f  x    x 2  x   ,  2  x  2
   nx, x  2
f  f  n    3n n  N , then f  2  is equal to 
_____. If y = f(x) has local maxima at x = –2, then
 1 range of  is
 1 
 2  sin  e x if x0 (a)  ,8
75. f  x    x
 (b)  8,  
 0 if x0
Number of points where f  x  has local (c)  8,8

extrema when x  0 be n1 and n2 be the (d)  ,  8  8,  

value of global minimum of f  x  then  e x  2  e 2 , x   2



n1  n2  81. f  x    x 2  x   ,  2  x  2
   nx , x  2
76. ABCD and PQRS are two variable rectangles, 
such that A, B, C and D lie on If y  f  x  is derivable at x = 2, then  
PQ, QR, RS and SP respectively and
(a) n  64 
perimeter ' x ' of ABCD is constant. If the
x (b) n  4 / e 
maximum area of PQRS is 32, then 
4 (c) n  4 / e 2 
77. f  x   2e   a  5a  6  e
x 2 x
(d) n  64 / e 2 
 10 a  2a 2  11 x  3 is increasing for all
DIFFERENTIATION, APPLICATIONS OF DERIVATIVES (AOD) 68

89. f  x is a polynomial function f :  91. f  x is a polynomial function f : 

such that f  2x   f   x  f   x  . such that f  2x   f   x  f   x  .


The value of f  3 is _____. Equation f  x  x has _____.
(a) 4 (b) 12 (a) Three real and distinct roots
(c) 15 (d) 18 (b) one real root
90. f  x is a polynomial function f :  (c) Four real and distinct roots
(d) Two real and distinct roots
such that f  2 x   f   x  f   x  . 92. Find minimum and maximum distance of
f  x is _____. x 2  xy  y 2  1 from origin
(a) one-one and onto
(b) one-one and into
(c) many-one and onto
(d) many-one and into
QUADRATIC EQUATIONS 69

Quadratic Equations

Part-A 1
5. If lies between roots of the quadratic
2
1. The product of uncommon real roots of the
equation x4 + 2x3 – 8x2 – 6x + 15 = 0 and x3 + equation 6 x 2  3 x cos   sin 2   0 , then true
4x2 – x – 10 = 0 is   3 
set of values of  in  ,  is equal to
(a) −6 (b) −5 2 2 
(c) 5 (d) 6  5 7 
(a)  , 
2. The number of value of k for which  6 6 
(x2 – (k – 2) x – 2k) (x2 + kx + 2k – 4) is a   5 
perfect square is (b)  , 
2 6 
(a) 1 (b) 2
 4 5 
(c) 0 (d) none of these (c)  , 
3. Consider a sequence of quadratic expressions  3 3 
Qr  x   ar x2  br x  1  2 4 
where (d)  ,    
r
 3 3 
ar   1 , br   2,   r  0,1, 2,.....n 6. If one root of equation
2 2
Statement-1 : If b r 1  b  4 and br  2  br  k
r x2  4 a x  a  f  a  0 is three times of the
where k is positive real constant. Then roots
other then minimum value of f  a  is
  0,1 of all the quadratic equations
1 1
(a) (b)
Qr  x   0 , will be in harmonic progression. 6 10
Statement-2 : x = 1, lies between and outside 1 1
(c) (d)
5 12
the roots of equation Qr  x   0 if r is odd and
7. Consider the quadratic equation
even respectively. 2
a x  b x  c  0, a, b, c  N . If the given
(a)Statement-1 is True, Statement-2 is a
equation has two distinct real roots belonging
correct explanation for Statement–1.
(b)Statement-1 is True, Statement-2 is True, to 1, 2  then
Statement-2 is not a correct explanation for (a) 1  a  5 (b) a  5
Statement-1. (c) a  4 (d) a  3
(c) Statement-1 is True, Statement-2 is False. 8. a , b, c are all different and non-zero real
(d) Statement-1 is False, Statement-2 is True. numbers which are in arithmetic progression.
x2  2 x  3 If the roots of the quadratic equation
4. If  1 , then x is equal to a x 2  b x  c  0 are  and  such that
x2  4 x  5
1 1
(a)  , 5   5,    ,  and 2  2 are in geometric
 
(b)  4, 4 progression, then the value of
a
is
c
(c)  , 5   4, 4   5,   (a) 1 (b) 2
(d) R (c) 3 (d) 4
QUADRATIC EQUATIONS 70

9. The set of values of ' a ' for which in-equation (a) P  R (b) R  T
2 (d) T  P  R
 a  1 x   a  1 x   a  1  0 is true for all (c) T  P
x  2 is 12. Consider quadratic equation

7   7  a  3 x2  2ax   a  2  0, a  3 . Let P, Q,
(a)  ,   (b) 1, 
3   3 R, S, T be five sets such that:
P is set of all value of a for which the product
(c)  ,1 (d)  3, 2
of roots of quadratic equation is positive.
10. Consider quadratic equation Q is set of all value of a for which the product
2
 a  3 x  2ax   a  2  0, a  3, a  R . Let of roots of quadratic equation is negative.
P, Q, R, S, T be five sets such that: R is set of all value of a for which the product
P is set of all value of a for which the product of real roots of quadratic equation is positive.
of roots of quadratic equation is positive. S is set of all value of a for which the roots of
Q is set of all value of a for which the product quadratic equation are real.
of roots of quadratic equation is negative. T is set of all value of a for which the roots of
R is set of all value of a for which the product quadratic equation are complex.
of real roots of quadratic equation is positive. Which is correct ?
S is set of all value of a for which the roots of (a) least positive integer for set S is 3
quadratic equation are real. (b) least positive integer for set R is 4
T is set of all value of a for which the roots of (c) greatest positive integer for set T is 2
quadratic equation are imaginary. (d) none of the above
Which is correct ? (where R is the set or real 13. Let a1  a2  a3  a4  a5  a6
numbers) p  a1  a2  a3  ....  a6
(a) P  Q   q  a1a3  a3 a5  a5 a1  a2 a4  a4 a6  a6 a2
(b) R  P
r  a1a3 a5  a2 a4 a6 , then the equation
(c) P  Q  R  2,3
2 x3  px 2  qx  r  0 has
(d) All of the above
(a) one root between (a1 , a2 )
11. Consider quadratic equation
(b) two roots between (a1 , a3 )
 a  3 x2  2ax   a  2  0, a  3 . Let P, Q,
(c) two roots lies between (a1 , a4 )
R, S, T be five sets such that:
P is set of all value of a for which the product (d) two roots lies between (a3 , a5 )
of roots of quadratic equation is positive. 14. f  x   ax2  bx  c is a quadratic polynomial
Q is set of all value of a for which the product
of roots of quadratic equation is negative. if 3a  3b  c  1 and 4a  2b  c  0 which
R is set of all value of a for which the product of the following is / are not possible
of real roots of quadratic equation is positive.  a, b, c  R 
S is set of all value of a for which the roots of (a) b 2  4ac  0
quadratic equation are real. (b) b 2  4ac  0
T is set of all value of a for which the roots of
(c) b 2  4ac  0
quadratic equation are complex.
Which is correct regarding sets P, R and T ? (d) c  4  2 3
QUADRATIC EQUATIONS 71

15. If a , b, c , d  R satisfies (a) A - P; B - S; C - R; D - Q


2 (b) A - Q; B - S; C - R; D - P
a 2  b 2  c 2  d 2  ab  bc  cd  d   0,
5 (c) A - P; B - R; C - S; D - Q
then (d) A - Q; B - R; C - S; D - P
18. Let a , b  N , a  b and the two quadratic
(a) c  a  b (b) b  d
2 equation (a  1) x 2   a 2  2  x  a 2  2a  0
c
(c) a  (d) d  a  c
3 and (b  1) x 2   b 2  2  x   b 2  2b   0 have
16. If a , b , c are positive integers such that a common root. The value of ab is
a  b  c and the quadratic equation 19. If α and β are the roots of the equation x2 –
 a  b  2 c  x2   b  c  2 a  x a(x + 1) – b = 0, where a, b  R  0 and
  c  a  2 b   0 has a root in  1,0  then  a  b  0, then the value of
(a) b  c  a 1 1 2
 2  is equal to
(b) c  a  2 b 2
  a   a a  b
(c) Both roots of the given equation are 20. If  and  are roots of equation
rational 2
 log 2 x   4  log 2 x   1  0 , then value of
(d) The equation a x 2  2 b x  c  0 has both
log    log   equals
negative real roots
17. Match the following: 1 1 1
21.    3 x3 has k real roots then k
Column I Column II x x 1 x  2
(A) If  ,      are the two (P) 5 is equal to
22. The number of real values of the parameter
roots of the equation 2
1  8  log10 x 
2 ‘k’ for which  log16 x   log16 x  log16 k  0
2
1 then will have unique solution
log10 x  2  log10 x 
23. The least positive integral value of real  so
the value of
 3
 1
that the equation (x – a) (x – c) (x – e) + (x –
 32
b) (x – d) = 0, (a > b > c > d > e) has distinct
(B) Number of solution(s) of (Q) 11 real roots is ______
equation
24. If x 2  b1 x  c1  0 has two roots  ,  and
1  1
log 2  x 2  3  log1/3  x   , x 2  b2 x  c2  0 has two roots   2,   2
2  x
x  0 is where b1 , b2 , c1 , c2 are real numbers and also
(C) Number of Integers (R) 0
satisfying the equation b12  b22  k  c1  c2  then the value of k is
4 x 2
4 equal to :
x  is 25. The number of integers n such that the
x x
equation nx2 + (n + 1) x + (n + 2) = 0 has
(D) If (S) 4
log c 2 log b 125  log10 8 log c 10 rational roots only is ___________ .
where c  0, c  1, b  0, b  1 ,
then b is
QUADRATIC EQUATIONS 72

26. If a , b , c , d are non-zero numbers such that f  x  ax2  2bx  b and


c, d are the roots of the equation
g  x   5x2  3bx  a meets at some point for
x  a x  b  0 and a , b are the roots of the
2

all real values of b.


equation x2  cx  d  0, then the value of p n
Let Tr    r  am  and Sn   Tr  n  N 
a  b  c  d is m 1
r 1

Sum of all the possible values of ‘n’ for


Part-B
which Tn vanishes, is ______
27. If the quadratic equation x 2  36 x    0 has
(a) 10 (b) 15
 
roots α and β such that α, β  N and    0 (c) 21 (d) 20
5
and  assumes minimum possible value then 31. If ax 2  bx  1  1 for all x in [0, 1] then

 2  2 (a) a  8 (b) b  8

  (c) b  0 (d) a  b  16
(where {.} denotes fractional part functions)
32. The set ' S ' of all real ' x ' for which
3 3 x1
(a) (b)
8 16  x2  x 1  1 contains
111 111 (a)  5, 1 (b)  1,1
(c) (d)
34 17
(c)  1,0 (d)  3,1
28.  a  1 x   a  1 sin sin 2  x
4 2 2 1 3

33. Given the inequality ax  k 2  0 . The


  cos cos 2  a  1  0 . Find
1 2
the set of
complete set of values of ‘a’ so that
values of a so that above equation have roots Column I Column II
of opposite in sign. (A) The inequality is valid for (P) R
(a) [-2, 3] (b) [-1, 1] all values of x and k is
(c) [-2, 1] (d)  (B) There exists a value of x (Q) 
29. The value of ‘a’ for which the quadratic such that the inequality is
expression ax2  2a  3 x  6 is positive for valid for any value of k is
exactly three integral values of x is (C) There exists a value of k (R) {0}
such that the inequality is
 3 1
(a)   ,   valid for all values of x is
 5 2
(D) There exists values of x (S) R  0
 3 1 and k for which inequality is
(b)   ,  
 5 2 valid is
 3 1 (T) {1}
(c)   ,  
 5 6 (a) A – P; B – S; C – R; D-Q
(d) None of these (b) A – Q; B – S; C – R; D-P
30. Let am  m  1,2,3,.... p  be the possible (c) A – P; B – R; C – S; D-Q
(d) A – Q; B – R; C – P; D-S
integral values of ‘a’ for which the graphs of
QUADRATIC EQUATIONS 73

34. If X 2  2Ps X  s  0 , s = 1, 2, 3 are three 36. If system of inequality x 2  2 x    0 and


equations of which each pair has exactly one x 2  x  2  0 has unique solution, then
root common and no root is common to all number of such value of  is / are _____
three equations, then the number of solutions 37. Let f(x) = 2x3 – 6x + 1. Then number of real
of the triplet  p1 , p2 , p3  is roots of equation f(f(x)) = 0 are ________

35. If  ,   ,   ,   and   ,  be the roots of


ai x 2  bi x  ci  0 for i = 1, 2, 3 respectively
and             
1
 3 a  b  c m
  i i i   n where m and n be the
i 1 ai 
integers then m2  n2  m  n is
SEQUENCE AND SERIES 74

Sequences and Series

Part-A 4. Let  an  be an arithmetic sequence whose


1. ABCD is a square of length a , a  N , a  1 . first term is 1 and  bn  be geometric

L1, L2, L3 , …… be points on BC such that sequence whose first term is 2. If the common
Let
ratio of geometric sequence is half the
BL1  L1 L2  L2 L3  ......  1 and M 1 , M 2 , M 3 .... common difference of arithmetic sequence,
be points on CD such that then the minimum value of
CM 1  M 1M 2  M 2 M 3  ......  1 . Then  a4b1  a3b2  2a1b3  is equal to
a 1
25 3
  AL 2
n  Ln M n2  is equal to (a) (b)
n 1 12 2
1 2 25 3
(a) a  a  1 (c)  (d)
2 12 2
1 5. The sum of first ' n ' terms of the series
(b)  a  1 2a  1 4a  1
2 1 3 7 15
    ..... is
1 2 4 8 16
(c) a  a  1 4a  1
2 (a) 2 n1 (b) 1  2  n
1 (c) 2  n  n  1 (d) 2  n  n  1
(d)  2a  1 4a  1
2 6. The sequence a1 , a2 , a3 ,.... satisfies
2. The sequence a1 , a2 , a3 ,....... satisfies a1  19, a99  99 & n  3, an is the
2 arithmetic mean of the first  n  1 terms,
a1  1, a2  2 and an  2   an , n  1, 2,3,....
an 1
then a2 
22009 (a) 179 (b) 81
the value of . a2012 is
2011 (c) 79 (d) 181
(a) 2010 C1005 (b) 2011
C1006 n n

2011 2012 7. Find the value of   rs 2r 3s where


(c) C1005 (d) C1006 r 1 s 1

3. Let A  {1 ,  2 ,  3 ,  4 ,  5 ,  6 }, ‘a’ is the  rs  0, if r  s



number of subsets of A consisting of 3  rs  1, if r  s
elements; ‘b’ is the number of ordered triplets 6
(a)  6 n  1 (b) 6 n  1
that can be formed using elements of A 5
without repetition and ‘c’ is the number of 1
(c)  6 n  1 (d) none
such triplets if repetition is allowed. Then the 5
digits of (a + b + c + 1) are in 
1
(a) increasing A.P. 8. Values of  r  r  1 r  2  r  3  ____
r 1
(b) decreasing A.P.
1
(c) increasing G.P. (a) 1 (b)
2
(d) decreasing G.P.
1
(c) (d) 0
18
SEQUENCE AND SERIES 77

20. a, b, c are positive integers forming an 2


27. If ,  are the roots of t1 x  4 x 1  0 and
increasing G.P and b  a is a perfect cube and
log 6 a  log 6 b  log 6 c  6 then a  b  c   ,  are the roots of t2 x2  6 x 1  0 and
21. The series of natural numbers is divided into  ,  ,  ,  are in H.P. then
group 1;2,3,4; 5,6,7,8,9,10,11; 12 to 26.... and (a) t1  t2  5 (b) t1  t2  12
th
so on. The sum of the numbers in the n (c) t1  8 (d) t2  5
group is a. 2 2 n 1   2n  b  2 n 1  n  1, then 28. Consider the sequence an given by
evaluate a + b. 1
2010 2 2010 a1  , an 1  an2  an , Let
2012! k .k ! k. k ! 2
22. The value of 2010
  k   k is
2 k 1 2 k 1 2 1 1 1
Sn    .....  then find the
______ a1  1 a2  1 an  1
50
 2r  value of  S 2012  , where  denotes greatest
23. If S   tan 1  4 2  and value of
r 1 r r 2 integer function.
2552 (a) 1 (b)  e / 2 
cot S  , then the value of k is _____.
425k
(c)  e  (d)   1
24.If in an A.P. sum of its first 3n terms is twice
the sum of next ' n ' terms of it and the ratio of 29. For ABC , if
4 4 4
the sum of its first 2n terms to the next 2n 81  144a  16b  9c  144abc, (where
11 k  7 notations have their usual meaning), then
terms of it is ' k ' then the value of is
14 (a) a  b  c
1 1 1 (b) A  B  C
25. Let Hn  1   ... and
2 3 n 3 3
1 (c) Area of ABC 
Tn  , then the value of 8
 n  1 H n H n1 (d) Triangle ABC is right triangle
T1  T2  T3  ...    is  
30. Let x, y, z   0,  are first three consecutive
 2
Part-B terms of an arithmetic progression such that
26. Let a1 , a2 , a3 ,.... an be in G.P. If the area cos x  cos y  cos z  1 and
bounded by the curves y 2  4 an x and 1
sin x  sin y  sin z  , then which of the
2
y  4an  an  x  be An , then the sequence 2
following is/are correct?
A1 , A2 , A3 ,....., An are in
(a) cot y  2
(a) A.P.
(b) G.P. 3 2
(b) cos  x  y  
(c) H.P. 2 2
(d) none of these (c) tan 2 y  2 2
(d) sin  x  y   sin  y  z   0
SEQUENCE AND SERIES 78

31. a1 , a2 .... are distinct terms of an A.P. We call 1


(c) 2
5.7 .13.53
(p, q, r) an increasing triad if a p , aq , ar are in
1
G.P. where p , q , r  N such that p  q  r . If (d) 2
5.7 .11.53
(5, 9, 16) is an increasing triad, then which of 34. Many sums can be easily evaluated by
the following option is/are correct telescoping them, that is, by putting them in
(a) If a1 is a multiple of 4 then every term of n
the Form  f  r  1  f  r  . The idea can be
the A.P. is an integer r 1

(b) (85, 149, 261) is an increasing triad extended to evaluating product also, that is,
by putting them in the form  F  r  1 . After
n
1
(c) If the common difference of the A.P. is , r 1 F r 
4
cancellation we collect the remaining terms to
1
then its first term is get our product.
3
th
The sum
(d) Ratio of the  4k  1 and 4kth term can 3 4 2009
  ......... 
be 4 1 2  3 2  3  4 2007  2008  2009
32. If an is no. of ways to split positive integer ‘n’ equals
1 1 1 1
in all possible ways, for example a3  4 as 3 (a)  (b) 
3 2009 2 2010
can be splitted as 1 + 1 + 1, 2 + 1, 1 + 2 and 3.
Then (c) 1  1
(d) 1  1
2 2009 3 2010
(a) an  2n  4an  2 35. Many sums can be easily evaluated by
(b) an  an1  an  2 ; n  2 telescoping them, that is, by putting them in
n
(c) an  an1  2  an 2 ; n  2 the Form  f  r  1  f  r  . The idea can be
r 1
(d) an  2 n 1 extended to evaluating product also, that is,
33. Many sums can be easily evaluated by by putting them in the form  F  r  1 . After
n

telescoping them, that is, by putting them in r 1 F r 

the Form
n
cancellation we collect the remaining terms to
 f  r  1  f  r  . The idea can be
r 1 get our product.
extended to evaluating product also, that is, by The sum equals
putting them in the form  F  r  1 . After
n
1 2 100
F r 
  ..........  equals
r 1 14  4 24  4 1004  4
cancellation we collect the remaining terms to 1 10000 5100 
(a)   
get our product. 4 10001 10202 
Let the nth term of a sequence be given by (b)
1 10000 5050 

 
2
n n2 2 10001 10202 
tn  ,n  3.
n2  3n (c) 1 10000  5050 
Then the product P = t3 t4 t5 ………. t50 equals 4 10001 10202 
1 1 10000 5050 
(a) (d)   
2
5 .7.13.53 2 10001 10202 
1
(b) 2
5.7 .13.53
SEQUENCE AND SERIES 79

36. Match Column I with Column II: 39. Let n1 , n2 , n3 ,...... be increasing sequence of
Column I Column II natural numbers each of which leaves
n
1 (p) 1
(A) lim  remainder ' r ' when it is divided by  r  1
n 
k 1  k  1 k  k k 1
is equal to for r  2, 3, 4, ....., 9 . When n2008 is divided
n
6k (q) 2 by 11, the remainder is
(B) lim 
k 1  3  2  3  2k 1  40. A cricket player played n  n  1 matches
n  k k k 1

is equal to during his career and made a total of

n k2 
1 (r) 3  n  1  2 n 1  n  2 
(C) lim  2 is equal to runs. If the player made
n  1 4
k 1 4
k 
4 k . 2 n  k  1 runs in the k th
match 1  k  n  .
1 (s) -1 Then the value of ' n ' is
(D) x1  and
2
n
1
xk 1  xk 2  xk .T   then
x
k 1 t  1

T  is equal to (where .


denotes G.I.F.)
(t) 0

(a) A-p; B-r; C-q; D-s


(b) A-r; B-p; C-s; D-r
(c) A-p; B-q; C-p; D-p
(d) A-r; B-p; C-q; D-r
 1 1 1 
1  2  2  2  ......upto  
2 3 4
37. The value of  
 1 1 1 
 1  2  2  2  ......upto  
 2 3 4 
is _______.
38. In a ABC , if a is the arithmetic mean and b,
c are two geometric means between any two
sin 3 B  sin 3 C
positive numbers, then is
sin A sin B sin C
equal to ______
MATRICES AND DETERMINANTS 80

Matrices and Determinants

Part-A 4. If A and B are two square matrices and AB =


I, then which of the following is not true?
1. ‘A’ is a 3  3 matrix with entries from the set
(a) BA = I (b) A–1 = B
1,0,1. The probability that ‘A’ is neither –1
(c) B = A (d) A2 = B
symmetric nor skew symmetric is 5. Let ‘A’ and ‘B’ be two given matrices such
39  36  33  1 that AB = A and BA = B, then A2 B2 equals
(a) (a) A (b) B
39
(c) I (d) 0
39  36  33
(b) 4 1
39 6. Let A    then A100 equals
 9 2 
39  1  301 100 
(c) 10 (a) 
3 
 900 299 
39  33  1  301 100 
(d) (b) 
39  900 301

a b   400 101 
2. A  and MA = A2m, m  N for some (c) 
 b a  
 901 200 
matrix M, then which one of the following is
 400 99 
correct? (d)  
 901 200 
 a 2mb 2m 
(a) M   2m   2 2 3
b a 2m 
 
7. If A0   1 3 4  and
m 1 0
(b) M   a 2  b 2     2 2 3
0 1 
1 0    4 3 3

(c) M  a m  b m    B 0   1 0 1  Bn  adj  B n 1  , n  N
0 1
m 1  a b  4 4 3 
(d) M   a 2  b 2   
 b a  and I is an identity matrix of order 3.
C1, C2 , C3 represent the column matrix of B 0
3. The number of positive integral solutions as shown,
 x, y, z  of the equation  4  3    3
   
C1   1  , C2   0  and C3   1  , then
x3  1 x2y x 2z
 4   4   3 
xy2 y3  1 y 2 z  11 is __
xz 2 yz 2 z3  1 answer the following .

(a) 0 (b) 3  
det A0  A02 B02  A03  A04 B04  ......upto10 terms 
(c) 6 (d) 12 (a) 1000 (b) –800
(c) 0 (d) –8000
MATRICES AND DETERMINANTS 81

 2 2 3 10. P is a non-singular matrix and A, B are two


8.
 
If A0   1 3 4  and matrices such that B  P 1 AP then the true
 2 2 3 statements among the following are
(a) A is invertible iff B is invertible
  4 3 3
(b) B n  P 1 An P  n  N
B 0   1 0 1  Bn  adj  B n 1  , n  N
 4 4 3 
(c)    R, B   I  P 1  A   I  P (I is the
identity matrix)
and I is an identity matrix of order 3.
(d) A,B are both singular matrices
C1, C2 , C3 represent the column matrix of B 0
11. Let A n is a n  n matrix in which diagonal
as shown,
elements are 1, 2,3,....., n
 4  3    3
    (i.e.,
C1   1  , C2   0  and C3   1  , then
a11  1, a22  2, a33  3,..., a ii  i,... a nn  n )
 4   4   3 
and all other elements are equal to ' n ' then
answer the following . B1  B 2  ......  B 49 
(a) An is singular for all ' n '
(a) B 0 (b) 7 B 0
(b) A n is non-singular for all ' n '
(c) 49 B 0 (d) 491
(c) det . A5  120
 2 2 3 (d) det . An  0
 
9. If A0   1 3 4  and 12. Match the following:
 2 2 3 For 3  3 matrix, a i j represents the elements
  4 3 3
of i t h row and j t h column.
B 0   1 0 1  Bn  adj  B n 1  , n  N
Column I Column II
 4 4 3 
(A) aij  a jk  aki  0, (p) This represents
and I is an identity matrix of order 3. a symmetric matrix
then
C1, C2 , C3 represent the column matrix of B 0
(B) (q) This represents
aij  a jk  aki  0,
as shown, a skew symmetric
then
 4  3    3 matrix
   
C1   1  , C2   0  and C3   1  , then (C) (r) This represents
 4   4   3  i j j k
 1 aij   1 a jk a matrix whose all
k i diagonal elements
answer the following .For a variable matrix the   1 aki  0, then
are zeroes
equation A0 X  C1 will have
(s) Value of
(a) Unique solution determinant is zero
(b) No solution (a) A → (q, r, s); B → (p, q, r, s);C → (q, r, s)
(c) Infinitely many solutions (b) A → (q, s); B → (p, q, r, s); C → (q, r, s)
(d) None of these (c) A → (q, r, s); B → (p, q, s); C → (q, r, s)
(d) A → (q, r, s); B → (p, q, r, s); C → (q, s)
MATRICES AND DETERMINANTS 82

13. Match the following: (a)  w , z  (b)  z , y 


Column-I Column-II
(A) If A 2 then 2A1
 (p) 1
(c)  y,x (d)  z , x 
17. If A and B are respectively a symmetric and a
(where A is a matrix of order
skew symmetric matrix such that AB = BA
3)
then
1 (q) 4 1
(B) If A then (a)  A  B  A  B is orthogonal matrix
8
adj  adj 2 A   (where A is a when (A – B) is non-singular
1
matrix of order 3) (b)  A  B   A  B  is orthogonal matrix
2
 A2  B 2 and (r) Does not
when (A + B) is non-singular
(C) If  A  B 1
exist (c) det [  A  B   A  B ]  1 and
A  2 & B  (where A and B
1
are matrices of odd order) det [ A  B   A  B ]   1
(D) A22  2, B23  3 (s) 0 (d) det [  A  B 
1
 A  B ]  1 and
and C44  4 & 1
det [ A  B   A  B ]  1 .
ABC 
  2 
(a) A → q; B → p; C → s; D → r 18. If A   1  (where  2  1 and 1, 2 ,
(b) A → q; B → r; C → p; D → s  1 2 
(c) A → q; B → r; C → r; D → s 1, 2 are non-zero) satisfies the
(d) A → q; B → q; C → s; D → r equation x 2  k  0 , then
14. Let A be the set of all 3  3 symmetric (a) trace A = 0
matrices all of whose entries either 0 or 1. (b) 1 2 < 0
Five of these entries are 1 and four of them are (c) det A = k
0. If n is the number of such matrices, then n/2 (d) det A = – k.
is 19. If the matrix A and B are of 3×3 and (I –
1 x x2 AB)–1A is invertible, then which of the
following statements is/are correct?
15. If x x2 1 3 then the value of
2
(a) I – BA is not invertible
x 1 x
(b) I – BA is invertible
x3  1 0 x  x4 (c) I – BA has for its inverse I + B(I –AB)–1A
0 x  x4 x3  1 is _______. (d) I – BA has for its inverse I + A(I –BA)–1B
x  x4 x3  1 0 20. If A, B are two square matrices of same order
such that A + B = AB and I is identity matrix
Part-B of order same as that of A, B, then
16. If x, y, z, w  R satisfy the following system of (a) AB = BA
equations x  y  z  w 1; (b) A  I  0
x  2 y  4 z  8 w  16 ; (c) B  I  0
x  3 y  9 z  27 w  81 and (d) A  B  0
x  4 y  16 z  64 w  256, then the pairs
which has H .C .F . as 2 is
MATRICES AND DETERMINANTS 83

21. Match the following: a b c


23. det P  c a b , where 'P' is an
2 2  b c a
The matrix A   aij    and
22
 2 1 orthogonal matrix. Then the value of
 cos   sin   a  b  c is
S  are such that STAS =
 sin  cos  
24. Statement 1: If
diagonal matrix =D =  dij  for n
2 2 1  x  x2   a0  a1 x  a2 x 2  .........  a 2 n x 2 n
  then
Some suitably chosen value of    0,  .
 4  an a n3 a n 3
50
Let M = A = mij  a n 1 a n4 a n4  0
22

Column-I Column- a n 2 a n5 a n5


II
Statement 2: If
(A) If  is the angle that makes (p) 0 n
S T AS a diagonal matrix then 1  x  x2   a0  a1 x  a2 x 2  .........  a 2 n x 2 n
3tan 2 is then a0  a3  a6  .........  0
4m11  m22 (q) 4 (a) Statement-1 is True, Statement-2 is True;
(B) The value of 51
is
3 Statement-2 is a correct Explanation for
(C) The value of d11  d 22 is (r) 1 Statement-1
m12  m21 (s) 5 (b) Statement-1 is True, Statement-2 is True;
(D) The value of is Statement-2 is NOT a correct explanation
350
for Statement-1
(a) A → q; B → r; C → p; D → s (c) Statement-1 is True, Statement-2 is False
(b) A → r; B → q; C → s; D → p (d) Statement-1 is False, Statement-2 is True
(c) A → q; B → s; C → r; D → p
(d) A → q; B → r; C → s; D → p
22. If  αβ  δβ  = γγγ such that α,β, δ, γ represents
a number from 1 to 9 & α,β, δ, γ are all
different digits & αβ, δβ are two digit numbers
& γγγ is a three digit number, and the trace of
 1 2 0
0  1 1  a
the matrix A   is a, then is
0 0  3 7
 
1 1 0 
equal to
PERMUTATIONS AND COMBINATIONS(PNC), BINOMIAL THEOREM 84

Permutations and Combinations (PnC), Binomial


Theorem

Part-A 7. The number of different hyperbolas


represented by the equation
1. Let A be a factor of 108. The number of 2 2 2
positive integral solution of x1 x2 x3  A are  5 x  5   5 y  5   m Cn  3 x  4 y  2 

(a) 160 (b) 320 where ' m ' and ' n ' are integers satisfying
(c) 200 (d) 960 1  n  m  5 is
2. The number of distinct throws when 10 (a) 25 (b) 10
identical dice are thrown is (c) 7 (d) 6
(a) 15 C5 (b) 15
C4 8. Statement 1: A  { x : x is a prime number,
15 x  30 } then number of distinct rational
(c) C9 (d) none of these
numbers whose numerator and denominator
3. n-similar balls each of weight w when belong to ' A' is 93
weighed in pairs the sum of the weights of all
the possible pairs is 120 when they are Statement 2: p  Q  q  0 and p , q  I
q
weighed in triplets the sum of the weights
(a) Statement-1 is True, Statement-2 is True;
comes out to be 480 for all possible triplets,
Statement-2 is a correct Explanation for
then n is
Statement-1
(a) 5 (b) 10
(b) Statement-1 is True, Statement-2 is True;
(c) 15 (d) 20
Statement-2 is NOT a correct explanation
4. In the xy-plane, how many straight lines
for Statement-1
whose x-intercept is a prime number and
(c) Statement-1 is True, Statement-2 is False
whose y-intercept is a positive integer pass
(d) Statement-1 is False, Statement-2 is True
through the point (4, 3)?
9. Let y  x  3, y  2 x  3, y  3 x  2, y  x  3
(a) 1 (b) 2
be four lines.
(c) 3 (d) 4
Statement – I: The number of triangles
5. If all the rectangles which are not squares are
formed on a chess board, how many different formed is 4 C 3 .
values of area of these rectangles will be Statement – II: Number of distinct points of
possible, if sides are odd numbers. intersection between various lines will
(a) 14 (b) 12 determine the number of possible triangles.
(c) 6 (d) 7 (a) Statement-1 is True, Statement-2 is True;
6. In how many different ways can a sum of Rs. Statement-2 is a correct Explanation for
20 be paid in one rupee coins, 50 paise coins Statement-1
and 25 paise coins if each variety of coins is (b) Statement-1 is True, Statement-2 is True;
available in unlimited number Statement-2 is NOT a correct explanation
(a) 231 (b) 210 for Statement-1
(c) 441 (d) 400 (c) Statement-1 is True, Statement-2 is False
(d) Statement-1 is False, Statement-2 is True
PERMUTATIONS AND COMBINATIONS(PNC), BINOMIAL THEOREM 85

10. Number of zeroes in the end in the product of 17. The number of triplets  x, y, z  of positive
56 × 67 × 78 × ……...× 3031 is
integers, satisfying
(a) 111 (b) 147
2 x  2 y  2 z  2336 is _____.
(c) 137 (d) 136
(a) 72 (b) 6
11. If a, b, c, d be four consecutive coefficient in
(c) 3 (d) 18
the binomial expansion of (1 + x)n, then
18. Statement-1 : Coefficient of x14 in (1 + 2x +
value of the expression
3x2 + …. + 16x15)2 is 560.
  b 2 ac 
     (where x > 0 and n n  n 2  1
  b  c   a  b  c  d  
Statement-2 :  r n  r  
r 1 6
n  N ) is (a) Statement-1 is True, Statement–2 is True;
(a) Positive (b) negative Statement-2 is a correct explanation for
(c) zero (d) depend on n Statement-1
12. The number of natural numbers less than or (b) Statement-1 is True, Statement–2 is True;
equal to 2012, which are relatively prime Statement-2 is NOT a correct explanation
to 2012 are for Statement-1
(a) 1004 (b) 1006 (c) Statement-1 is True, Statement–2 is False
(c) 1005 (d) 4 (d) Statement-1 is False, Statement–2 is True
28
13. The coefficients of x in the expansion of (2 –
19. Statement-I: when 2323 is divided by 53, the
x3 + x6)30 is
remainder is 30
(a) 0 (b) 1
Statement-II: If ‘n’ is the odd natural
(c) 14 (d) 28
number (finite), greater than 1, then the
37
14. If 3  80   k , where   , then k is
number of zeroes at the end of 99n  1 is 2
_____. (a) If both the statements are TRUE and
(a) 78 (b) 3 Statement-2 is the correct explanation of
(c) 2 (d) 9 Statement-1
15. If each coefficient in the expansion of the (b) If both the statements are TRUE but
n
expression x 1  x   n   in powers Statement-2 is NOT the correct
of x is divided by the exponent of explanation of Statement- 1
corresponding power, then the sum of the (c) If Statement-1 is TRUE and Statement-2
values thus obtained is equal to _____. is FALSE.
2n 2n  1 (d) If Statement-1 is FALSE and Statement-2
(a) (b) is TRUE.
n 1 n 1
n
2 1 2n 1  1
(c) (d)
n 1 n 1
16. The number of distinct terms in the expansion
13 14

of x  y 2  
 x2  y  is _____.
(a) 27 (b) 29
(c) 28 (d) 25
PERMUTATIONS AND COMBINATIONS(PNC), BINOMIAL THEOREM 86

m 3
(c) 7! 15!
20. If J m   m Cr mCr 3 and m1 , m2 are two
r 0 (d) None of these
values of m satisfying 5 Jm = 3Jm+1 , then 25. There are 8 events that can be scheduled in a
correct statement is/are (where [.] denotes week, then the total number of ways that
greatest integer function) these 8 events are scheduled on exactly 6
8 days of a week is
(a) m1  m2 
7 (a) 210  6!
46 (b) 7!  266
(b) m1m2 
7 (c) 56  7!
(c) 1  m1 1  m2    8 (d) 210  7!
(d) 1  m1 1  m2    8 26. Match the following columns:
21. For natural number m, n if (1 – y)m (1 + y)n = Column I Column II
(A) A Five digit number (p) 692
1  a1 y  a 2 y 2  .... and a1  a2  10 then
divisible by 6 is to be formd by
(a) m < n
using the digits 0, 1, 2, 3, 4 and
(b) m > n
8 without repetition.
(c) m + n = 80
(B) The number of ways in (q) 236
(d) m – n = 20
which we can arrange 4 letters
22. Which of the following statement (s) is/are
of the word MATHEMATICS
true?
is given by
(a) maximum value of P such that 3P divides (C) Out of 10 white, 8 black (r) 150
100! is 48 and 6 red balls, the number of
(b) maximum value of P such that 3P divides ways in which one or more
50! is 22 balls can be selected is given
(c) maximum value of P such that 3P divides by
99  97  95  .........  51 is 14 (D) The letters of the word (s) 1680
P
(d) maximum value of P such that 3 divides MOLANA are written in all
25! is 10 possible(s) orders and these
23. There are 8 events that can be scheduled in a words are written out as in a
week, then dictionary. Then the rank of the
The total number of ways in which the events word MOLANA is
can be scheduled is (t) 2454
8 (a) A–r; B–t; C–p; D–q
(a) 87 (b) 7
(b) A–r; B–p; C–s; D–r
(c) 7! (d) 8
(c) A–s; B–q; C–p; D–r
24. There are 8 events that can be scheduled in a
(d) A–r; B–p; C–q; D–r
week, then
The total number of ways that the schedule has
at least one event in each days of the week is
(a) 28  5040
(b) 7!8!
PERMUTATIONS AND COMBINATIONS(PNC), BINOMIAL THEOREM 87

27. Match the following: position, is


Column-I Column-II (c) The total number of ways in (r) 40320
(a) The position of the word (p) 4 which they can sit around a
AITS in the dictionary circular table such that an
formed by the words using Indian wife and an American
the letters of the word AITS wife are always on either side
(b) The sum of all numbers of (q) 2 of the unmarried person, is
12! (d) If all the nine persons are to (s) 24320
the form where
a !b!c ! be interviewed one by one then
a, b, c  N satisfy the total number of ways of
a  b  c  12 is 33k
where ‘ k ’ arranging their interviews such
is that no wife gives interviews
(c) Total number of even (r) 3 before her husband, is
divisors of 1323000 which
are divisible by 105 is 2k  10 (a) (a)  (r); (b)  (s); (c)  (q); (d)  (p)
, then ‘ k ’ is (b) (a)  (s); (b)  (r); (c)  (q); (d)  (p)
(d) In the process of (s) 6 (c) (a)  (r); (b)  (s); (c)  (q); (d)  (p)
numbering the pages of a (d) (a)  (r); (b)  (s); (c)  (p); (d)  (q)
large book, the printer uses 29. Match the following:
1890 digits. The number of Column-I Column-II
pages in the book is a three (A) Number of distinct terms (p) 212
digit number which is 6 K 6 in the expansion of
then K  x  y  z
16
is
(a) (a)  (q); (b)  (p); (c)  (s); (d)  (r)
(B) Number of terms in the (q) 97
(b) (a)  (q); (b)  (p); (c)  (s); (d)  (s)
expansion of
(c) (a)  (q); (b)  (s); (c)  (p); (d)  (r) 6 6

(d) (a)  (q); (b)  (r); (c)  (p); (d)  (s)   


x  x2 1  x  x2 1 
28. Match the following: is
There are 2 Indian couples, 2 American (C) The number of irrational (r) 4
couples and one unmarried person 100

Column-I Column-II terms in  8


562  is
(a) The total number of ways in (p) 22680 (D) The sum of numerical (s) 153
which they can sit in a row coefficients in the expansion of
such that an Indian wife and an 12
 x 2y 
American wife are always on 1    is
 3 3 
either side of the unmarried
person, is
(a) A → (s); B → (r); C → (q); D → (p)
(b) The total number of ways (q) 5760
(b) A → (s); B → (r); C → (p); D → (q)
in which they can sit in a row
(c) A → (r); B → (s); C → (q); D → (p)
such that the unmarried person
(d) A → (s); B → (q); C → (r); D → (p)
always occupy the middle
PERMUTATIONS AND COMBINATIONS(PNC), BINOMIAL THEOREM 88

30. Suppose Mr. Ramesh have rupee 2,3 and 5 number and 1  1   2   3 ...   k then value
notes. In how many ways he can get a sum of
 1 1 1 
rupees 83 such that atleast one note of each of 1   ...   is _____.
type is present and the number of 2 rupee  1  2 k 
note(s) is less than number of 3 rupee note(s) Part-B
which is again less than the number of 5 38. There are ' n ' coplanar straight lines, no two
rupees not (s). being parallel and no three are concurrent.
31. If the maximum value of ' p ' such that 3 p The number of different new straight lines
divides 99  97  95  ......  51is 2K , then K  that will be formed by joining the intersection
32. If n is the number of ways in which 15 points of the given lines are
identical blankets can be distributed among six n  n  1 n  2  n  3 
(a)
beggars such that everyone gets atleast one 8
blanket and two particular beggars get equal   2 
n  1 n 
(b)
blankets and another three particular beggars 2
get equal blankets, then the value of n/2 is n  n  1
(c)
33. Let ai denote the number of non-negative 4
integral solutions of the equation n  n  1 n  2 
(d)
X1  X 2  ...  X11  i  i . If i varies from 1 to 9 4
and a1  a2  a3  ...  a9  S , then find the 39. Total number of integers ' n ' such that
2  n  2000 and H .C.F of ' n ' and 36 is
value of S.
one, is equal to
34. If the coefficient of x5 in (1 + x)21 + (1 + x)22 +
(a) 666
….+(1+x)30 is  nCr +  mCk, , , n, r, m, k
(b) 667
being integers and nCr, mCk are binomial
(c) 665
coefficients, then the value of (n + r) + (m (d) 668
+ k) is 40. If a seven digit number of the form
9 10
Cr a1a2 a3 a4 a5 a6 a7 (all digits being different) is
35. The value of r 0
10
Cr 10 Cr 1
is equal to
randomly formed, then the probability that
36. When the terms in the binomial expansion of this will satisfy the condition
n
 1  a1  a2  a3  a4  a5  a6  a7 is _____.
 x  1 4  are arranged in decreasing
 2x  5 7
(a) (b)
powers of x , the coefficients of the first three 1134 1134
terms are in arithmetic progression. The 9 5
(c) (d)
number of terms in the expansion with integer 1134 648
powers of x is _____. 41. The number of positive integers n such that 2n
37. 1, 1 ,  2 ,  3 ,..., k are divisions of number divides n is
(a) exactly 1
N  2 n  1  2n  1 where 2n  1 is a prime
(b) exactly 2
(c) infinite
(d) none of these
PERMUTATIONS AND COMBINATIONS(PNC), BINOMIAL THEOREM 89

42. If (a) Statement-1 is True Statement-2 is True


n and Statement-2 is correct explanation of
 
f  n     r n. n 1Cr 1  r. nCr 1   2 r  1 nCr 
Statement-1.
r 1
, then _____. (b) Statement-1 is True Statement-2 is True
and Statement-2 is not a correct
(a) f  n   n2  1
explanation of Statement-1.
2
(b) f  n    n  1  1 (c) Statement-1 is True and Statement-2 is
2 False
(c) f  n    n  1  1
(d) Statement-1 is False and Statement-2 is
10
True
(d)  f  n   374
n 1 46. Which of the following is/are true
43. Let n be a prime number. (a) 56  5C1.46  5C2 .36  5C3 .26  5C4 .16
Statement-1: 2nCn – 2 is divisible by n2.
 6 C2 . 5
Statement-2: nC1, nC2, ......, nCn–1 are divisible
by n. (b) 65  6C1.55  6C2 .45  6C3.35  6C4 .25
(a) Statement-1 is True, Statement-2 is True;
 6C1.15  0
Statement-2 is a correct explanation for
Statement-1 (c) 66  6C1.56  6C2 .46  6C3 .36  6C4 .26
(b) Statement-1 is True, Statement-2 is True;  6C5 .16  720
Statement-2 is NOT correct explanation
for Statement-1 (d) 65  6C1.55  6C2 .45  6C3 .35  6C4 .25
(c) Statement-1 is True, Statement-2 is False  6C5 .15  5C2 . 6
(d) Statement-1 is False, Statement-2 is True 47. Let there are m seats in a row, out of which n
44. Statement 1: 24 n  2n  7 n  1 is divisible by are to be occupied by n persons, given m is an
square of 14  n  even number and m  2n .
Statement 2: The number of seating arrangement such that
n n n n each person has exactly one neighbor (assume
1  x   1  C1 x  ...  Cn x  n 
n is even)
(a) Statement-1 is True, Statement-2 is True;
(a) m n Pn
Statement-2 is a correct Explanation for
m   n /2 
Statement-1 (b) C n /2  n !
(b) Statement-1 is True, Statement-2 is True; (c) m  n 1
Cn / 2  n !
Statement-2 is NOT a correct explanation m  n 1
(d) Pn / 2
for Statement-1
48. Let there are m seats in a row, out of which n
(c) Statement-1 is True, Statement-2 is False
are to be occupied by n persons, given m is an
(d) Statement-1 is False, Statement-2 is True
45. Statement-1: If ‘n’ is an odd prime then even number and m  2n .
n n Out of any two seats located symmetrically
integral part of  5 2   
5  2  2n1 is about the middle of the row, atleast one is
divisible by 20n empty then total number of way of seating
Statement-2: If ‘n’ is a prime, then (a) m /2 Cn 2n. n ! (b) m Pn
n
C1 , n C2 ,...n Cn 1 must be divisible by n (c) m /2
Pn (d) m Pn  m /2 Pn
PERMUTATIONS AND COMBINATIONS(PNC), BINOMIAL THEOREM 90

n 5

49. If 1  px  x 2   1  a1x  a2 x 2  ...  a2 n x 2 n , 1 
expansion of   x log10 x  is
where n  , p . x 
1000 and x  1 , then the
If n  40, p  3, then which of the following
value of x is
is true.
(a) A → (p); B → (q); C → (s); D → (r)
(a) 135a5  6a6  4a4
(b) A → (p); B → (r); C → (q); D → (s)
(b) 105a5  6a6  76a4 (c) A → (p); B → (q); C → (r); D → (s)
(c) 105a5  6a6  36a4 (d) A → (q); B → (p); C → (r); D → (s)
(d) 135a5  6a6  36a4 53. If the number of ordered pairs of positive
n integers  m, n satisfying m  2n  60 &

50. If 1  px  x 2   1  a1x  a2 x 2  ...  a2 n x 2 n ,
n  2m  60 is 2 K 3  K 2  2 K , then K 
where n  , p  .
54. If the number of ordered triplets  x, y, z  such
The remainder obtained when
a1  5a2  9a3  13a4  ...   8n  3 a2 n is that L.C.M (x, y) = 3375, L.C.M (y, z) = 1125,
L.C.M (z, x) = 3375 is equal to ' k ' then
divided by  p  2  is _____. k  47 is equal to
(a) 1 (b) 2 55. Number of integral solutions of
(c) 3 (d) 0 a  b  c  d  e   11, where a, b < 5, c  4
2 n

51. If 1  px  x  2
 1  a1x  a2 x  ...  a2 n x 2n
, and d, e  -2 is N, then unit digit of N is
where n  , p . _________
If p  3 and n is even number, then the 56. Let 2a1  3a2  5a3  7a4  9a5 be divisible by 4
where a1 , a2 , a3 , a4 , a5 are digits. Largest
value of a1  3a2  5a3  7a4  ......
possible number of values of
  4n  1 a2n is _____.
( a1 , a2 , a3 , a4 , a5 ) is  9000 k then k is
(a) n (b) 2 n  1
_____.
(c) 2 n  2 (d) 2n
52. Match the following:
Column-I Column-II
(A) The minimum value of (p) 18
abcd if
log3  a  b   log3  c  d   4
is
(B) The number of distinct (q) 225
terms in the expansion of
14
 1 1
 x  y    is
 x y
(C) The reminder when (r) 89
86
 23  is divided by 100 is
(D) If the third term in the (s) 100
SOLUTION OF TRIANGLES (SOT) 91

Solution of Triangles (SOT)

Part-A 6. If in a right angled triangle ABC of perimeter

1. In  ABC ,  
3  3 , 4sin A cos B  1  0 and tan A is

cos A  cos B  cos C  tan A tan B tan C , then real then _____.
(a) angles are in HP
the triangle will
(b) angles are in GP
(a) be acute angled (c) In-radius of triangle ABC is 1
(b) be right angled (d) circum radius of triangle ABC is 1
(c) be obtuse angled 4 24
7. In a triangle, sin A  , tan C  , c = 1000
(d) not exist 5 7
2. In a  ABC , if b 2  c 2  1999 a 2 , then units. The area of triangle in square units is
cot B  cot C _____.
is equal to _____.
cot A 106
(a) 10 4 (b)
1 1 3
(a) (b)
999 1999 104
(c) 106 (d)
(c) 999 (d) 1999 3
3. If 2a cosB  c in a  ABC with sides a, b, c 8. Let ABC be a triangle such that
2 2 2 2
A A C  A  B  C   6s 
then tan  tan  2tan  is equal to  cot    2cot    3cot    
2 2 2  2  2  2   7r 
where s and r denotes its perimeter and its
_____. inradius respectively.
(a) 1 (b) 21 In triangle ABC a : b : c is _____.
1 1 (a) 7 : 13 : 19 (b) 13 : 40 : 45
(c) (d)
2 3 (c) 7 : 20 : 23 (d) 26 : 81 : 90
9. In any triangle ABC,
4. In a  ABC , if A  1,2  and internal angle
Statement-I :
bisectors through B and C are y  x and  A B C
log e  cot  cot  cot 
y  2 x . The in-radius r of  ABC is equal  2 2 2
to _____. A B C
 log e cot  log e cot  log e cot
1 2 2 2 2
(a) (b) Statement-II: In a triangle ABC,
3 3
cot A cot B  cot B cot C  cot C cot A  1
1 1 (a) If both the statements are TRUE and
(c) (d)
3 2 Statement-2 is the correct explanation of
5. A triangle ABC has A  B  120 and the Statement-1
circumradius is 8 times the inradius. The value (b) If both the statements are TRUE but
of cos C equals _____. Statement-2 is NOT the correct
explanation of Statement- 1
7 15 (c) If Statement-1 is TRUE and Statement-2
(a) (b)
8 16 is FALSE.
3 2 (d) If Statement-1 is FALSE and Statement-2
(c) (d)
4 3 is TRUE.
SOLUTION OF TRIANGLES (SOT) 92

10. Following the usual notations, in a triangle 14. Let O be a point inside  ABC such that
ABC, if  
3  1 a  2b and A = 3B, then C OAB  OBC  OCA   .
cannot be equal to _____. cosec2 A  cosec2 B  cosec2C is equal to
  _____.
(a) (b) (a) cot 2  (b) cosec2
3 4
2  (c) tan 2 (d) sec2
(c) (d)
3 6 15. Let O be a point inside  ABC such that
11. The in-circle of  ABC touches side BC at D OAB  OBC  OCA   .
Then difference between BD and CD (R is Area of  ABC is equal to _____.
circum-radius of  ABC )  a 2  b2  c 2 
A BC
(a)   tan 
 4 
(a) 4 R sin sin
2 2
 a2  b2  c2 
A BC (b)   cot 
(b) 4 R cos sin  4 
2 2
 a 2  b2  c 2 
(c) b  c (c)   tan 
 2 
bc
(d)  a2  b2  c2 
2 (d)   cot 
 2 
12. Let triangle ABC and DEF be inscribed in the
same circle. If the triangle are of equal 16. Let ABC is a acute angled triangle and AD is
perimeter, then perpendicular from A on side BC. AD is
(a) cos A  cos B  cos C  cos D  cos E  cos F produced and it is intersecting at point P with
(b) sin A  sin B  sin C  sin D  sin E  sin F circum circle of  ABC . H is the orthocenter
(c) The areas of the two triangles are same of  ABC and R is the circum radius of
A B C D E F  ABC
(d) cos cos cos  cos cos cos
2 2 2 2 2 2 Area of  BHP  _____.
13. Which of the following statements is/are true?
(a) 2R 2 cos 2 Bsin 2 C
(a) In a triangle if the diameter of any
escribed circle equals the perimeter of the (b) 2R 2 cos B sin Bsin 2C
triangle, then the triangle is right angled. (c) 2R 2 sin 2 Bcos2 C
(b) If the in-radius of a triangle is 3cm and (d) 2R 2 cos 2 Bsin 2C
r1 , r2 , r3 be the ex-radii of a triangle, then 17. Let ABC is a acute angled triangle and AD is
1 1 1 1 perpendicular from A on side BC. AD is
   produced and it is intersecting at point P with
r1 r2 r3 3
circum circle of  ABC . H is the orthocenter
1 of  ABC and R is the circum radius of
(c) In a triangle, abc  R rs where a, b, c
4  ABC
are sides of a triangle, R the circum-radius,
Radius of circum circle of  AHC is _____.
r the in-radius and s the semi perimeter.
R
(d) The in-radius of a triangle having sides 16, (a) (b) R
2
63, 65 cm is 7 cm. (c) 2R (d) 3R
SOLUTION OF TRIANGLES (SOT) 93

18. If in a  ABC , 2a 2  4b2  c 2  4ab  2ac , 20. Match the following:


then match the following. If in  ABC , r  2 , r1  4 , s  12 and
Column – I Column – II a  b  c then
(A) cos A 1 Column -I Column -II
(p)
4 (A) Area of  ABC is (p) 22
(B) cos B 7 (B) 4  4R is (q) 24
(q)
8 (C)  B of triangle is 4
(r) sin 1
(C) cos C (r) 0 5
(D) (s) 1 3
(s) sin 1
sin  A  C  4
1 (a) A → (q); B → (q); C → (r)
(t)  (b) A → (p); B → (q); C → (r)
2
(c) A → (q); B → (p); C → (r)
(a) A–p; B–r; C–q; D–s
(d) A → (q); B → (q); C → (p)
(b) A–r; B–p; C–s; D–r
21. Match the following:
(c) A–s; B–q; C–p; D–r
Column-I Column-II
(d) A–q, s; B–p, q, s, t; C–q, s, t; D–q, s, t
(A) If the equations x  y  1 , (p) 1
19. If a, b, c are the sides of a triangle and
a 3 , b 3 , c 3 are the roots of x 3  px 2  qx  r  0 ,
 c  2 x   c  4 y  6 ,
2 2
then match the following.  c  2  x   c  4  y  36
Column-I Column-II are consistent, then c can be
(B) Number of solutions of (q) 2
(A) sin A  sin B  sin C (p)
3 3 3
the equation
3sin A sin Bsin C  8 3 p  14, r  8
1 1 2
(B) (q)   in
sin x sin 2 x sin 4 x
2 2
a sin A  b sin B  c sin C 2 p  36, r  27  0, 2  does not exceed
 14 2 (C) In triangle ABC, with (r) 3
(C) sin AsinBsinC  23 (r) usual notation, the value of
p  9, r  8  a  b  c  a b c 
(D) (s)      is
 r1  r2  r3  r1 r2 r3 
a cos 2A  b cos 2B  c cos 2C 63 not less than
p  , r  27
 2s   
2
2 (D) Let z1 , z2 be complex (s) 4
numbers such that z1  z  1
(t)
. Then
p  1, r  8
z1  1  z2  1  z1 z2  1
cannot be less than
(a) A–p, q; B–p, s; C–p, r, t; D–t
(t) 5
(b) A–r; B–p; C–s; D–r (a) A→q,s; B→q,r,s,t; C → p,q,r,s; D→p
(c) A–s; B–q; C–p; D–r (b) A→q,s; B→q,r,s,t; C→p,q,r,s; D→p, q
(d) A–r; B–p; C–q; D–r (c) A→q,s; B→q,r,t; C→p,q,r,s; D→p, q
(d) A→q; B→q,r,s,t; C→p,q,r,s; D→p, q
SOLUTION OF TRIANGLES (SOT) 94

22. Match the following: 5


25. For a triangle ABC, R  and r  1 . Let I
Column-I Column-II 2
(A) If p  2 p cos x  673 (p) 8
2 be the in-centre of the triangle and D, E, F be
the feet of the perpendiculars from I to BC,
x CA, AB respectively. Then the value of
and tan  7 , the integral
2 IA  IB  IC
is equal to _____.
value of p is 2ID  IE  IF
(B) If sin   cos   m then (q) 9 26. Perpendiculars are drawn from the angles A,
B, C of an acute-angled triangle on opposite
the maximum value of m2 is
sides and produced to meet the
(C) r1 , r2 , r3 are the radii of (r) 2 circumscribing circle. If these produced parts
the circles drawn on the are   ,  ,  respectively, then the value of
altitudes  a /     b /     c /   is _____.
PD, PE and PF of  PBC , tan A  tan B  tan C
 PCA ,  PAB respectively Part-B
as diameter where P is the A B
27. In  ABC , if BC  1 , sin  x1 , sin  x2 ,
circumcentre of the acute 2 2
angled  ABC . The A B
cos  x3 and cos  x4 with
minimum value of 2 2
2010 2009
1  a 2 b2 c2   x1  x 
    is ( a , b , c     3   0 ,then length of AC
18  r12 r22 r32  x x
 2  4
are sides of  ABC ) is equal to _____.
(D) In  ABC , a  6 , b  3 (s) 25 (a)  1 (b)  1
4 1
and cos  A  B   then the (c) (d) 1
5 2
area of the  ABC is 28. In a scalene acute angled triangle  ABC ,
the line joining circumcentre and orthocentre
(a) A→ (s); B→ (r); C → (p); D → (q) is parallel to the side BC, then  A  .
(b) A→ (s); B→ (p); C → (p); D → (q)    
(c) A→ (r); B→ (r); C → (p); D → (q) (a)  0,  (b)  , 
 6  6 3
(d) A→ (s); B→ (q); C → (p); D → (r)
   
23. In a  ABC , the internal angle bisectors AI, (c)  ,  (d)  , 
 6 4  3 2
BI and CI are produced to meet the opposite
29. If r1, r2 , r3 are the ex radii of triangle ABC
sides in A , B and C , respectively. Find the
27  AI  BI  CI  which are the roots of
maximum value of . x3  11 x2  36 x  36  0 and I1, I2 ,I3 are the
AA  BB  CC
24. If in triangle ABC, the line containing ex-centres opposite to the vertices A,B,C
circumcentre and ortho–centre is parallel to respectively, then the area of triangle I1 I 2 I3
AC, then tan A tan C  _____. is (in sq.units)
(a) 36 (b) 30
(c) 6 (d) 12
SOLUTION OF TRIANGLES (SOT) 97

41. Match the following: 42. The length of the altitude on side AC of the
Column-I Column-II triangle ABC is equal to AB  BC . The value
(A) Area bounded by the (p) 23 CA CA
of sin sin  cos is _____.
curves 2 2
y   cos A  cos B  cos C ,
 A B C
y   7sin sin sin 
 2 2 2
where A, B, C are angles of
a triangle and [.] denotes
greatest integer function and
the curve x  y  2 is
 (q) 3
n
(B) If I n   e x  sin x  dx ,
0

25I 7
then is equal to
I5
(C) If the total number of (r) 2
positive unequal integral
solutions of the equation
x  y  z  w  20 is 24k ,
then k 
(D) If (s) 21
y 2
 5 y  3 x  x  1  2 x
2

x  , then the number of


lying in the range of y are

(a) A→ (q); B→ (s); C → (p); D → (r)


(b) A→ (r); B→ (s); C → (p); D → (q)
(c) A→ (q); B→ (s); C → (r); D → (r)
(d) A→ (q); B→ (p); C → (p); D → (r)
TRIGONOMETRY, TRIGONOMETRIC EQUATIONS 98

Trigonometry, Trigonometric Equations


sin x  sin x
Part-A 5. e e  4a  0 will have exactly four
10 10
r different solutions in  0, 2  if
1
1.  tan
r 1 s 1
   _____.
s (a) a 
(a) 55  e 1
(b) a    ,  
(b) coefficient of x50 in expansion of  4 4
1   x  51x 50
 1  e 2 
2
(c) a   , 
1  x   4 
(c) 50 (d) none of these
25 x 6. Sum of the series
(d) coefficient of x 25 in expansion of n n n
1  x  C1 sin a  C 2 sin 2a  ...  C n sin na is equal
2. Solution of the equation to _____.
 x 1 1  x  1 
a  na 
tan 1  1 (a) 2n cos n   .sin  
  tan    tan  7  is
2  2 
 x 1   x 
_____. a  na 
(b) 2n sin n   .cos  
1 2  2 
(a) 2 (b)
2 a  na 
(c) 2n sin n   .sin  
(c) 4 (d) none of these 2  2 
 a  na 
3. If 0    , then   sec   is _____. (d) 2n cos n   .cos  
3 2  2 
 7. If
(a) less than
3 1  tan11  tan 21  tan 3  ... 1  tan 45   2n ,
 then n is equal to _____.
(b) greater than
3 (a) 19 (b) 21
2 (c) 23 (d) 25
(c) less than
3 8. If x y z  , tan x tan z  2 and
2
(d) greater than tan y tan z  18 , then tan 2 z  _____.
3
(a) 15 (b) 16
4. The equation sin8 x  b sin 4 x  1  0 will have
(c) 19 (d) 20
a solution if b belongs to 7
 K 
(a)  , 2 9. The value of  tan 2   is _____.
K 1  16 
(b)  2,  
3
(a) (b) 105
(c)  ,  2 2
(d) none of these (c) 35 (d) 21
TRIGONOMETRY, TRIGONOMETRIC EQUATIONS 99

  13. A circle centred at O has radius 1 and


10. Statement 1:  cot 1 2 k 2 
k 1 4 contains the point A. Segment AB is tangent
to the circle at A and AOB   . If point C
Statement 2:
lies on OA and BC bisects the angle ABO,
1
tan 1 2  tan 1  2 k  1  tan 1  2 k  1 , then OC equal to _____.
2k
cos2 
k  NI (a) 1  sin  (b)
1  sin 
(a) Statement-1 is True, Statement-2 is True;
Statement-2 is a correct Explanation for 1 1  sin 
(c) (d)
Statement-1 1  sin  cos2 
(b) Statement-1 is True, Statement-2 is True; 14. If the equation
Statement-2 is NOT a correct explanation sin x  cos  k  x   cos  k  x   2 has real
for Statement-1 solutions, then sin k can be
(c) Statement-1 is True, Statement-2 is False 1 1
(d) Statement-1 is False, Statement-2 is True (a) (b)
4 2
11. Let 3 3
b cos x b  sin x (c) (d) 
 , b . 4 4

2cos 2 x  1 cos x  3sin 2 x tan x
2
 15. If
3
 sin x   sin y 
1 1 3
 3sin 1 x sin 1 y  1 ,
Which of the following are correct?
then which of following may be true?
(a) the given equation has a solution if
sin 1 x 
 1  1 (a) 1

b   ,   1, 0,  1  cos y 2
 2  3
x y
(b) the given equation has a solution if (b)  2
sin1
 1   1
b   ,    1,  (c) sin 1 x  sin 1 y  1
 2   3
x y
(c) the given equation has no solution if b = 10 (d) 1
(d) the given equation has no solution if cos1
16. Which one of the following quantities is/are
b = –10
positive?
12. The number of real solutions of the equation
(a) cos  tan 1  tan 4  
 tan x  1 tan x  3 tan x  5  tan x  7   33
_____. (b) sin  cot 1  cot 4  
   (c) tan  cos 1  cos 5  
(a) will be 2 in the interval   , 
 2 2
(d) cot  sin 1  sin 4  
  
(b) will be 4 in the interval   , 
 2 2
  
(c) will be 3 in the interval   , 
 2 2
  3 
(d) will be 4 in the interval   , 
 2 2 
TRIGONOMETRY, TRIGONOMETRIC EQUATIONS 100

17. Match the following: such way that


Column-I Column-II d  P,OA   d  P, AB   d  P,OB   3 where
(A) The number of real roots (p) 3
d  P,OA  , d  P,AB  , d  P,OB represents
of the equation 3 x 2  x  1
perpendicular distances of P from the sides
is/are OA, AB and OB respectively. If area of the
(B) The number of real roots (q) 4 region representing all possible positions of P
of the equation
is k then k 3  _____.

3 cos 1 x   x   0 is/are
2 Part-B
(C) The number of real roots (r) 2 23. If S be the set of all positive solutions of the
of sin x cos y  1 equation
 x, y   0, 2  is/are 1  a  cos  cos  2  b   1  a cos 2  cos   b  ,
which depends on a and b, then the conditions
(D) The least degree of (s) 1
that should be imposed on a and b such that S
polynomial with integer
is non-empty is _____.
coefficient one of the root
may be cos12 is/are a
(a) sin b  1
2
(a) A → (q); B → (p); C → (p); D → (q) a
(b) sin b  1
(b) A → (q); B → (s); C → (q); D → (q) 2
(c) A → (q); B → (s); C → (p); D → (q) (c) a sin b  1
(d) A → (p); B → (s); C → (p); D → (q)
(d) none of these
18. If 2 cot 2 x  5cosec x is equal to 1 for exactly 6
24. Number of solutions of the equation
distinct values of x   0, n  , then the greatest  x   1  3
tan 1  2 
 tan 1  3   belonging to
value of n is _______  1 x  x  4
1  the interval  0, 1 is _____.
19. If   tan 1  2 tan 2    tan 1  tan  
and
 3
 (a) 0 (b) 1
distinct values of tan  are  ,  ,  then find (c) 2 (d) infinite
    . A B
25. In  ABC , if BC  1 , sin  x1 , sin  x2 ,
20. If 3sin   sin  2    , then find the value of 2 2
A B
 cot   cot       cot   3cot  2     . cos  x3 and cos  x4 with
2 2
21. If the system of equations 2010 2009
 x1  x 
2
  3   0 ,then length of AC
 
sin x cos 2 y  a 2  1  1 and  
 x2   x4 
cos x sin 2 y   a  1 has a solution, then the is equal to _____.
value of a  2 is _____. (a)  1 (b)  1
22. Consider a  OAB formed by the point 1
(c) (d) 1
2
 
O  0,0  P  x, y  , A  2,0  , B 1, 3 , be any
arbitrary interior point of  OAB moving in
TRIGONOMETRY, TRIGONOMETRIC EQUATIONS 101

26. The set of values of the parameter  for (d) Statement–1 is False, Statement–2 is True.
which the equation n cos  2  3r 1 
      29. If S n    ,   18 and
2  cos  cos  sin    cos  sin   0 r 1 sin  3r  
2 2 2  2 2
has a solution is _____.
n cos  2  3r 1  
P  n   ,   36 , then
(a) R    4,0  (b)   4,0 r 1 cos  2r  

(c)  ,   (d)  1, 0  _____.


(a) S(2012) + P(2012) = 2010
27. Statement-1:
(b) S(2012) + P(2012) = 2012
   1  sin 2 x      1  cos 2 x   (c) P(2011) – S(2011) = 2009
tan   2    tan   2  1
 4  1  sin y    4  1  cos y   (d) P(2011) – S(2011) = 2008
 1  sin 2 x  30. If
Statement-2: If f  x, y     1 ,
2
 1  sin y  sin x  cos x  tan x cot x  sec x  cosecx  7

   and sin 2 x  a  b 7 , then a  b  14 is


then f  x, y   0, f   x,  y   0
2 2  divisible by _____.
  (a) 2 (b) 3
 x, y   0, 
 2
(c) 4 (d) 7
(a) Statement-1 is True, Statement-2 is True,
Statement-2 is a correct explanation for 31. Which of the following statements is/are true ?
Statement-1. (a) cos  cos 1 x  sin 1  x  2    0
(b) Statement-1 is True, Statement-2 is True,
Statement-2 is not a correct explanation  1 1
(b) sin  sin 1  cos 1   1  2 6
for Statement-1.  2 3
(c) Statement-1 is True, Statement-2 is False.
cos  cos 1 x 2   tan  cot 1 x 2 
(d) Statement-1 is False, Statement-2 is True. (c) minimum value of e    

28. Statement–1: The set of values of  for


is e 2
which the equation cos 1 x  cos x   exhibits
a solution is   cos1,   cos1 3
(d) tan 1 2  tan 1 3 
4
  
Statement–2 : For all     ,  1 , the
 2 

equation cos1 x   x  exhibits exactly one
2
solution.
(a) Statement–1 is True, Statement–2 is a
correct explanation for Statement–1.
(b) Statement–1 is True, Statement–2 is True,
Statement–2 is not a correct explanation
for Statement–1
(c) Statement–1 is True, Statement–2 is False.
TRIGONOMETRY, TRIGONOMETRIC EQUATIONS 102

32. Match the following: 


n (q) 3
(B) If I n   e x  sin x  dx ,
Column-I Column-
0
II
25I 7
(A) If xi  8  1 for i  1, 2, 3 (p) 9
2 then is equal to
I5
where   0, 1, 2, 3,... , then the (C) If the total number of (r) 2
number of positive integral positive unequal integral
solutions of equations solutions of the equation
x1  x2  x3  21 is x  y  z  w  20 is 24k ,
(B) If D n is the digit in the place (q) 55 then k 
n (D) If (s) 21
of the number  r ! , then y 2
 5 y  3 x  x  1  2 x
2
r 1

D1  D3  D5  ...  D 21 is equal to x  , then the number of


lying in the range of y are
(C) Let S  a  , a  100 . If (r) 2
(a) A→ (q); B→ (s); C → (p); D → (r)
the equation
(b) A→ (r); B→ (s); C → (p); D → (q)
 tan 2 x   tan x  a  0 has real (c) A→ (q); B→ (s); C → (r); D → (r)
roots, then number of elements in (d) A→ (q); B→ (p); C → (p); D → (r)
S (where [.] represents greatest 34. If
integer function) 3 p sin x   p  sin x   p 2  p sin x  sin 2 x   1
(D) The number of solutions for (s) 37
has a solution for x, then the number of
3 sin 2 x  cos 2 x  2
 3 sin x  cos x  4 integral values of p are _________
35. If
is/are where x    ,  
f  x   cot 1  4 x 2  6 x  3  cot 1  4 x 2  10 x  7 
(a) A→ (q); B→ (q); C → (p); D→ (r)
 cot 1  4 x 2  14 x  13  ... up to 10 terms,
(b) A→ (s); B→ (s); C → (p); D→ (r)
(c) A→ (q); B→ (r); C → (p); D→ (r)  61
then f   0  is equal to ______
(d) A→ (q); B→ (s); C → (p); D→ (r) 10
33. Match the following: 36. If p  q 0, pr  1  qr and
Column-I Column-II  pq  n
1  q  r  1  r  p 
(A) Area bounded by the (p) 23 tan 1    tan    tan   ,
 1  pq   1  qr   1  rp  2
curves
then n  _____.
y   cos A  cos B  cos C , 37. Consider a triangle having vertices at the
 A B C  2 i / 2   2 i / 6 
y   7sin sin sin  points A e , B e ,
 2 2 2  3   3 
where A, B, C are angles of a
 2  i 5 / 6 
triangle and [.] denotes C e  . Let P be any point on its
 3 
greatest integer function and
incircle, then the value of AP2  BP2  CP 2 is
the curve x  y  2 is
_____.
STRAIGHT LINES 103

Straight Lines

Part-A (d) x2  y2  25
1. Let A  4,  1 , B and C be the vertices of a 4. The equation of circum-circle of a  ABC is
triangle. Let the internal angular bisectors of x2  y2  3x  y  6  0.
angles B and C be x – 1  0 and x – y – 1  0 If A  1,  2 , B   3,2 and the vertex C
respectively. Let D, E and F be the points of
varies then the locus of ortho-centre of
contact of the sides BC, CA and AB
 ABC is a
respectively with the incircle of triangle ABC.
The slope of BC is _____. (a) Straight line (b) Circle
1 (c) Parabola (d) Ellipse
(a) (b) 2 5. The sides of the quadrilateral formed by the
2
(c) 3 (d) 12 angular bisectors of a parallelogram of sides
a , b and angle between sides being  , where
2. Let A  4,  1 , B and C be the vertices of a
triangle. Let the internal angular bisectors of  a  b
angles B and C be x – 1  0 and x – y – 1  0 (a)  a  b  sin  ,  a  b  cos 
respectively. Let D, E and F be the points of 2 2
contact of the sides BC, CA and AB (b)  a  b  sin  ,  a  b  cos 
2 2
respectively with the incircle of triangle ABC.
(c)  a  b sin ,  a  b cos 
 B  C
cot   cot    _____.
2 2 (d)  a  b sin ,  a  b cos 
(a) 2 (b) 3 6. ABC is a triangle right angled at A with
(c) 4 (d) 6 vertices A,B,C in the anti-clockwise sense in
3. Let A  4,  1 , B and C be the vertices of a that order. A  1,2 , B   3,1 and vertex C
triangle. Let the internal angular bisectors of lies on the X – axis. BCEF is a square with
angles B and C be x – 1  0 and x – y – 1  0 vertices B,C,E,F in the clockwise sense in
respectively. Let D, E and F be the points of that order. ACD is an equilateral triangle with
contact of the sides BC, CA and AB vertices A,C,D in the anti-clockwise sense in
respectively with the incircle of triangle ABC. that order. Slope of AF is
If D, E , F are the images of D, E, F in the (a)
7
(b)
7
internal angular bisectors of angles A, B, C 10 9
respectively, then the equation of the (c) 9 (d) 11
circumcircle of  D E F  is _____. 10 10
2
(a)  x  1  y 2  5
2
(b) x 2   y  1  25
2 2
(c)  x  1   y  1  5
STRAIGHT LINES 104

7. ABC is a triangle right angled at A with 10. Statement 1: Consider the point A 0,1 and
vertices A,B,C in the anti-clockwise sense in
B 2,0 and ' P ' be a point on the line
that order. A  1,2 , B   3,1 and vertex C
4 x  3 y  9  0, then coordinates of 'P '
lies on the X – axis. BCEF is a square with
vertices B,C,E,F in the clockwise sense in that such that PA  PB is maximum is
order. ACD is an equilateral triangle with
 12 17 
vertices A,C,D in the anti-clockwise sense in  , 
 5 5
that order.
The abscissa of centroid of  BCE is Statement 2: PA  PB  AB
(a) Statement-1 is True, Statement-2 is True;
(a) –1 (b) 1
2 Statement-2 is a correct Explanation for
1 2 Statement-1
(c) (d)
3 3 (b) Statement-1 is True, Statement-2 is True;
8. ABC is a triangle right angled at A with Statement-2 is NOT a correct explanation
vertices A,B,C in the anti-clockwise sense in for Statement-1
that order. A  1,2 , B   3,1 and vertex C (c) Statement-1 is True, Statement-2 is False
(d) Statement-1 is False, Statement-2 is True
lies on the X – axis. BCEF is a square with
vertices B,C,E,F in the clockwise sense in that 11. Consider the three linear equations,
order. ACD is an equilateral triangle with ax  by  c  0 , bx  cy  a  0 ,
vertices A,C,D in the anti-clockwise sense in cx  ay  b  0 , where a , b , c  . Which of
that order. the following is (are) correct?
2
If D   , then  4   4   (a) If a + b + c = 0 and a2 + b2 + c2 = ab + bc
+ ca, then the lines represent the entire
(a) 2 (b) 3
XY plane.
(c) 4 (d) 5
(b) If a + b + c = 0 and a2 + b2 + c2  ab +
9. ABCD is a parallelogram whose side lengths
bc + ca, then the lines are concurrent.
are a & b  a  b  . The angular bisectors of (c) If a + b + c  0 and a2 + b2 + c2 = ab + bc
interior angles are drawn to intersect one + ca, then the lines are coincident.
another to form quadrilateral. Let '' be one (d) If a + b + c  0 and a2 + b2 + c2  ab +
angle of parallelogram. bc + ca, then the lines are neither
The area of the quadrilateral formed by the coincident nor concurrent.
angular bisectors is 12. If one vertex of an equilateral triangle of side
1 2  1 2 a is the origin and the other lies on the line
(a)  a  b  sin (b)  a  b  sin 
2 2 2 x  3 y  0 , then the coordinates of the third
1 2  1
(c)  a  b  cos (d)  a  b  2 cos  vertex are _____.
2 2 2
(a) (0, a) (b) (0, –a)
 3a a   3a a 
(c)  ,   (d)   , 
 2 2  2 2 
STRAIGHT LINES 105

13. The triangle formed by the lines outside the circle. The length of the tangent
x  y  0, 3 x  y  4  0 and x  3 y  4  0 is drawn to the circle from vertex D is 2 units.
(a) isosceles (b) scalene The radius of the circle is
(c) acute angled (d) obtuse angled (a) 5 (b) 1 10
14. Consider a triangle ABC. The median AD 2
meets the side BC at D. A point E on AD (c) 1 12 (d) 8
is chosen such that AE : DE  1: 3. The 3
19. Each side of triangle ABC is divided into 3
straight line BE extended meets the side AC
equal parts. The ratio of the area of hexagon
CF
at the point F. Then equals PQRSTU to the area of the triangle ABC is
AF
15. A curve ' C ' is such that x–intercept of the (a) 5 (b) 2
9 3
normal drawn at any point ' P ' on it exceeds
the abscissa of ' P ' by 3 and the curve passes (c) 1 (d) 3
2 4
through 1,0 . If the line y  x  k where ' k ' 20. If ' S ' is the area of the given parallelogram
is a variable constant is a tangent to ' C ' then and Q is the area of the quadrilateral formed
the value of 2 k  1 is by the angular bisectors then ratio of the
larger side to smaller side of the
Part-B parallelogram is
16. L1 : 2x  y  50 and L2 : y  mx 1 are two (a)  S  Q 
S
lines in the coordinate plane. A point  x, y is
S  Q  2Q S
said to be an integral point if x , y  . (b)
S
The greatest integral value of m for which the
S  Q  Q2  2 Q S
point of intersection of L1 and L2 has integral (c)
S
coordinates, is _____.
S  Q  Q2  2 Q S
(a) 5 (b) 9 (d)
(c) 47 (d) 51 S
21. P and Q are two points on a line passing
17. L1 : 2x  y  50 and L2 : y  mx 1 are two through (2, 4) and having slope m. If a line
lines in the coordinate plane. A point  x, y is segment AB subtends a right angle at P and Q
said to be an integral point if x , y  . where A  (0, 0) and B  (6, 0), then range of
m is
The square of the distance between the
L1 and L2  2 3 2 2 3 2 
integral points of intersection of (a)  , 
for the greatest and the least integral values of  4 4 
m is _____.  2 3 2   2  3 2 
(a) 5 (b) 20 (b)  ,   ,  
 4   4 
(c) 25 (d) 30
18. ABCD is a square of side 1 unit. A circle (c) (–4, 4)
passes through vertices A,B of the square and (d) (–, –4)  (4, )
the remaining two vertices of the square lie
STRAIGHT LINES 106

22. A curve C which is not a straight line lies in  3 


the first quadrant. The tangent at any point on (c)  2,  (d)  5,7 
 2
C meets the positive directions of the
26. Given pair of lines
coordinate axes at the points A,B. Let ' d ' be
the minimum distance of the curve C from the 2 x2 5x y  2 y2  4 x  5 y  a  0 and the
origin O. line L : b x  y  5  0 then
If OA  OB  1 then d  (a) a = 2
(a) 1 (b)
1 (b) a = – 2
2 2 2 (c) There exist no circle which touches the
(c) 1 (d) 2 pair of lines and the line L if b  5
2 (d) There exist no circle which touches the
23. A curve C which is not a straight line lies in pair of lines and the line L if b   5
the first quadrant. The tangent at any point on 27. Match the following:
C meets the positive directions of the Let ABC be a isosceles triangle with
coordinate axes at the points A,B. Let 'd ' be
AB  AC. If AB lies along x  y  10 and
the minimum distance of the curve C from the
AC lies along 7 x  y  30 and area of
origin O.
If OA.OB  4 then d  triangle be 20 sq. units.
Column-I Column-II
(a) 1 (b) 1 (a) Coordinate of (p) (10, 0)
2 2 2
point B cannot be
(c) 1 (d) 2
2 (b) Coordinate of (q) (4, –2)
24. A curve C which is not a straight line lies in point C cannot be
the first quadrant. The tangent at any point on (c) Centroid of ABC  5 5 
(r)  , 
C meets the positive directions of the cannot be  2 2
coordinate axes at the points A,B. Let 'd ' be (d) Circumcentre of  13 
(s)  3, 
the minimum distance of the curve C from the ABC cannot be  3
origin O. (t) (0, 10)
If AB  1 then d  (a) Aq, s; B  p, r, s, t; Cp, q, r, t; D 
1 1 p, q, s, t
(a) (b)
2 2 2 (b) Aq, r, s; B  p, t; Cp, q, r, t; D  p,
(c) 1 (d) 2 q, s, t
2
(c) Aq, r, s; B  p, r, s, t; Cp, q; D  p,
25. Consider the straight lines q, s, t
L1 : x  y  2, L2 : 2 x  y  3  0 and a (d) Aq, r, s; B  p, r, s, t; Cp, q, r, t; D
variable point P  a , a 2  where a  R. ' P ' lies  p, q, s, t

in the acute angle not containing the origin if


' a ' lies in the interval
 3 5
(a)   4,  3 (b)  , 
 2 2
CIRCLES 107

Circles

Part-A (c) x2  y 2  2ay tan   a 2


1. A pair of tangents are drawn from a point P to (d) x2  y 2  2ay tan   a 2
the circle x 2  y 2  1 . If the tangents make an 5. The equation of the circle of radius 1 unit
intercept of 2 units on the line x  1 , then the touching the circles x2  y 2  2 x  0 is
locus of P is _____. _____.
(a) a straight line (b) a pair of lines
(a) (a) x 2  y 2  2 2 x  1  0
(c) a parabola (d) a hyperbola
2. Let C1 and C2 be the centres of circles (b) x 2  y 2  2 3 y  2  0

touching the X axis at the points T1  2, 0 and (c) x 2  y 2  2 3 y  1  0


(d) x 2  y 2  2 2 x  1  0
T2  8, 0 respectively. P  7, 2 is a point on a
6. The circle, which passes through the points of
line perpendicular to C1C2 and bisecting TT
1 2. intersection of the circles
If PT1 and PT2 intersect the circles with x2  y 2 – 4 x – 6 y  12  0 and
centres C1 and C2 at points A and B 2 2
x  y – 8x  12 y  50  0 , and also passes
PA through the origin, is _____.
respectively, then  _____.
PB (a) (a) 19 x2  19 y 2 – 52 x – 222 y  0
5 29 (b) 19  x 2  y 2  – 2  34 x  111 y   0
(a) (a) (b)
29 5
5 29 (c) 19  x 2  y 2  – 117 x  26 y  0
(c) (d)
29 5 (d) such circle does not exist
3. The coordinates of the point P on the line 7. Let the straight line x  y  8 intersect the
2x  3y  1  0 such that PA  PB is coordinate axes at A and B respectively. Let a
circle C be drawn through the points A and
maximum, where coordinates of A and B are
B. The circle C cuts off an intercept of
respectively A  2, 0 and B 0, 2 , is _____.
minimum length on the line y  x . The
(a) (a) (4, –3) (b) (7, –5) centre of the circle C is _____.
(c) (10, –7) (d) (–8, 5)
(a) (a)  4, 4 (b)  6, 6
4. A circle with centre at the origin and radius
equal to a meets the X axis at the points (c)  3,  3 (d) none of these
A  – a, 0 and B a, 0 . P   and Q    are 8. Let P  ,   be a point in the first quadrant.
two points on this circle so that     2 , Circles are drawn through P touching the
where  is a constant. The locus of the point coordinate axes.
of intersection of AP and BQ is _____. The radius of one of the circles is _____.
2 2
(a) (a) x2 – y 2 – 2ay tan   a 2 (a) (a)     (b)    
(b) x2  y 2 – 2ay tan   a 2
(c)      (d)     2
CIRCLES 108

9. Let P  ,   be a point in the first quadrant. 4


(c) 2  m  (d) m 1
Circles are drawn through P touching the 3
coordinate axes. 14. P and Q are two points on a line passing
The relation between  and  , for which two through (2, 4) and having slope m. If a line
segment AB subtends a right angle at P and Q
circles are orthogonal, is _____.
2 where A  (0, 0) and B  (6, 0), then range of
(a) (a)  2   2  4 (b)      4 m is
(c)  2   2   (d)  2   2  2  23 2 23 2 
(a)  , 
10. Let P  ,   be a point in the first quadrant.  4 4 

Circles are drawn through P touching the  23 2   23 2 
coordinate axes. (b)  ,   ,  
 4   4 
The equation of the common chord of the two
circles is _____. (c) (–4, 4)
(d) (–, –4)  (4, )
(a) x  y     (b) x  y  2 
15. ABC is a right angled triangle right angled at
(c) x  y     (d)  2   2  4 A, with side AC  1 and AB  a , a circle
11. The equation of circum-circle of a  ABC is having AC as diameter cuts the side CB at D
x 2  y 2  3 x  y  6  0. If if CD  b then:
(a) ab  1 (b) ab  1
A  1,  2  , B   3, 2  and the vertex C
b 1 b 1
varies then the locus of ortho-centre of (c)  (d) 
a a2  1 a a2  1
 ABC is a 2 2
(a) Straight line (b) Circle 16. Three are two circles in a parallelogram. One
(c) Parabola (d) Ellipse of them of radius 3 is inscribed in the
12. Let AB be any chord of the circle parallelogram, and the other touches two
x 2  y 2  4 x  4 y  4  0 which subtends an sides of the parallelogram and the first circle.
The distance between the points of tangency
angle of 90 at the point  2,3 then the locus
which lie on the same side of the
of the midpoint of AB is a circle whose centre parallelogram is equal to 3.
is
3
 3 (a) The radius of the other circle is
(a) (1, 5) (b)  1,  4
 2 75
(b) Area of the parallelogram is equal to
 5  5 2
(c)  1,  (d)  2, 
 2  2 (c) Let d1 , d 2 denote the lengths of the
13. The line intersects the circle and at point A diagonals of parallelogram, then the
and B (points being other than origin). The
product d1 , d 2 is equal to 75
range of such that origin divides internally
is (d) Let d1 , d 2 denote the lengths of the
3 4 diagonals of parallelogram, then the
(a) 1  m  (b) m  or m  2 product d1 , d 2 is equal to 95
4 3
CIRCLES 109

1 24. ABCD is a square of side 1 unit. A circle


17. If a tangent of slope of the ellipse passes through vertices A,B of the square and
2
the remaining two vertices of the square lie
x2 y 2 outside the circle. The length of the tangent
 1 is normal to the circle
a 2 b2 drawn to the circle from vertex D is 2 units.
x2  y 2  4 x  2  0 , then the maximum value The radius of the circle is
25. A circle passes through the points (3, 4) and
of ab is _____.
(5, 2) and its centre lies on the line y = 2x.
18. If 7a  2b  14c  t , then the value of t for The radius of the circle is
which the line ax  by  c – 1  0 always 26. The number of points inside or on the circle
passes through a fixed point is _____. x 2  y 2  4 , satisfying
19. A pair of circles, passing through the points tan 4 x  cot 4 x  1  3sin 2 y is
 3, 0 and  5, 0 , and also touching the Y 27. There are two circles whose equations are x2
+ y2 = 9 and x2 + y2 – 8x – 6y + n2 = 0, n  I,
axis, intersect at an angle  . The value of
having exactly two common tangents. Then
cos  is equal to _____. the number of possible values of n is
20. The radius of the circles which pass through 28. The number of points  b, c  lying on the
the point  2,3  and cut off equal chords of 2
circle x 2   y  3  8 such that the equation
length 6 units along the lines y  x 1  0 and
x 2  b x  c  0 has real roots is
y  x  5  0 is ' r ' then  r  is (where [.] 29. Consider the two circles
2 2
denotes greatest integer function) S1  x   y – 1  9  0 and
21. The centres of two circles C1 and C 2 each of 2 2
S2   x – 3    y – 1  9  0 . Match the
unit radius are at a distance of 6 units from following.
each other. Let ' P ' be the midpoint of the line Column – I Column – II
segment joining the centres of C1 and C 2 and (A) The number of common (p) 3
C be a circle touching circles C1 and C 2 tangents to S1 and S2 is
_____.
externally. If a common tangents to C1 and C
(B) The length of the (q) 9
passing through ' P ' is also a common tangent common chord of S1 and S2
to C 2 and C. Then the radius of the circle C is _____ units.
is (C) The length of the (r) 3
22. The number of integral values of  for which common tangents of S1 and
the point (  1,  + 1) lies in the larger S2 is _____ units.
segment of the circle x2 + y2 – x – y – 6 = 0 (D) The greatest distance (r) 2
made by the chord whose equation is x + y – 2 between two points, one on
= 0 is S1 and one on S2 , is _____
23. A curve ' C ' is such that x–intercept of the units.
normal drawn at any point ' P ' on it exceeds (t) 3 2
the abscissa of ' P ' by 3 and the curve passes (a) (a) A-s; B-r; C-p; D-q
through 1,0  . If the line y  x  k where ' k ' (b) A-s; B-r; C-q; D-p
(c) A-s; B-p; C-r; D-q
is a variable constant is a tangent to ' C ' then (d) A-p; B-s; C-p; D-q
the value of 2 k  1 is
CIRCLES 113

45. Let A1, A2 , A3 ,....., An be the vertices of a (b) The area of triangle ABD is 2 2
regular polygon inscribed in a circle of (c) The length of the segment DE is 2
radius1. The product (d) ABD is a triangle of perimeter 2 3

sin sin
2
......sin
 n  1  equals 48. The circle ‘S’ touches the sides AB and AD
n n n of the rectangle ABCD and cuts the side DC
n 1 at single point F and the side BC at a single
(a) n 1 (b) n1
2 2 point E. If AB  32, AD  40 and BE  1
n 1 (a) The angle between pair of tangents drawn
(c) n (d) n
2 2 form the point D to the circle ‘S’ is
46. Let A, B, C, D lie on a line such that  15 
AB  BC  CD  1. The points A and C are   tan 1  
8
also joined by a semicircle with AC as (b) The area of trapezium AFCB is 1180
diameter and P is a variable point on this
(c) The radius of circle is 25
semicircle such that PBD   , 0     . (d) The angle between pair of tangents drawn
Let R is the region bounded by are AP, the form the point D to the circle ‘S’ is
straight line PD and line AD.  15 
(a) The maximum possible area of region R is   2 tan 1  
8
2  3 3 49. A pair of tangents drawn from the point
6 P 1, 8 to the circle
(b) If ‘L’ is the perimeter of region ‘R’, then L
is equal to 3      5  4 cos  x2  y 2 – 6 x – 4 y – 11  0 touch the circle at
(c) The maximum possible area of region R is the points A and B respectively. If the radius
of the circle, which passes through the points
2  3 3
of intersection of the circles
6 2 2
(d) If ‘L’ is the perimeter of region ‘R’, then L
x  y – 2x – 6 y  6  0 and

is equal to 3      5  4 cos  x2  y 2  2 x – 6 y  6  0 , and also intersects


47. Consider two circles S1 and S2 (externally the circumcircle of the PAB orthogonally, is
touching) having centres at points A and B p
equal to , where p, q  , then find the
whose radii are 1 and 2 respectively. A q
tangent to circle S1 from point B touches the minimum value of p  q .
circle S1 at point C, D is chosen on circle S2 50. The sum of the radii of the smallest and
so that AC is parallel to BD and two segments  1
largest circles passing through  a,  and
BC and AD do not intersect. Segment AD  a
intersect the circle S1 at E. The line through B touching the circle
and E intersects the circle S1 at another point 4 x 2  4 y 2  4 x  8 y  31  0 where ' a ' is the
F. least ratio of the sides of the regular ' n ' sided
polygon inscribing and circumscribing the
2 3
(a) The length of segment EF is unit circle is
3
CIRCLES 114

51. Square of radius of circle touching pair of 54. y = f(x) is a parabola of the form y = x2 + ax
lines 7x2 - 18xy + 7y2 = 0 and the circle x2 + + 1, its tangent at the point of intersection of
y2 - 8x - 8y = 0 and contained in it is y-axis parabola also touches the circle x2 + y2
52. If two circles having radii 4 and 9 touch = r2. It is known that no point of parabola is
externally then the radius of the circle below x-axis. The slope of the tangent when
touching both of them externally and also a radius of the circle is maximum, is
common tangent of the circles is 55. y = f(x) is a parabola of the form y = x2 + ax
53. y = f(x) is a parabola of the form y = x2 + ax + + 1, its tangent at the point of intersection of
1, its tangent at the point of intersection of y- y-axis parabola also touches the circle x2 + y2
axis parabola also touches the circle x2 + y2 = = r2. It is known that no point of parabola is
r2. It is known that no point of parabola is below x-axis. The minimum area bounded by
below x-axis. The radius of circle when ‘a’ the tangent and the coordinate axes is
attains its maximum value is
COMPLEX NUMBERS 115

Complex Numbers

Part-A 3
(a)
1. Let A( z1 ) and B  z2  are distinct non-real 2
3
complex numbers in the argand plane such (b)
2
z z
that 1  1  2 . Then value of  ABO is 
z2 z2 (c)
2
_____.
(d) none of these
 
(a) (b) 6. z  3i  4  z  ki  4  k is an ellipse on
6 4
 argand plane for k = _____.
(c) (d) none of these
2 1
(a) 0 (b)
2. The number of different non-singular matrices 2
1 a c  3
(c) (d) 2
of the type A  1 1 b  , where   ei 2
7. A regular heptagon (seven sides) is inscribed
0    in a circle of radius 1 unit. Let A1 , A 2 , ..., A 7
and a, b, c   z : z 4  1  0 are _____. be its vertices, G 1 the centroid of  A1A 4 A 5
(a) 44 (b) 48 and G 2 the centroid of  A 3A 6 A 7 . P is the
(c) 16 (d) 55 centroid of  OG 1G 2 , where O is the centre of
3. The vertices of a triangle in argand plane are the circumscribing circle.
A(2iω + 1), B(1 – 2ω ) and C(1 – 2i) where  POA1 is equal to _____.
2
  2
 e 3
. Then triangle ABC is _____. (a) (b)
7 7
 5 6
(a) acute,  A 
6 (c) (d)
7 7
(b) equilateral 8. Consider a triangle having vertices at the
2  i    11i    5i  
(c) isosceles, C  points A  2 e 4  , B  2 e 12  , C  2 e 12  . Let
3
     
 the in-circle of  ABC touches the sides BC,
(d) obtuse, B 
6 CA and AB at D, E and F respectively. Which
4. Let z be a root of x5 – 1 = 0, with z  1 , then are represented by the complex number
z15  z16  z17  ...  z 50 equals _____. Z d , Z e , Z f in order. If P  z  be any point on
(a) 1 (b) –1 the in circle, then If the altitude through
(c) 0 (d) none of these vertex ' A ' cuts the circum circle of  ABC at
5. Perimeter of the locus represented by Q then the complex number representing Q is
 z i   (a)  2 1  i  (b) 2 1  i 
arg   (where i  1 ) is equal to
 z i  4
 1  i  1
_____. (c) (d) 1  i 
2 2
COMPLEX NUMBERS 116

9. Let M, S, R be three sets of complex numbers (d) z1, z4 , z2 are in A.P


as defined below:
12. Let z1, z2 , z3 be complex numbers associated
  Z  10    
M   Z : Z  2  2 Z  4 ,arg    with the vertices A,B,C of triangle ABC
  Z 2  2 
which is circumscribed by the circle z  1.
S   Z : Z  2  Z  10 
Perpendicular from A is drawn which meets

R  Z : Re  1  i  Z   2  BC at D and circle z  1 at E. If P is the
The number of elements in the set M  S  R image of E about BC and F is the image of E
is about origin ‘O’. Then answer the following
(a) 1 (b) 2 questions:
(c) 3 (d) 4 The complex number associated with E is
10. Let z1, z2 , z3 be complex numbers associated  z2 z3 z z
(a) (b) 1 3
with the vertices A,B,C of triangle ABC which z1 z2
is circumscribed by the circle z  1. z2 z3 z1 z2
(c) (d)
Perpendicular from A is drawn which meets z1 z3
BC at D and circle z  1 at E. If P is the 13. If z1 , z2 , z3 are any three roots of the equation
image of E about BC and F is the image of E 6  z z 
about origin ‘O’. Then answer the following z 6   z  1 , then arg  1 3  can be equal
 z2  z3 
z1  z z3  z2
questions: If  0 and to _____.
z1  z z3  z2 (a) 0 (b) 
z2  z z2  z  
 0 then 'z' is complex (c) (d) 
z1  z3 z1  z3 4 4
number associated with 14. Let a, b, c be distinct complex numbers with
(a) E (b) O a  b  c  1 and z1, z2 be the roots of the
(c) P (d) F
equation a z 2  b z  c  0 with z1  1. Let P
11. Let z1, z2 , z3 be complex numbers associated
with the vertices A,B,C of triangle ABC which and Q represent the complex numbers z1 and

is circumscribed by the circle z  1. z2 in the argand plane with POQ   ,


Perpendicular from A is drawn which meets 00    1800 (where O being the origin) then
BC at D and circle z  1 at E. If P is the 2
(a) b 2  a c ;  
image of E about BC and F is the image of E 3
about origin ‘O’. Then answer the following 2
(b)   ; PQ  3
questions: If the tangents drawn at z1, z2 to the 3

circum-circle intersect at z4 then (c) PQ  2 3; b2  ac



(a) z1, z2 , z4 are in H.P (d)   ; b2  a c
3
(b) z1, z2 , z4 are in A.P
(c) z1, z4 , z2 are in H.P
COMPLEX NUMBERS 117

15. Z1, Z2 are two complex numbers satisfying 22. z1, z2 , z3 are vertices of a triangle ABC having
2 2
i z1 z2  z2 z1  z1  iz 2 . Then which area  satisfying
of the following is/are correct.  z3  z1   1  i 
3  z2  z1  and
2
z  3 z2  z3  k  then value of k is
(a) Re  1   0
 z2  23. a, b, c are complex numbers and all roots of
z  z3 + az2 + bz + c = 0 satisfy z  1 . Now
(b) I m  1   0
 z2  match the following.
Column-I Column-II
(c) z1 z2  z1 z2  0
(A) a  b is equal to (p) 2
(d) z1 z2  1 or z1  z2 (B) Number of distinct real (q) 0
16. Let z1 , z2 , z3 be three complex number such roots of the equation
3 2
z  a z  b z c 0
that z1  z2  z3  1 and
(C) If z 3  a z 2  b z  c  0 , (r) 1
z12 z2 z2
 2  3  1  0 , then sum of possible then z is equal to
z2 z3 z1 z3 z1 z2
(D) c is equal to (s) 3
values of z1  z2  z3 is _____.
(t) 4
17. If the vertices of a triangle ABC,
(a) A-q; B-t; C-r; D-r
A  z1  ,B  z2  and C  z3  lie on the circle (b) A-q; B-r; C-q; D-r
z  3  16 , such that z1  z2  z3  9 , then the (c) A-q; B-p; C-r; D-r
(d) A-q; B-r; C-r; D-r
z1 tan A  z2 tan B  z3 tan C 24. Match the following:
value of is equal
tan A tan B tan C Consider the two circles in the complex plane
to _____. z i
C1 : 2
18. If ω is a non-real complex root of z28 = 1 and z 1
such that   1 is maximum and C2 : z z   3  4i  z   3  4i  z  9  0
1 1 Let 1 , 1  be the centre of C1 and  2 ,  2 
x   , then 8x4 + 4x3 – 8x2 – 3x + 4 is
2  be that of C2
equal to _____. Column-I Column-II
(A) 21  1 equals (p) 11
3  2i 1 i
(B)  2  2  2 equals (q) 3
19. If z  2 3  2i 1  i and z  z  k z
(C) The radius of circle C1 (r) 4
1 i i 3
equals
then 3k is _____. (D) The radius of circle C2 2 2
(s)
20. Let z1 , z2 , z3 be complex numbers such that equals 3
(t) 2
z1  z2  z3  0 and z1  z2  z3 then
(a) A → q; B → p; C → s; D → r;
z12  z22  z32 is _____. (b) A → q; B → r; C → s; D → p;
(c) A → q; B → s; C → p; D → r;
21. If  is a non-real complex cube root of unity
(d) A → p; B → q; C → s; D → r;
and 1    2 2  3 2  ... up to 2013 terms is
a  b 2 , then the value of a  b is _____.
COMPLEX NUMBERS 118

25. Match the following: Part-B


Let A Z1  , B  Z2  , C  Z3  and O be the
27. Locus of z given by z  6i  6 is rolled over
origin.
real axis. If a point P on it has following
Column-I Column-
positions.
II
At t = 0 : Pinitial(12i)
(A) If Z1  iZ 2  1  i  Z 3 then (p) Right
ABC is angled 
At t = T : Pfinal   3  i (6  3 3) 
triangle Then angle of rotation of point P in span of
(B) If Z1  Z 2  3Z1Z 2  0 then (q) Right
2 2 time T, w.r.t. its centre is _____.
OAB is angled  
(a) (b)
isosceles 12 6
triangle  
(c) (d)
(C) If (r) 4 2
 
2  Z 2  Z3   1  i 3  Z1  Z 3  Equilatera 28. The least value of
l triangle 2 2
z  3  4i  z  2  7i  z  5  2i
2
occurs
then ABC is
(D) If (s) when
(a) 1 + 3i (b) 3 + 3i
 
2  Z1  Z 2   1  i 3  Z 3  Z 2   0 Isosceles
(c) 3 + 4i (d) none of
triangle
then ABC is
these
(t) Obtuse
29. Let A  z1  ,B  z2  ,C  z3  are complex
angled
triangle numbers satisfying z  3i  1 and
(a) A → p, q, s; B → s; C → r; D → s, t;
(b) A → p, s; B → s; C → r, s; D → s, t; 3 z1  3i  2 z2  2 z3 . z1  z2 
(c) A → p, q, s; B → s; C → r, s; D → s, t; 1
(a)
(d) A → p, s; B → s; C → r, s; D → s, t; 2
26. Statement 1: If ' Z ' is a complex number then
3
the equation Z 2  1  Z 2  1 has infinitely (b)
2
many solutions. 3
(c)
Statement 2: If Z1, Z 2 are complex numbers 4
then Z1  Z2  Z1  Z2 iff Z1, Z 2 are positive (d) none of these

real numbers.
(a) Statement-1 is True, Statement-2 is True;
Statement-2 is a correct Explanation for
Statement-1
(b) Statement-1 is True, Statement-2 is True;
Statement-2 is NOT a correct explanation
for Statement-1
(c) Statement-1 is True, Statement-2 is False
(d) Statement-1 is False, Statement-2 is True
COMPLEX NUMBERS 119

30. Let A  z1  ,B  z2  ,C  z3  are complex numbers 33. Consider a  ABC with A, B & C represented

satisfying by the complex numbers z1 , z2 & z3


z  3i  1 and
respectively. The in-circle of  ABC with
3 z1  3i  2 z2  2 z3 .
centre O (the origin) touches the sides
Area of quadrilateral ABCD, where D is 3i BC, CA & AB at the points z4 , z5 & z6
is _____. respectively. AO cuts the circum circle of
1  ABC at z .
(a)
2 2 z  z2 z
 equals
7 z z  z3
(b)
4 (a) 1 (b) 2
7 1
(c) (c) (d) 4
6 2
(d) none of these 34. Consider a  ABC with A, B & C represented
31. Let A  z1  ,B  z2  ,C  z3  are complex numbers by the complex numbers z1 , z2 & z3
satisfying z  3i  1 and respectively. The in-circle of  ABC with
centre O (the origin) touches the sides
3 z1  3i  2 z2  2 z3 .
BC, CA & AB at the points z4 , z5 & z6
z2  3i
The value of is equal to _____. respectively. AO cuts the circum circle of
z3  3i  ABC at z .
1  7i 1  7i  z  z2   z3  z5  equals
(a) (b)  z   z  z 
2 2 3 3   3 4 
1  63i 1  19i (a) 1 (b) -1
(c) (d)
8 2 5 (c) 2 (d) -2
32. Consider a  ABC with A, B and C 35. If from a point P, representing the complex
represented by the complex numbers number z1 , on the curve z  2 , two tangents
z1 , z2 & z3 respectively. The in-circle of are drawn to the curve z  1 , meeting the
 ABC with centre O (the origin) touches the curve at points Q  z2  and R  z3  , then
sides BC, CA and AB at the points z4 , z5 & z6 _____.
respectively. AO cuts the circum circle of z1  z2  z3
(a) the complex number will lie
 ABC at Z. The point z3 is 3
1 1 1 on the curve z  1
(a) 
z4 z5
(b) z  z 
2 4 5  4 1 1  4 1 1 
(b)         9
(c) 2 (d)
2  z1 z2 z3  z1 z2 z3 
 1 1
z4  z5  z  2
   (c) arg  2  
 z4 z5   z3  3
(d) orthocenter and circumcenter of PQR
will coincide
COMPLEX NUMBERS 120

36. Let f(x) = x13 + 2x12 + 3x11 + ... + 13x + 14 and 40. Statement 1: If ' z ' is a complex number
2 2 z
  cos
15
 i sin
15
.  z  1 then  1  arg z
z
If N = f(α) f(α2)… f(α14), then _____.
Statement 2: In a unit radius circle chord
(a) number of divisors of N is 196
(b) number of divisors of N is 256  AP   arc  AP 
(c) number of divisors of N which are perfect (a) Statement-1 is True, Statement-2 is True;
squares is 49 Statement-2 is a correct Explanation for
(d) number of divisors of N which are perfect Statement-1
cubes is 16 (b) Statement-1 is True, Statement-2 is True;
37. Let z1, z2 , ... zn be equimodular non-zero Statement-2 is NOT a correct explanation
for Statement-1
complex numbers such that
(c) Statement-1 is True, Statement-2 is False
 n n zj 
z1  z2  ....  zn  0 . Then Re    is (d) Statement-1 is False, Statement-2 is True
 j  1 k  1 zk 
equal to _____.
38. Let  be a non-real complex cube root of
unity and let a, b, c  I. If a, b, c are not all
equal, then the minimum value of
a  b  c 2  a  b 2  c is _____.
39. Let 1, 1 ,  2 , 3 ,......10 be the eleven 11th
10
roots of unity. Let    r  r  11r  . The
r 1

  11
value of equals
11
CONIC SECTIONS 121

Conic Sections

Part-A 5. The locus of the centre of the rectangle


1. The radius of a circle passing through the formed by the tangents and the normals at the
focus of parabola x 2  4 y and touching it at ends of the focal chords of parabola y 2  4ax
is _____.
the point  6, 9  is _____.
(a) 2ax  y 2 – 2a 2
(a) 5 (b) 5 10
(b) y 2  2a  x – a 
(c) 5 5 (d) 10
(c) y 2  4a  x – a 
2. The eccentricity of the ellipse 3 x 2  4 y 2  12
is changing at a rate of 0.1 per second. The (d) y 2  4a  x  a 
time at which it coincides with the auxiliary 6. A rectangular hyperbola has one of its
circle is _____. asymptotes as x – y  2  0 . The tangent at
(a) 2 seconds (b) 3 seconds the point P  0, 1 is x  0 . The length of the
(c) 5 seconds (d) 6 seconds
latus rectum is _____.
3. A series of ellipses are described with given
focus and corresponding directrix. The locus (a) 2
of the extremities of their minor axes is _____. (b) 3
(a) pair of straight line (b) circle (c) 2
(c) parabola (d) ellipse (d) none of these
4. Let 5 x  3 y  8 2 be the normal at the point 7. A  0, 2  , B and C are three points on the
 5 3  x2 y 2 parabola y2  x  4 and such that
P ,  to the ellipse   1, a  b .
 2 2 a 2 b2 
 CBA  . The range of the ordinate of C
If M, M  are the feet of the perpendiculars 2
from the foci S, S respectively on the tangent is _____.
at P, then the point of intersection of SM and (a)  , 0   4,  
SM is _____. (b)  ,0    4,  
5 
(a)  , 0  (c)  0, 4
2 
 5 (d)  ,0    4,  
(b)  0, 
 2 8. The locus of a point P moving in the XY
plane, such that the ratio of its distance from
 41 3 
(c)  ,  S 1, 2  and line x  y  3 is equal to k , is
 10 2 2 2 
_____.
 3 41 
(d)  ,  (a) a hyperbola if k  1
 2 2 10 2 
(b) a pair of straight line for k  0
(c) an ellipse if k  1
(d) a parabola if k  1
CONIC SECTIONS 122

9. The equation of the transverse axis of a (c)  4, 4  and  4,  4 


hyperbola which passes through  2, 4 and
whose asymptotes are given by the equation
  
(d) 4 2, 4 2 and 4 2,  4 2 
14. A conic passes through the point  2, 4  and is
 x  y  x  y  1  0 is _____.
such that the segment of any of its tangents at
(a) x  y  0
any point contained between the coordinate
1
(b) y   axes is bisected at the point of tangency.
2
The equation of its directrices are _____.
1
(c) x   (a) x  y  8
2
(b) x  y  4
(d) none of these
10. The maximum number of common normals of (c) x  y  4 2
2 2
y  4ax and x  4by is equal to _____. (d) none of these
(a) 3 (b) 4 15. Let S, S be the foci of the ellipse
(c) 5 (d) 6 x2 y 2
  1 whose eccentricity is e. P is a
11. If the variable line y  kx  2h is a tangent to a2 b2
the ellipse 2 x 2  3 y 2  6 , then the locus of variable point on the ellipse. Consider the
locus of the incentre of the  PSS .
P  h, k  is a conic C, whose eccentricity
The locus of the incentre is _____.
equals _____.
(a) an ellipse (b) a hyperbola
5 7 (c) a parabola (d) a circle
(a) (b)
2 3 
16. Let S, S be the foci of the ellipse
7 x2 y 2
(c) (d) 2   1 whose eccentricity is e. P is a
3 a2 b2
12. A conic passes through the point  2, 4  and is variable point on the ellipse. Consider the
such that the segment of any of its tangents at locus of the incentre of the  PSS .
any point contained between the coordinate The eccentricity of the locus of P is _____.
axes is bisected at the point of tangency. 2e
(a)
Its eccentricity is _____. 1 e
(a) 2 (b) 2 2e
(b)
3 1 e
(c) 3 (d)
2 (c) 1
13. A conic passes through the point  2, 4  and is (d) none of these
such that the segment of any of its tangents at
any point contained between the coordinate
axes is bisected at the point of tangency.
The foci of the conic are _____.
 
(a) 2 2, 0 and 2 2, 0  
(b)  2  
2, 2 2 and 2 2,  2 2 
CONIC SECTIONS 123

x2 y 2 20. A straight line L is drawn through the origin


17. Let S, S be the foci of the ellipse  1 and parallel to the tangent to the curve
a 2 b2
whose eccentricity is e. P is a variable point on f  x, y   0 at an arbitrary point M on the
the ellipse. Consider the locus of the incentre curve. P is the point of intersection of the line
of the  PSS . L and the straight line parallel to the Y–axis
The maximum area of the rectangle inscribed and passing through the point M. If
in the locus is _____.  a  a2  x2 
2abe 2 f  x, y   y  a 2  x 2  a ln  
(a)  x 
1 e  
2abe then the locus of P is a _____.
(b) (a) Straight line
1 e
abe (b) Parabola
(c) (c) Circle
1 e
(d) none of these (d) Central conic
18. A straight line L is drawn through the origin 21. From a point P, common tangents are drawn
and parallel to the tangent to the curve to the ellipse x 2  4 y 2  8 and parabola
f  x, y   0 at an arbitrary point M on the y 2  4 x . The equation of the common
curve. P is the point of intersection of the line tangents to the ellipse and the parabola are
L and the straight line parallel to the Y–axis _____.
and passing through the point M. If (a) x  3   2 y
f  x, y   y  log b x then the locus of P is a (b) x  4   2 y

_____. (c) x  4   2 y
(a) Straight line (d) x  3   2 y
(b) Parabola 22. A circle externally touches the two circles
(c) Circle x 2  y 2  1 and x 2  y 2  4 x . The locus of
(d) Central conic the centre of the circle is a conic C and e is its
19. A straight line L is drawn through the origin eccentricity. Then, _____.
and parallel to the tangent to the curve (a) C is an ellipse
f  x, y   0 at an arbitrary point M on the (b) C is a hyperbola
curve. P is the point of intersection of the line (c) e  2
L and the straight line parallel to the Y–axis (d) e  2
and passing through the point M. .If 23. A hyperbola has a focus at origin, its
f  x, y   y 2  4 a x then the locus of P is a eccentricity is 2 and the corresponding
_____. directrix is x  y  1  0 . The equation of its
(a) Straight line asymptotes is/are _____.
(b) Parabola (a) x  1  0 (b) x  1  0
(c) Circle (c) y  1  0 (d) y  1  0
(d) Central conic
CONIC SECTIONS 124

24. If the tangents at the points A( x1 , y1 ) and 1


(d) Directrices are x    
B  x2 , y2  to the parabola y 2  4 x intersect at 2
28. The number of points on the hyperbola
C  x3 , y3  , and l1 , l2 , l3 are the lengths of the
x2 y 2
perpendiculars on any of the tangents of the  1 from where mutually
a 2 b2
given parabola from the points A, B, C
perpendicular tangents can be drawn to circle
respectively, then _____.
x 2  y 2  a 2 is _____.
(a) x1 , x3 , x2 are in GP
29. If a tangent of slope 2 of the ellipse
(b) y1 , y3 , y2 are in AP
x2 y 2
(c) l1 , l3 , l2 are in HP   1 is normal to the circle
a2 b2
(d) l1 , l3 , l2 are in GP x 2  y 2  4 x  1  0 , then maximum value of
25. Let A, B and C be three distinct points on ab is _____.
2
y  8 x such that the normals at these points 30. The normal chord at a point t on the parabola
are concurrent at P. The slope of AB is 2 and y 2  4ax subtends a right angle at its vertex.
4 Find the value of t 2 .
the abscissa of the centroid of  ABC is .
3 31. A parabola with directrix x y20
Which of the following is (are) correct?
touches a line 2 x  y – 5  0 at  2, 1 . If m is
(a) area of ABC is 8 square units
(b) coordinates of P are (6,0) the length of the latus rectum of the parabola,
(c) angle between normals are 45°, 45°, 90° m
then the value of is _____.
(d) angle between normals are 30°, 30°, 60° 2
26. Let L be the point  t , 2  and M be a point on 32. The number of circles of the form
x 2  y 2  r 2 that can be drawn, such that they
the y axis such that LM has slope  t . Then
neither touch nor intersect the curve xy  8 ,
the locus of the midpoint of LM. as t varies
over all real values, is a parabola, whose. where r  is _____.
(a) Vertex is  0, 2  33. If two distinct chords of a parabola y 2  4ax

(b) Latus – rectum is of length 2 passing through  a, 2a  are bisected by the


 17  line x  y  1 , and 4a is a natural number,
(c) Focus is  0, 
 8  then the maximum length of the latus–rectum
(d) Directrix is 8 y  15  0 is _____.
34. Area of trapezium whose vertices lie on the
27. Let A   1, 0  and B   2, 0  be two points
parabola y 2  4 x and its diagonals pass
on the x – axis. A point M is moving in the xy
25
– plane in such a way that  MBA  2  MAB through 1, 0  and having length unit
4
. Then the point M moves along a conic whose
75
(a) Eccentricity is 2 each, is . Then  is _____.

1
(b) Eccentricity is
2
(c) Latus – rectum is of length 6
CONIC SECTIONS 125

35. Normals drawn through the point P  2, 3 (a) A → q, r, s, t; B → p; C → p, q, r; D → p


(b) A → q, r, s, t; B → p; C → p, r; D → p
intersects the curve xy  4 at
(c) A → q, t; B → p; C → p, q, r; D → p
A  x1 , y1  , B  x2 , y2  , C  x3 , y3  , D  x4 , y4  . (d) A → q, r, t; B → p; C → p, q, r; D → p
The value of 37. Match the following:
1 1 1 1  Column-I Column- II
 x1  x2  x3  x4       is (a) The normal chord at a point (p) 4
 y1 y2 y3 y4 
on the
_____.
36. Match the following. parabola y 2  4 x subtends a
Column-I Column-II right angle at the vertex,
(A) If three normals can be (p) 0 (b) The area of the triangle (q) 2
drawn to the curve y 2  x inscribed in the curve y 2  4 x ,
from the point  c, 0  , then c the parameter of coordinates
can be _____. whose vertices are 1, 2 and 4 is
(B) If the sides and the angles (q) 1 (c) The number of distance (r) 3
of a plane triangle vary in  11 1 
normal possible form  , 
such a way that its  4 4
circumradius remains to the parabola y 2  4 x is
constant, then
(d) The normal at  a, 2a  on (s) 6
da db dc
  
cos A cos B cos C y 2  4ax meets the curve
_____. again at  at 2

, 2at , then the
(C) If the point  a, a  lies (r) 2
value of t  1 is
between the lines x  y  6 ,
(a) (a) (p); (b)  (s); (c)  (q); (d)  (p)
then  a  is _____, where [.] (b) (a) (q); (b)  (r); (c)  (q); (d)  (p)
denotes the greatest integer (c) (a) (q); (b)  (s); (c)  (q); (d)  (r)
function and |.| denotes the (d) (a) (q); (b)  (s); (c)  (q); (d)  (p)
absolute value function)
(D) The point  p  1 ,  p  (s) 3
lies inside the region
bounded by the circles
x 2  y 2  2 x  15  0 and
x2  y2  2 x  7  0 . The
number of possible values of
p is _____, where [.] denotes
the greatest integer function
and |.| denotes the absolute
value function).
(t) 4
CONIC SECTIONS 126

38. Match the following: 39. Match the following:


Column-I Column-II Column-I Column
(a) The parabola (p) 1 -II
2
y  ax  bx  c has vertex (a) The straight line joining the (p) – 4
 p, p  any y - intercept ‘ points  0, 3 and  5,  2  is a

 p ’, where p  0 . The value C


tangent to the curve y 
of ‘ b ’ is not less than x 1
then C 
(b) Two of the altitudes of a (q) 2
scalene triangle ABC have (b) For each parabola (q) 2
2
length 4 and 12. If the length y  x  px  q ( p , q are
of third altitude is ‘ p ’ it is parameters meeting) coordinate
also an integer, then ‘ p ’ can axes at 3 distinct points, if circles
be are drawn through these points,
and the family of circle passes
(c) The number of points in (r) 3
the complex plane satisfying through the fixed point  h, k 
z  4  8i  10 and then h  4k 
(c) If OB is the one of the (r) 0
z  3  5i  z  5  11i  4 5
rhombus of area 2 3 sq. units
is not more than
where O   0, 0  and B   2, 0 
(d) The straight line (s) 4
3 x  4 y  12  0 intersects the and the other vertices are

x2 y 2  x1, y1  ,  x2 , y2  in the first


ellipse   1 at two
16 9 quadrant then x1  x2  y1  y2 
points A and B. There is a (d) The distance between the (s) 4
point p on the ellipse such lines
2
that area of  PAB is 3. The  x  7y  4 2  x  7 y   42  0
number of such points ‘ p ’ is .
not greater than (a) (a)  (q); (b)  (s); (c)  (s); (d)  (q)
(t) 5 (b) (a)  (s); (b)  (p); (c)  (s); (d)  (q)
(c) (a)  (s); (b)  (s); (c)  (s); (d)  (q)
(a) a  p, q, r, s; b  s, t; c  q, r, s, t; d  (d) (a)  (r); (b)  (s); (c)  (s); (d)  (q)
q, r, s, t
(b) a  p, q, s; b  s, t; c  q, r, s, t; d  q,
r, s, t
(c) a  p, q, r, s; b  s, t; c  q, t; d  q, r,
s, t
(d) a  p, q, r, s; b  s; c  q, r, s, t; d  q,
r, s, t
CONIC SECTIONS 127

40. Match the following: 42. Statement I : The locus of a point P, with
For the ellipse respect to which, the chord of contact of the
2 42 2 curve y 2  4ax subtends 90° at origin, is
 3x – 6   3 y – 9 
 5 x  12 y  6 
169 x  4a .
Column-I Column-II because
(a) Length major axis 24 Statement II : If the normal drawn at the
(p)
5 point t1 on the curve y 2  4ax cuts the
(b) Length minor axis 16
(q) parabola again at t2 if t1 is real, when
5
(c) Length of Latus t2  2 2 .
16
(r)
Rectum 3 (a) Statement I is True; Statement II is True;
(d) Distance between 72 Statement II is a correct explanation
(s) for Statement I.
directrices 5
48 (a) Statement I is True; Statement II is True;
(t) Statement II is not a correct explanation
5
for Statement I.
(a) a  r; b  q; c  r; d  s
(c) Statement I is True; Statement II is False.
(b) a  t; b  q; c  r; d  s
(d) Statement I is False; Statement II is True.
(c) a  t; b  s; c  r; d  s
43. Statement –I : If the circles
(d) a  t; b  q; c  q; d  s
2
x 2  y 2  2 x  4 y  4  0 and
41. From a point P on the parabola y  4ax ,
x 2  y 2  4 x  2 y  c  0 intersect such that the
normals at the points Q and R, are drawn to
the same parabola. Now, a circle is drawn to common chord is longest, when c  4 .
Statement – II : If two circles intersect, then
circumscribe the  PQR. If the circumcircle of
the common chord so obtained, is longest
 PQR cuts the parabola at another point M,
when it is a diameter of the smaller circle.
then,
(a) Statement-1 is True, Statement-2 is True;
Statement I : The common chords QR and
Statement-2 is a correct Explanation for
PM intersect each other at the directrix.
Statement-1
Statement II : The chord of contact of the
(b) Statement-1 is True, Statement-2 is
parabola with respect to any point lying on
True; Statement-2 is NOT a correct
the directrix passes through the focus of the
explanation for Statement-1
parabola.
(c) Statement-1 is True, Statement-2 is False
(a) Statement I is True; Statement II is True;
(d) Statement-1 is False, Statement-2 is True
Statement II is a correct explanation
for Statement I.
(a) Statement I is True; Statement II is True;
Statement II is not a correct explanation
for Statement I.
(c) Statement I is True; Statement II is False.
(d) Statement I is False; Statement II is True.
CONIC SECTIONS 128

44. Statement 1: The equation 48. Two congruent circles of the largest possible
2 2 radii having the following properties.
x  2  x y  y  2 x  2 y  4  0 represents
an ellipse (i) They intersect each other orthogonally.
if    1, 1 . (ii) They touch both the curves 4  y  2   x 2
Statement 2: The general equation of second and 4  2 – y   x2 in the region
degree represents an ellipse if   0 ,
x   2 2, 2 2  .
2
h  ab  0
Then, the radii of these circles are each equal
(a) Statement-1 is True, Statement-2 is True;
to _____.
Statement-2 is a correct Explanation for
Statement-1 (a) 2 (b) 3
(b) Statement-1 is True, Statement-2 is True; 1 3
(c) (d)
Statement-2 is NOT a correct explanation 3 2
for Statement-1 49. AB is a chord of a parabola with vertex A.
(c) Statement-1 is True, Statement-2 is False The axis of the parabola is inclined at angle of
(d) Statement-1 is False, Statement-2 is True 30o with the X axis. BC is drawn
perpendicular to AB, meeting the axis of the
Part-B
parabola at C. If the length of the
45. Let P be a variable point on the ellipse
perpendicular drawn from the focus to the
x2 y 2 directrix is 8 units, then the projection of BC
  1 with foci F1 and F2 . If A is the
10 1 on the axis of parabola is _____ units long.
area of the triangle PF1F2 , then the maximum (a) 8 (b) 16
value of A is _____. (c) 24 (d) 32
(a) 3 (b) 4 50. The locus of the focus of an ellipse with
(c) 5 (d) 6 length of major axis 2a and minor axis 2b ,
46. If the rectangular hyperbola  x –1 y – 2   4 where a  b , which touches the X axis at
origin, is _____.
cuts a circle x 2  y 2 – 7 x – 9 y  c  0 at the 2 2

points  3, 4 ,  5, 3 ,  2, 6  and the point 


(a) xy 2 – b 2 x  y 2
– b2  
 4 a2 – b2 y 2 
2 2
P  u, v  , then the value of 9  u  v  is equal to (b)  xy 2  b 2 x    y 2 – b 2  y 2  4  a 2 – b 2  y 4
2 2
_____.
(a) –8 (b) –9

(c) xy 2  b 2 x  –y 2

– b2 
 4 a2 – b2 y 2
2 2 2
(c) 8 (d) 9 (d)  xy 2
 b x   y
2 2
b   4a 2
–b y
2 2

47. If three parabolas touch all the three lines


x2 y 2
x  0 , y  0 and x  y  2 , then the 51. Consider an ellipse
 1  0 .E
16 4
maximum area of the triangle formed by Suppose that C is any circle concentric with
joining their foci is _____. E. Let A be a point on E and let B be a point
(a) 3 (b) 6 on C, such that AB is tangential to both E and
C.
3 3 3 3 The maximum length of AB is _____.
(c) (d)
4 2 (a) 2 (b) 4
(c) 2 2 (d) 4 2
CONIC SECTIONS 129

x2 y 2 56. Consider a parabola (P) having focus at


52. Consider an ellipse E  1  0 .
16 4 F 1, 2  and touching both the coordinate
Suppose that C is any circle concentric with E. axes. From point R on axis of parabola P,
Let A be a point on E and let B be a point on three real and distinct normals are drawn to
C, such that AB is tangential to both E and C. the parabola, then coordinate of R will be
The slope of the tangent AB, when the length _____.
of AB is maximum is _____. 12
1 (a)  2  3,   /  
(a) 2 (b) 5
2 12
(b)  2  3,   /  
3 2 5
(c) (d)
2 3 12
2
(c)  2  3,   / 0   
x y2 5
53. Consider an ellipse E   1  0 .
16 4 (d)  2  3,   /  
Suppose that C is any circle concentric with E. 57. If a parabola touches the lines y  x and
Let A be a point on E and let B be a point on
C, such that AB is tangential to both E and C. y  x at P  3, 3 and Q  2,  2 
The radius of circle C, when length of AB is respectively, then _____.
maximum, is _____.  30 6 
(a) focus is  ,   .
(a) 4 2 (b) 3 2  13 13 
(c) 2 2 (d) 4 (b) equation of directrix is 5 x  y  0 .
54. An ellipse whose major axis is parallel to X – (c) equation of line through origin and focus
axis such that the segments of the focal chords is x  5 y  0 .
are 1 and 3 units. The lines a x  b y  c  0 (d) equation of line through origin and
are the chords of the ellipse such that a , b, c parallel to axis is x  5 y  0 .
are in A.P. and bisected by the point at which 58. A hyperbola centred at C has one focus at
they intersect. The equation of its auxiliary P  6, 8 . If its directrices are 3 x  4 y  10  0
circle is x 2  y 2  2  x  2  y  2  1  0 and 3 x  4 y – 10  0 , then _____.
then _____. (a) CP  10
Equation of the auxiliary circle is (b) eccentricity = 5
(a) x 2  y 2  2 x  4 y  1  0 (c) CP  8
(b) x 2  y 2  2 x  2 y  3  0 5
(d) eccentricity =
(c) x 2  y 2  2 x  4 y  1  0 2
(d) x 2  y 2  4 x  2 y  3  0
55. Consider a parabola (P) having focus at
F 1, 2  and touching both the coordinate axes.
Equation of directrix of parabola p is _____.
(a) y  x  0 (b) y  2 x  0
(c) y  3 x  0 (d) y  4 x  0
CONIC SECTIONS 130

x2 y2 63. Let C1 : x 2  y 2  5 and


59. Two ellipses  1 and
cos2  sin 2  C 2 : x 2  y 2  8 y  3  0 be the equations of
x2 y2   a hyperbola and a circle respectively. The
2
 2
 1  0     intersect at
sin  cos   4 curves C1 and C2 touch each other at  3, 2 
four points P,Q, R, S then which of the
following statement(s) is /are true ? 
. P 0, 4  13  is a point on the curve C2 .
(a) PQRS is a square with length of the side Let a line through P meet C1 at m number of
sin 2
points and C2 at n number of points. If
(b) PQRS lie on a circle whose centre is origin
sin 2  m  n   3 , then the number of such straight
and with radius
2 lines is _____.
(c) eccentricity of the two given ellipses are 64. From the origin, tangents OA and OB are
2 2
same drawn to the curve  x – 2   y – 2   1 . If
(d) there are two points on the line PQ, where P and Q are respectively
x2 y2 the midpoints of OA and OB, touches the
  1 whose reflection in
sin 2  cos 2  2
curve  y  3  4a  x  4  and the length of
y  x lie on the same ellipse
l
60. If p is the length of the perpendicular from a latus rectum of the parabola is l , then is
7
focus upon the tangent at any point P on the
_____.
x2 y 2 65. Coordinates of the vertices B and C are
ellipse 2  2  1 and r is the distance of P
a b
 2, 0  and  8, 0  respectively. The vertex A
2a b 2
from the focus, then  2  _____. B C
r p is varying in such a way that 4 tan tan  1
2 2
61. If the length of the latus rectum of a standard
. If the locus of A is an ellipse then the length
hyperbola of eccentricity 2 is equal to the limit
of its semi major axis is _____.
3 5 7
of the series 2  2  2  ... and 66. A parabola is drawn through two given points
1 1  2 1  22  32
2
A 1, 0  and B  1, 0  such that its directrix
r is the radius of the director circle of its
conjugate hyperbola, then r 2  _____. always touches the circle x 2  y 2  4. If the
62. Let P, Q be points on the ellipse maximum possible length of semi latus–
96 rectum is k then  k  is (where [.] denotes
16 x 2  25 y 2  400 , so that PQ  and P, Q
25 greatest integer function)
lie above the major axis. The circle drawn
with PQ as diameter touches the major axis at
the positive focus. If m is the slope of PQ,
1
then find the value of .
m
CONIC SECTIONS 131

67. Match the following:


Column-I Column-II
(a) No of points of intersection (p) 1
of curves y  ln x and
2
 x  1  y2  4  0 .
n (q) 2
(b) Sn   r ! n  6  then
r 1

S 
Sn  7  n  is (where [ x ]
7
denotes integer less than or
equal to x )
(c) Equation of tangent z0 to (r) 3
the circle z r is
 z 
Re     , then  is
 z0 
(d) Normal drawn at any point (s) 5
x2 y 2
of an ellipse   1 , is
25 9
tangent to the circle
2 2 2
x y r then maximum
value of r is
(t) 5

(a) a  s; b  s, t; c  p; d  q
(b) a  r; b  s, t; c  p; d  q
(c) a  r; b  s; c  p; d  q
(d) a  r; b  s, t; c  s; d  q
ANSWER KEYS 132

MATHEMATICS
UNIT - I
INDEFINITE AND DEFINITE INTEGRATION
Part-A
1. (c) 2. (d) 3. (a) 4. (b) 5. (c) 6. (c) 7. (b) 8. (a) 9. (a) 10. (a)
11. (b) 12. (c) 13. (a) 14. (d) 15. (c) 16. (d) 17. (c) 18. (c) 19. (b) 20. (a)
21. (c) 22. (c) 23. (c) 24. (a) 25. (c) 26. (a) 27. (d) 28. (c) 29. (b) 30. (b, c)
31. (a,b) 32. (b, d) 33. (a, b) 34. (a, b, c, d) 35. (a, c) 36. (a, b) 37. (a, c) 38. (a, b, d) 39. (c)
40. (bc) 41. (d) 42. (a) 43. (c) 44. (a) 45. (d) 46. (002) 47. (001) 48. (006)
Part-B
49. (b) 50. (a) 51. (c) 52. (d) 53. (a) 54. (d) 55. (b) 56. (b) 57. (d) 58. (a)
59. (b) 60. (d) 61. (b) 62. (a) 63. (b, c, d) 64. (c) 65. (d) 66. (b) 67. (a) 68. (b)
69. (d) 70. (a) 71. (005) 72. (001) 73. (009) 74. (002) 75. (001) 76. (008) 77. (002) 78. (008)
79. (002)

UNIT – II
AREA UNDER THE CURVES, DIFFERENTIAL EQUATION
Part-A
1. (b) 2. (b) 3. (d) 4. (a) 5. (b) 6. (c) 7. (a) 8. (c) 9. (c) 10. (a)
11. (b) 12. (b) 13. (d) 14. (b) 15. (a) 16. (a) 17. (a) 18. (c) 19. (a, b, d) 20. (a, b)
21. (a) 22. (0.00) 23. (8.00) 24. (4.00) 25. (d) 26. (c)
Part-B
27. (c) 28. (a) 29. (d) 30. (b) 31. (a) 32. (a) 33. (a) 34. (2.00) 35. (2.00) 36. (8.00)
37. (1.00) 38. (4.00) 39. (d) 40. (a)
PROBABILITY
Part-A
1. (a) 2. (a) 3. (c) 4. (a) 5. (c) 6. (d) 7. (a) 8. (d) 9. (a) 10. (c)
11. (a, b) 12. (a, c) 13. (a, b) 14. (b, c) 15. (a, c) 16. (a, c) 17. (d) 18. (b) 19. (a, b, d) 20. (a)
21. (b) 22. (a) 23. (4.00) 24. (2.00) 25. (9.00)
Part-B
26. (a) 27. (a) 28. (c) 29. (a, d) 30. (d) 31. (b) 32. (a) 33. (a) 34. (c) 35. (c)
36. (d) 37. (a) 38. (b) 39. (5.00) 40. (5.00) 41. (7.00) 42. (9.00) 43. (9.00)

UNIT – III
VECTORS & 3D COORDINATE GEOMETRY
Part-A
1. (a) 2. (d) 3. (b) 4. (d) 5. (a) 6. (b) 7. (a) 8. (b) 9. (c) 10. (c)
11. (b) 12. (b) 13. (b) 14. (a) 15. (a) 16. (d) 17. (c) 18. (d) 19. (a) 20. (d)
21. (c) 22. (c) 23. (d) 24. (d) 25. (b) 26. (a, c) 27. (c, d) 28. (a, c) 29. (a, c) 30. (a, b, c)
31. (a, b) 32. (b, c) 33. (b, c) 34. (a, c) 35. (1.00) 36. (7.00) 37. (5.00) 38. (8.00) 39. (3.26) 40. (5.00)
41. (6.00) 42. (3.00) 43. (9.00) 44. (4.00) 45. (3.00) 46. (d) 47. (a) 48. (d) 49. (a) 50. (b)
Part-B
51. (b) 52. (a) 53. (c) 54. (d) 55. (d) 56. (c) 57. (b) 58. (c) 59. (c) 60. (d)
61. (b) 62. (d) 63. (a) 64. (d) 65. (a) 66. (b) 67. (c) 68. (c) 69. (a, b) 70. (b, d)
71. (b, d) 72. (3.00) 73. (9.00) 74. (0.16) 75. (0.50) 76. (61.00) 77. (25.00) 78. (0.00) 79. (7.00) 80. (2.00)
81. (d) 82. (c) 83. (d)
ANSWER KEYS 133

UNIT – IV
LIMIT CONTINUITY AND DIFFERENTIABILITY
Part-A
1. (d) 2. (c) 3. (c) 4. (c) 5. (d) 6. (c) 7. (b) 8. (a) 9. (c) 10. (a)
11. (a, d) 12. (b, c, d) 13. (a, b, c) 14. (a, c, d) 15. (b, c) 16. (b, c) 17. (a, b) 18. (a, b, c) 19. (a, b) 20. (0.00)
21. (4.00) 22. (2.00) 23. (b) 24. (b) 25. (a) 26. (a) 27. (a) 28. (d) 29. (c) 30. (d)
Part-B
31. (c) 32. (d) 33. (c) 34. (c) 35. (a, b, c, d) 36. (b, c) 37. (b, c, d) 38. (4.00) 39. (0.00)
40. (1.00) 41. (5.00) 42. (4.00) 43. (c) 44. (d) 45. (b) 46. (b) 47. (c) 48. (d) 49. (c)
FUNCTIONS, INVERSE TRIGONOMETRIC FUNCTIONS (ITF)
Part-A
1. (c) 2. (c) 3. (d) 4. (b) 5. (c) 6. (b) 7. (a) 8. (a) 9. (a, c) 10. (a)
11. (a,c,d) 12. (a,b,d) 13. (11.00) 14. (3.00) 15. (9.00) 16. (1.00) 17. (0.00) 18. (4.00) 19. (3.00) 20. (a)
21. (d) 22. (d) 23. (d) 24. (b) 25. (d) 26. (c) 27. (a) 28. (a)
Part-B
29. (a) 30. (d) 31. (b) 32. (b) 33. (a,b,c) 34. (b,d) 35. (8.00) 36. (5.00) 37. (4.00) 38. (8.00)
39. (4.00) 40. (b) 41. (a) 42. (c) 43. (a) 44. (a) 45. (c) 46. (b)

UNIT – V
DIFFERENTIATION, APPLICATIONS OF DERIVATIVES (AOD)
Part-A
1. (c) 2. (c) 3. (a) 4. (c) 5. (d) 6. (b) 7. (d) 8. (c) 9. (b) 10. (b)
11. (c) 12. (a) 13. (b) 14. (b) 15. (c) 16. (b) 17. (c) 18. (b) 19. (c) 20. (d)
21. (b) 22. (d) 23. (a) 24. (c) 25. (d) 26. (c) 27. (a, b) 28. (a, b, c, d) 29. (a, b, c, d)
30. (a, b) 31. (a,c, d) 32. (a, c) 33. (a, b, c) 34. (b, d) 35. (a, b, c, d) 36. (a, c) 37. (a, b, c) 38. (a, b, c)
39. (b, d) 40. (a, b, c, d) 41. (3.00) 42. (12.00) 43. (5.00) 44. (9.00) 45. (3.00) 46. (6.00) 47. (8.00)
 2 a 
48. (4.00) 49. (c) 50. (b) 51. (b) 52. (b) 53. (b) 54. (c) 55. (b) 56.  , 
 a 3
Part-B
57. (a) 58. (b) 59. (b) 60. (c) 61. (a) 62. (c) 63. (c) 64. (c) 65. (b) 66. (c)
67. (a) 68. (a) 69. (a) 70. (a, b) 71. (a, b, d) 72. (a, d) 73. (6.00) 74. (3.00) 75. (0.00) 76. (4.00)
77. (2.00) 78. (1.00) 79. (7.00) 80. (b) 81. (d) 82. (a) 83. (a) 84. (b) 85. (a) 86. (d)
2
87. (a) 88. (b) 89. (b) 90. (a) 91. (a) 92. 2,
3

UNIT – VI
QUADRATIC EQUATIONS
Part-A
1. (d) 2. (a) 3. (b) 4. (c) 5. (d) 6. (d) 7. (b) 8. (c) 9. (a) 10. (d)
11. (c) 12. (b) 13. (a, c) 14. (c, d) 15. (a, b, c, d) 16. (b, c, d) 17. (d) 18. (8.00) 19. (0.00)
20. (18.00) 21. (4.00) 22. (1.00) 23. (1.00) 24. (4.00) 25. (3.00) 26. (2.00)
Part-B
27. (a) 28. (d) 29. (a) 30. (b) 31. (a, d) 32. (a, c) 33. (b) 34. (2.00) 35. (8.00) 36. (2.00)
37. (9.00)
ANSWER KEYS 134

SEQUENCES AND SERIES


Part-A
1. (c) 2. (a) 3. (a) 4. (c) 5. (d) 6. (a) 7. (a) 8. (c) 9. (a, b,c,d) 10. (b, d)
11. (b, c) 12. (b, c) 13. (b) 14. (b) 15. (c) 16. (b) 17. (d) 18. (a) 19. (a) 20. (189.00)
21. (8.00) 22. (2.00) 23. (6.00) 24. (1.00) 25. (1.00)
Part-B
26. (b) 27. (a) 28. (a, b) 29. (b, c) 30. (a, b, c) 31. (a, b, c) 32. (c, d) 33. (b) 34. (c) 35. (a)
36. (c) 37. (2.00) 38. (2.00) 39. (5.00) 40. (7.00)

UNIT – VII
MATRICES AND DETERMINANTS
Part-A
1. (a) 2. (d) 3. (b) 4. (d) 5. (a) 6. (a) 7. (c) 8. (c) 9. (b) 10. (a, b, c)
11. (b, c) 12. (a) 13. (a) 14. (6.00) 15. (9.00)
Part-B
16. (c) 17. (a, b) 18. (a, c) 19. (b, c) 20. (a, c) 21. (d) 22. (4.00) 23. (1.00) 24. (c)
PERMUTATIONS AND COMBINATIONS(PNC), BINOMIAL THEOREM
Part-A
1. (c) 2. (a) 3. (b) 4. (b) 5. (c) 6. (c) 7. (d) 8. (d) 9. (d) 10. (c)
11. (a) 12. (a) 13. (a) 14. (b) 15. (d) 16. (c) 17. (b) 18. (a) 19. (b) 20. (a, b, c)
21. (a, c) 22. (a, b, c, d) 23. (b) 24. (a) 25. (b) 26. (a) 27. (b) 28. (a) 29. (a)
30. (10.00) 31. (7.00) 32. (6.00) 33. (102.00) 34. (10.00) 35. (5.00) 36. (3.00) 37. (2.00)
Part-B
38. (a) 39. (a) 40. (a) 41. (d) 42. (b) 43. (a) 44. (a) 45. (a) 46. (a, b, c) 47. (c)
48. (a) 49. (b) 50. (c) 51. (d) 52. (c) 53. (6.00) 54. (3.00) 55. (8.00) 56. (5.00)

UNIT – VIII
SOLUTION OF TRIANGLES (SOT)
Part-A
1. (d) 2. (a) 3. (a) 4. (d) 5. (a) 6. (d) 7. (b) 8. (b) 9. (b) 10. (a, b, d)
11. (a, c) 12. (b, d) 13. (a, b, d) 14. (b) 15. (a) 16. (d) 17. (b) 18. (d) 19. (a) 20. (a)
21. (b) 22. (a) 23. (8.00) 24. (3.00) 25. (5.00) 26. (2.00)
Part-B
27. (d) 28. (d) 29. (b) 30. (a) 31. (a, b, c, d) 32. (a, b, c, d) 33. (a, d) 34. (b, d)
35. (a, b) 36. (b) 37. (d) 38. (a) 39. (a) 40. (a) 41. (a) 42. (1.00)
TRIGONOMETRY, TRIGONOMETRIC EQUATIONS
Part-A
1. (d) 2. (b) 3. (c) 4. (c) 5. (d) 6. (a) 7. (c) 8. (b) 9. (c) 10. (a)
11. (a, c) 12. (a, d) 13. (c, d) 14. (a, b) 15. (b, c) 16. (a, b, c) 17. (c) 18. (6.00) 19. (1.00) 20. (6.00)
21. (1.00) 22. (3.00)
Part-B
23. (c) 24. (a) 25. (d) 26. (a) 27. (c) 28. (b) 29. (a, d) 30. (a, c, d) 31. (a, c, d) 32. (d)
33. (a) 34. (4.00) 35. (6.00) 36. (2.00) 37. (5.00)
ANSWER KEYS 135

UNIT – IX
STRAIGHT LINES
Part-A
1. (b) 2. (d) 3. (a) 4. (b) 5. (a) 6. (d) 7. (c) 8. (b) 9. (b) 10. (d)
11. (a, b, c, d) 12. (a, b, c, d) 13. (a, d) 14. (6.00) 15. (0.00)
Part-B
16. (c) 17. (b) 18. (b) 19. (b) 20. (c) 21. (b) 22. (a) 23. (d) 24. (b) 25. (a, d)
26. (a,c) 27. (d)
CIRCLES
Part-A
1. (c) 2. (a) 3. (b) 4. (b) 5. (b) 6. (d) 7. (a) 8. (d) 9. (a) 10. (c)
11. (b) 12. (d) 13. (a) 14. (b) 15.(b, c) 16. (a, b, c) 17. (1.00) 18. (14.00) 19.(0.88) 20. (4.00)
21. (8.00) 22. (1.00) 23. (0.00) 24.(3.00) 25. (7.21) 26. (4.00) 27. (9.00) 28. (2.00) 29. (a) 30. (a)
31. (a) 32. (d)
Part-B
33. (d) 34. (c) 35. (c) 36. (b) 37. (c) 38. (b) 39. (a) 40. (a) 41. (a) 42. (b)
43. (c) 44. (c) 45. (a) 46. (a, b) 47. (a , b, c) 48. (a, b, c) 49. (77.00) 50. (3.00) 51. (8.00)
52. (1.44) 53. (0.00) 54. (0.00) 55. (0.25)

UNIT – X
COMPLEX NUMBERS
Part-A
1. (c) 2. (d) 3. (d) 4. (a) 5. (b) 6. (d) 7. (a) 8. (a) 9. (a) 10. (c)
11. (c) 12. (a) 13. (a, b) 14. (a, b) 15. (a, c, d) 16. (3.00) 17. (3.00) 18. (3.00) 19. (6.00) 20. (0.00)
21. (1.00) 22. (6.00) 23. (d) 24. (a) 25. (c) 26. (c)
Part-B
27. (b) 28. (d) 29. (a) 30. (b) 31. (c) 32. (c) 33. (b) 34. (a) 35. (a, b, c, d) 36. (a, c)
37. (0.00) 38. (2.00) 39. (0.00) 40. (a)
CONIC SECTIONS
Part-A
1. (b) 2. (c) 3. (c) 4. (c) 5. (b) 6. (c) 7. (a) 8. (b) 9. (c) 10. (c)
11. (c) 12. (b) 13. (c) 14. (b) 15. (a) 16. (b) 17. (a) 18. (a) 19. (b) 20. (c)
21. (b) 22. (b, d) 23. (a, c) 24. (a, b, d) 25. (a, b, c) 26. (a, c, d) 27. (a, c, d) 28. (4.00) 29. (4.00) 30. (2.00)
31. (9.00) 32. (3.00) 33. (3.00) 34. (4.00) 35. (1.00) 36. (a) 37. (d) 38. (a) 39. (c) 40. (b)
41. (b) 42. (b) 43. (d) 44. (d)
Part-B
45. (a) 46. (b) 47. (d) 48. (a) 49. (b) 50. (b) 51. (a) 52. (b) 53. (c) 54. (a)
55. (b) 56. (a) 57. (a, b, c, d) 58. (a, b) 59. (a, b, ,c, d) 60. (1.00) 61. (2.00) 62. (1.00)
63. (5.00) 64. (5.00) 65. (5.00) 66. (3.00) 67. (b)
SOLUTIONS 136

Indefinite and Definite Integrals


e 1 t 1 dt
1. f e    dt  
2
e 1 2 2
t 1 2 t 2 1
f  f e  e 1 2 1
2 2

2
t  1  ln t  t 2  1  c
2
 
 e 
  f  e  f  f e
1
de   
1  e2  1
 e  de
 
1 4 1
x  1  ln x 2  x 4  1  c  
2 2
2 x
 e2  3 1 2
  e2  1   
2 1 2
5. 0 f  t  dt  2  f  x  

x
tdt 1 Differentiating with respect to x,
2. lim  
19 f  x  f  x f  x
 x x  4  x  a3  2t  104
x 0
0

  f   x   1 f  x   0
x
tdt Since f  x   0  f   x   1
0 a t
  104  f  x  x  c
 lim 3 2
x 0 xx  4  x  f 0  0  c
0
tdt  c0
0 a t Now, f  2   2  0  f  2   2
  104
3 2 1 
  6.
2
I   ecos x cos3   2n  1 x  dx, n  ... 1
1  4  0  19
0
contradiction 
cos 2   x 
3. Statement-1  e cos3   2n  1  x   dx
f  x   0 and sin x  0  f  x  sin x  0 0
a a
  
 Using  f  x  dx   f  a  x  dx 
 f  x  sin xdx  0
0
 contradiction
 0 0 

Similarly f  x   0  f  x  sin x  0
2
 I   ecos x cos3  2n  1    2n  1 x  dx
 0

  f  x  sin xdx  0
0
 again contradiction 
2
I   ecos x cos3  2n  1 xdx ...  2 
0
 f  x  changes sign Statement  2 is true
Adding 1 and  2  we get 2I  0  I  0
4. Substitute x 2  t  2 xdx  dt 2 1 2 1

1 t 1 7.  f  x  dx   f  x  dx   f  x  dx   f  x  dx
I  dt 0 0 1 0
2 t 1
2
1 t 1
 
2
dt   f  x  dx  0
t 2 1 1

If f  x   0 or f  x   0  x  1, 2 ,
SOLUTIONS 137

2 2
f  x  dx  0 or
 4t  1 dt  11 dt  3 dt
then 1
 f  x  dx  0
1
I 
 t  3 t  1  2  t  3 2  t  1
respectively 11 3
so f  x   e x  e  x   ax 2
 bx  c  must be I ln sin x  3  ln sin x  1  c
2 2
zero 3 11
On comparing, a  , b
for at least one value of x in 1,2  2 2
10. Since

As e x  e  x  2 so ax 2  bx  c must be zero  sin x 1
for at least one value of x in 1,2  0   x
2 2
8. We know that
b d
 sin x 
1   0
 f  x  dx   f  x  dx  bd  ac,
a c
 2 
 cos x 
where, f  x  is an increasing and invertible Similarly  0
function.  2 
4 
Also, f  b   d , f  a   c and  sin x dx  4 sin x dx  8
g  x   2  2sin x , then 0 0

11. f  x   ae2 x  be x  cx
g 1  x   sin 1  log 2  x  2  
Hence,  f   x   2ae 2 x  be x  c

2 4
f   log 2   31  8a  2b  c  31 ... 1
  2  2  dx   sin  log  x  2   dx
sin x 1
2 f  0   1  a  b  1 ...  2 
5
 log 4
2 2 39
 2 
5   f  x   cx  dx  2
0
4 log 4
39
  ae  be x  dx 
 2x
2

sin x 2
 2   2 dx 0
log 4
  ae 2 x  39
2   be x  
4
5  2 0 2
  sin 1  log 2  x  2   dx  2 
4  aelog16  a  39
5
2
  belog 4     b  
  2  2  2
2 4
5 15 39
 2sin x dx   sin 1  log 2  x  2   dx   a  3b   5a  2b  13 ...  3
 5 4 2 2

2 2 Solving, we get a  5, b  6, c  3
2 sin 2 x  cos x
9. I  dx
sin 2 x  4 sin x  3


 4sin x  1 cos xdx
 sin x  3 sin x  1
let sin  t  cos x dx  dt
SOLUTIONS 138

4
g  x   1  x  ln x  5 g  x
12. f  x    e x  t dt dx  c
0
17.  x  ln x 
6
 ln x 
5

x 4
  e x  t  dt   e x  t  dt g  x   1  x  ln x  5   x ln x  g   x   5 g  x 
 6
 6
0 x x  ln x  x  ln x 
x 4 x
 f  x  e  e 2 g x 1
 1  x  g  x   xg   x    1
 f  x  e  e x 4 x
0 g  x x
So f  0   f  4   e4  1 and  ln g  x   x  ln x  g  x   kxe x
f  2   2  e 2  1  g 1  ke  e  k  1  g  x   xe x
 g n  x   ne x  xe x
ba  e 4
 1  2e2  2  e2  1
 g n 1   n  1 e   n  1 g 1
13.    x   3sin x  4cos x  0
 x 
   2 sin 4
  x  is increasing in  ,  2 sin x
18. I   dx  2  dx
6 3 x x sin x  cos x
0 sin
 cos 0
 2 2
 f  x  attains maximum value at x 
3 Let sin x  A  sin x  cos x   B  cos x  sin x 
2 5
1 1 1
14.  f  x  dx  2 f  2 x  dx
1 1
A  B  1 and A  B  0, A  , B  
2 2
2 5 
1  1
  f  x  dx   f 1  2 x  d  2 x  I  2    2  log  sin x  cos x   04
1 1
2 4 2
2 10 
  f  x  dx   f 1  t  dt   log 2
4
1 2

 10  2  2 1 19.  
log 2 1  6 x  x 2  8  0  x   2, 4
 18 Maximum of     f  2   f  4 
15. f  x   0 at x  2 and x  3 x
x3
f   2   1  f  x     t 2  2t  2  dt   x2  2x
0
3
f   3  1 8 32
 f  2  8  and
m1m2  1 3 3
 64 136
sin x f  4   24 
16. I   1 x 2
dx ......  i  3 3

32 136 104
b b
 maximum of     
  f  x  dx   f  a  b  x  dx 3 3
a a x
 20.  3012
dx
 sin x dx
I   ....  ii  2 2  2 x  2 2012


1  x2 1  x   2 
 1 x 
Adding (i) and (ii)
2I  0  I  0
SOLUTIONS 140

1 10 dt 4  sin x 
26. g  x   I    
 a  b  b  c  c  a  9 9  t 2  9  5  4 cos 
 f  a  c  b  f  b  a  c  f  c  b  a    x
 

xa x b xc
 10  tan 2  4  sin x 
   1
tan    
f a 27  3  9  5  4 cos x 
1 a  
xa
1 a a2 10 4
f b A , B
1 b  1 b b2 27 9
xb 2
1 c c2 17
 x  x  1  3  x  1  2 x  dx
26
f c 28.
1 c 0
xc 2 2
18 17
f a   3x 26  x  1 dx  2 x 27  x  1 dx
1 a  0 0
xa 2 2
1 a a  3  x  118 x 27  227
f b  2   
  g  x  dx  1 b  xb 1 b b  
 27 0 9
1 c c2
f c 1000
1 c  29. I  sec1 x  dx
xc 
2
2
1 a f  a  ln x  a 1 a a 
 1  sec1   x2
1 b f  b  ln x  b  1 b b 2  k 2
1 c f  c  ln x  c 1 c c2  sec1 x   1  x  2
1000
sin x
27. Let t 
5  4cos x
I   1 dx  998
2
2
dt 5  4 cos x  5  9  1  30. By the concept of definite integration as the
    
dx 4  5  4 cos x 2 4  5  4 cos x  limit of sum, only option ‘B’ and ‘C’ shows
the limits range from (1, 2), where as option
5 1 9 A givens limit from (0, 1) option D gives
t  dx  I
4  5  4 cos x  4 limit from (0, 2)
9  sin x 5 1 31. Slope = 1
I   dx f  x  2 x
4 5  4 cos x 4  5  4 cos x 
x  2 x 1
sec2 dx
5 2 4  sin x  1 1
I      x  y
9  2 x  9  5  4 cos x  2 4
 9  tan 
 2 1 1
Hence tangents are y  x  and y  x 
x x 4 4
Put tan  t ,sec2 dx  2dt x
2 2 4t  1
32. f  x    dt
0
2t  1
x x
4t  1
f  x   dt   2t  1 dt
0
2t  1 0
SOLUTIONS 142

1 cos 6 x cos 7 x cos8 x cos9 x


Now area of i th loop (square)  (diagonal)2 39. Let f  x   3
2 1  e2sin 4x

1 2 2
As f   x   f  x 
Ai   2bi   2  bi 
2 
cos 6 x cos 7 x cos 8 x cos 9 x
So,
A i 1 2 bi 1

  
1  I  10 
1  e 2sin
3
4x
dx ... 1
2 0
Ai 2  bi  4
So the area from a GP series I  10 
 cos 6 x cos 7 x cos8 x cos 9 x e 2sin  3
4x
 dx
So, the sum of the GP Upto infinite terms 2sin 3 4 x
0 1 e
1 2 1 ...  2 
 Ai  2  32  
1 r 1
1 
4 2I  10 cos 6 x cos 7 x cos8 x cos 9 xdx
2 4
  8 2 0
 2  32        32  square units 
3
  3 2
38. f  4  x   f  4  x  I  5  cos 6 x cos 7 x cos8 x cos 9 xdx
0
 f  2   2  x   f  x  
2
 f  x  is periodic with period 4  10  cos 6 x cos 7 x cos8 x cos 9 xdx
50 48 0

 f  x  dx   f  x  dx  k  10
0 0 
50 4 2 2
  f  x  dx  12  f  x  dx   f  x  dx 40. I  5 cos 6 x cos 8 x  cos16 x  cos 2 x  dx
48 0 0 0

 Put x  t  48  2

 2 4
 I  5  cos 6 x cos8 x cos 2 x  dx
 12   f  x  dx   f  x  dx   5 0
0 2  
2
2 2
   5 cos 6 x cos8 x cos16 xdx
 12   f  x  dx   f  4  x  dx   5
0
0 0 

 24  5  5  125 4
46 2  2  48  10   cos 6 x cos8 x cos 2 x  dx  0
 f  x  dx   f  x  dx   f  x  dx 0
4 4 2    10
2 4

  f  x  4  dx  12 f  x  dx 4
0 0 41. I  5  cos14 x  cos 2 x  cos 2 xdx
2 2 0
  f  x  dx  24  f  x  dx 

0 0
 5
4
 cos16 x  cos12 x   cos 4 x  1 dx
52 4 4  48
0
2
 f  x  dx   f  x  dx   f  x  dx  125
2 2 4

5 sin 4 x sin12 x sin16 x  4
51  x   
2 4 12 16  0
But  f  x  dx  125
1 5

8
SOLUTIONS 144

y
tan1
 
0dx  1dx  lim f  x   f  x   lim f  x  x   0, 

1

tan1 y  tan1 x

2
x

2
 2
lim y
 lim 1
y  y y 1 sin x
2
  1
1dx
1
 x
Matrix Matching : 3 dy x
45. (A) Given    m1
dx y
44. (A) Slope of line joining origin to  x, y  is
2 2
For any x   0, 1 , 1  x  1  x 4
 y
m2 
1 1 x
  1 mm
1 x 2 1 2  1  curves are orthogonal
1  x2
(B) (B) Given, dy   y
For any x   0, 1 , dx x
y
slop of line joining origin to  x, y is m2 
4  2 x2  4  x 2  x3  4  x 2 x
1 1 1 (C) dx  dy  0  y   x  c
  
4  x2 4  x 2  x3 4  2 x2 Its family of straight line with slope 1
(C) dx dy
(D)   0  y  xc
For any x   0, 1 , x y
2 Its family of straight line passing through
1  x 2  1  x 6  1  x 
origin.
1 1 2 5
  1 46. f   x   x  e x  1  x  1 x  2   x  3
1 x 1  x6
f  x  has maximum at x  2  k  2

 2 2 1  I  1  Now sin x  cos ecx  2  k  2
So, C → q, s
(D)  sin x  1
sin x    sin n x  cosecn x  2
Let f  x   x   0, 
x  2  52 
2

1 1 
x cos x  sin x g  x  47. I    f  x  dx   f  x  dx 
f  x   2  0 
x2 x 
0

where g  x   x cos x  sin x 1  5  
   1
    4 4 
Now, g   x    x sin x  0x   0,  e
 2 1  ln x
48. I   dx
  x ln x
 g is decreasing function for all x   0,  1
 2 Putting  x ln x   t 2 ,
   ln x  1 dx  2tdt , we get
 g  x   g  0  x   0,   g  x   0
 2 e
2tdt
  I  2 e
 f   x   0x   0,  t
 2 0
2
  4 2 2 5 a
 f is decreasing function  0,  49. x a x x   1
 2 x
SOLUTIONS 145

sin x
dx et dt
52. g  x   e 2 x  
I x3 0
cos 2 x  2t sin x  t 2
2
a g  0  1
x2   1
x sin x
et
2x sin x
a 2
2a 2 g  x  e  e  dt
Let  t , so  3 dx  dt 0
1 t2
2
x x sin x
et esin x e  sin x
1 t g   x   2e 2 x  esin x  cos x  dt  cos x
I 4  dt 0
1 t2 1  sin 2 x
2a t 1
g 0  3
1 t 1 1 1
 4  dt  4  dt g   0   4  1  5
2a t 1 2a t 1
1
1 1 1 xa  xb
  4  t  1 dt  4  dt 53. Let f  a    dx
2a 2a t 1 0
log x
1 a 1
1  1  3 d x log x 1
 4  t  1   t  1   c 2
da
 f  a   
log x
dx   x a dx 
a 1
a  3  0 0

1
1  a x 2 2
1 3
f a   da  c  log a  1  c
 4  3  a2  x2  2   c a 1
a  x 3x  a 1
f  b   0  f  a   log
50. I   x8
 x2  20  dx
b 1
2
 x 5
sin x  5 x 4
cos x   f  a   log a  1  log b  1  log
a 1
b 1

x 5
 20 x3  cos x

x5
dx 3
2 sin x  x cos x  sin x 
x 5
sin x  5 x cos x 
4 cos x 54.   cos2 x  dx
e

x5 1
  5  sec2 x  C   esin x  x cos x  sec x tan x  dx
cos x  x sin x  5 x 4 cos x  
  esin x   x cos x  1  1  sec x tan x   dx
x
  tan x  C
cos x  x sin x  5cos x   esin x  x   esin x  sec x   c
1
2  esin x  x  sec x   c
51. Consider    x  f  x  dx
0 55. cos  sin    Re  e i sin  
1
   2 f  x   2 xf  x   x 2 f  x   dx  ecos cos  sin    Re  ecos i sin 
0 cos  cos 2
  2  2 2   2  1   ...
1! 2!

0 cos 2
 I   1  cos    ....d  
However f  x  assumes only 0
2!
positive values that is in  0, 1 56. Conceptual

2 2
   x   f  x   0 57. I n   cos n x  cos nx dx
 integral can't be zero 0
SOLUTIONS 146

 4x
sin x 2 sin nx tan z  2
 cos x n
  n cos n1 x sin x dx x 4
n 0
n 2
 x  z  x  2 tan 1  
2  x
   cos n1 x cos nx cos x  cos  n  1 x   x2 
0  d  x  z   2  dx
  I n  I n1  x 4
So, I  ln cosec y  cot y  C
cos  nx  x   cos nx cos x  sin x sin nx 
y  x z
1 1
x2  1 6 1
0 x 4  x 2  1 dx  0 1  x  1  x  dx
4
2 I n  I n 1 61.
I n 1
2 1
In  x5 x 7 x11 
x    ... 
1
 5 7 11 0
 x n 1  1 x n 1 1
58. I n   tan 1 x     2
dx 1 1 1 1
 n 1 0 0 n  1 1  x  1      ...
1 5 7 11 13
 x n 1
 n  1 In   2
dx n
k
62.   1 n Ck 
1
4 0 1 x
k 0 k  m 1
1
 xn 3 n 1
 n  3 I n  2   dx    1 nCk  t k  m dt
k
4 0
1  x2
k 0 0
1  1 n
  n  1 I n   n  3 In  3   k
2 n2  t m    1 n Ck t k dt
0 k 0
 an  n  3  a1 , a2 , a3bn ...are in AP
1
n
bn   n  1  b1 , b2 ... are in AP   t m 1  t  dt
0
 1 1 1
cn   not in any progression m
k m k m
n
k m
2 n2   1 Ck  t dt   t n   1 C k t k dt
1 k 0 k 0
0 0

 1  2 x  x  f  x  dx  0
2
59. 1
m
0   t n 1  t  dt
1 0
2
  1  x 
0
f  x  dx  0 1
  1  t  t m dt
n

 f  x  has to change its sign 0


1

 e 
x3 3
atleast once in  0, 1 63. I   e x dx
2 1
x
dx 1

60. I   2
x 4
x 4
4x
2
 3
 2 e x  e x dx  2
0
3


2
sin x  2 cos x 3 3
x 4 x 4 Since e x  e  x is concave upwards,
d x  z
 I  f  0   f 1
sin  x  z 
1
I  2e
e
SOLUTIONS 148

10   1
xx 
 1
xx    max value of f  x   6
I    xe  2   xe  2   dx  0
  (D)
0  
 1  1 1 1  x   x  ;  x   odd
  xx 
2
 x  x     
2 2 2 f  x  
 e    e    x   x   1 ;  x   even
 
4 1 0 1
 1   1
 x   x  1
 2 
x x 
 2  f  x  dx  3     x  1    x  1   1  x 
e e 2 2 1 0
(d) 2 3 4

 e x  e x     x  1    3  x     x  3  3
Let P be any point  x,  1 2 3
 2  1 0 1
2 2
D be distance between  0, 0 
 x  x   x2 
  x     x   x  
 2 2  2  1  2 0
and PD  0  x  0
2 3 4
 Minimum of D  1  x2   x2   x2 
   x    3x      3x   3
 2 1  2 2  2 3
70. (A) 3 1 1 3 5 7
x 2  y 2  xy  60  1      1  3    3  3
2 2 2 2 2 2
  2x  y  0
2 x  2 yy  xy  y  0  y  
2 y  x 71. I n   x n  a  x  2 dx
1

 dy    2h  k  3 3
   2 2n n 1
 dx  h ,k   2k  h    xn  a  x  2   x  a  x  2 dx
3 3
Equation of line from  0,0 which cuts the 2 3
2n n 1 1
  xn  a  x  2  x  a  x  a  x  2 dx

curve at  h, k  is 3 3 
3 1
2 2na n 1
 2k  h  x  2hy  ky  2kx  hx   xn  a  x  2   x a  x2
y 3 3
 2h  k  2n n 1
 x  a  x  2 dx
 It passes through  h, k   h   k 3 
3
Also, h 2  k 2  hk  60  h  k  20  2n  2 2an
 1   In   x n  a  x  2  I n 1
 3  3 3
 S.D. of origin and  h, k   2 10 3
  2n  3 I n  2 x n  a  x  2  2anI n1
(B) I 
2011
1  x  
9
1 x 18

dx  2011 Putting n  2, we get



2011
2x 9
3
a 7I 2  2 x 2  a  x  2  4aI1
  f  x  dx  0  f  x  is odd 3
2 4a
a  I2   x 2  a  x  2  I1
2011
1 7 7
I   dx  2011  0    2,   2,   1
2011
2
1 1
(C) 72. y  1  x 2  3 and y  1  x3  2 are inverse of

f  x   max x2 , x  1,3x ; x   0, 2  each other.
 f  x   3xx   0, 2
SOLUTIONS 149

 n 1
 n  1 1 1 1 2
73. I n     x k 1    2k  1 x k 1  dx, n  As     ... 
0  k 1  k 1  12 22 32 42 6
2 2
1 1 1  1  
  1  x  x 2  ...  x n 1  or,  2  2  ...     
0
1 3  4 6  6
1
1  3x  5 x 2
 ...   2n  3 x n  2   2n  1 x n 1  dx ln x 3 2 2
 0 1  x 2 dx     
Substitute x  t 2  dx  2tdt 4 6 8
1

 sin x  cos x  dx
n 1
I n   2t 1  t 2  t 4  ...  t 2 n  2  77. I n   2  sin x  cos x 
0
0
 2   n  1
1  3t 2
 5t 4  ...   2n  3  t 2 n  4   2n  1 t 2 n  2  dt

1 n2 2
  2  t  t  t  ...  t3 5 2 n 1
   sin x  cos x   cos x  sin x 
0
2
dx
0
 2   n  1
1  3t 2
 5t 4  ...   2n  3 t 2 n  4   2n  1 t 2 n  2  dt 
n2
 2   sin x  cos x 2  dx
Substitute y  t  t 3  t 5  ...  t 2 n 1   sin x  cos x 
0
2
 
 dy  1  3t 2  5t 4  ...   2n  3 t 2 n  4  2  2  n  1 I n  2   n  1 I n
  2n  1 t 2 n  2  dt  nI n  2  n  1 I n  2  2
n
x
I n  2  tdt  n 2  I9  92  9 78. Put t  tan ,
0 2
 3
 1
 30  2t 2  dt
 
2 2
1
1  x 1
 I
2 2
75. I n   x n 1 x 1  x 2 dx   x n 1   dx 0 15  t 
0 0  3  1 1
  1 1
1 3  3 dt  60 dt
1 2 2 2
0 15  t 
   n  1 x n  2 1  x 2  2 dx 0
15  t
30 1 1
1 1
 3I n   n  1  I n  2  I n  I1   dt , I2   dt ,
2 2 2
0 15  t 
0
15  t
In n 1
or   1 as n   1 1
In  2 n2 I1  30I2  , I  , 56I  8
1
14 7
ln x 
76.  1 x 2
dx 2

0 79. I1   3 sin 2 x sin xdx ....  i 


1 1 0
 1 1 x  1  1 x  1 b b
  ln x  ln    ln    dx
 2 1 x 0 2 0  1  x  x   f  x  dx   f  a  b  x dx
0 0
 x 2 x3   x 2 x3  
1 
x    ... 
   x    2
1  2 3   2 3  dx
 0   I1   3 sin 2 x  cos xdx ....  ii 
20 x 0
1
 x2 x4 
    1    ...  dx
 i    ii 
0
3 5 

2

1 1 1 1  2 I1   3 sin 2 x sin x  cos x  dx


   2  2  2  2  ...  0
1 3 5 7 
SOLUTIONS 150

2a a

 f  x  dx  2 f  x  dx if f  x   f  2a  x 
0 0

4
I1   3 sin 2 x sin x  cos x dx
0

4
 
I1   3 sin 2 x  sin   x   2dx
0 4 

4
 I1   3 cos 2 x  cos x 2dx
0

 k  2 or k 2  2
SOLUTIONS 153

2 4 when y  2

2 1 1 
x  4  2 
 a4 &b2 e e 
 dy  dy 1   e  1
20.   2 x     0  4 2 
 dx  dx 2 x   e 
1
 dy  2 x dx or dy  dx  x   0 when y2
2x 2
1 23. Let x  64 cos t
 y  x 2  c and y  ln x  c are solutions y  64 sin t
2
d2y dx
 192 cos 2 t   sin t 
2 dy dt
21. (A) dx 2  e 2 y
 dy  dx Equation of tangent at t
   sin t
 dx   y  64sin 2 t   x  64 cos2 t 
d2y cos t
2 y  sin t
  dx 2 dx   e 2 y dy   64sin 2 t   x  64 cos 2 t 
 dy  sin t cos t
  x y
 dx    1
64 cos t 64 sin t
1 e2 y
   C1  Segment of tangent between  and 
dy 2
2
dx   64  cos 2 t  642 sin 2 t  64
 e2 y  Length  64  x 2
  C1  dy  dx
 2    8
2y
e 24. Area
  C1 y  C2  x
4  0
 x2 2 
(B) Homogeneous DE 
2
    2  4  x  dx
 2
(D) Bernoulli’s Equation
22. ye x dx  2 xe y dy  y 3dy  1 
 
dx 2 2 3
 ye y  2 xe y  y 3
dy 1
 
dx 2 x 3
   y 2 e y      1 3  4
dy y 1
1 25. y   y  c , where c   y  x  dx
I.F.  2
y 0

x dy
  e  y dy  e  y  C  dx
y 2 yc
ln  y  c   x  d
 0  e 1  C
1 y  0  1
C 
e  ln 1  c   d
1 1  y  1  c  e x  c
x  y2   y 
e e 
SOLUTIONS 154

1 1
c   y dx  1  c  e  1  c   2 log x  log y  C or
xy
0
1
e 1 2 2 log x  log y   C
x , 1 c  xy
3e 3e
x 32. f  x   1  ln x
2e  1  e
y  x  1
3e  1
Area  2 e y 1dy  2 1  
2e x 0  e
y  x   0
3e 33. Put U  y 2 then equation will be transformed
2 into the linear equation
26. Subnormal = y  0  . y   0  
3e d  1  1 sin U
2  2  2  IF  eU
27. y  2xy  40 x  400  0 can be factorized du  2 x  2 x 2
as  y  20 y  2x  20  0 for x  0 and  1  eU sin U
 Solution is eU   2    dU give
 2x  2
 y  20 y  2x  20  0 for x < 0 the result A.
The bounded region is parallelogram of area
34. p  sin  px  y 
20 × 40 = 800 sq. units
0  px  y  sin1 p ... i 
28.   5  2  ln  x   x  dx
1/2
Differentiate w.r.t. x
0
 10  ln  x  1  5 1  ln 2 x
dp

1 dp
1/2 dx 1  p 2 dx
30. The equation can be written as
d x dp 1
  0 or x
dx

e  f   x   f  x   2  dx 1 p2
 e  x  f   x   f  x    2 x  c1 x2  1
 p put in equation
 f  x    x 2  c1 x  c2  e x and x
f   x    x 2   c1  2  x  c1  c2  e x x2 1
y  x 2  1  sin 1
x
Given that f   x   0  c12  4c2  4  0 a = 1, b = 1, c = 1
 c12  4c2  0  f  x   0  dy dx  1 1
35. x 2 y 2  2  2   x 2 y 3    dy  0
31. Given differential equation is y x   y x
 xy 2  2 x 2 y 3  dx   x 2 y  x3 y 2  dy  0 1 1 1 1
3 3
 d     y    dy  0
Dividing by x y x y  y x
 1 2  1 1 1 1
  2   dx   2   dy  0 d  
 x y x  xy y x y
   y dy
 ydy  xdy  2 1 1 1

 2 2   dx  dy  0 x y
 x y  x y
 Solution is 1 1 y2
 ln   c
x y 2
k 2
36.
SOLUTIONS 158

0  1 p  3 1
1 
1   p  2 p 6
1  p  2 ...  ii  1 2 3 4 5 6
1   2    3    4    5    6  
6 6 6 6 6 6
1 2 p
Also, 0  1 1
2 
0  1 2 p  2 1  2  3  4 2  52  6 2
2 2 2

6 1
1  2 p  1  
6  7 13 91
1 1 16. Probability
 p ...  iii 
2 2 2 2 2 2

1 4 p 1 p 1 2 p 
 n
C0    nC1    nC2   ....   nCn 
Also, 0    1
4 3 2 2 n.2n
2n
0  3  12 p  4  4 p  6  12 p  12 Cn

0  13  4 p  12 22 n
13  4 p  1 
 2 n !
2n
2  n ! n!
13 1
 p  ...  iv   2n  2n  2  ....6.4.2   2n  1 2n  3 ....5.3.1
4 4 
From (i), (ii), (iii) and (iv) 22 n  n!  n!
1 1 2n  n !  1.3.5....  2n  3 2n  1 
 p
4 2 
22 n  n !  n !
15. A1  Bag contains exactly 1 white ball
1.3.5....  2n  3 2n  1
A2  Bag contains exactly 2 white balls 
2n  n !
-------------------------------------------------
An  Bag contains exactly n white ball
6
C1   5 C1  5C1  5C1  5  60
17. 18

B  a white ball is drawn C6 1547
A  18. Required probability
P 1  6
B C  2  3C  3C1 45
 2 18 2 
B C3 136
P  A1  .P  
 A1  19. ADD or BDD
 111 3 111 5
B  B 3
C1 .  C1 
P  A1  .P    P  A2  .P   266 3 6 6 72
 A1   A2 
20. ABA or BAB
 B 111 11 5
....  P  An  .P     
 An  2 3 2 3 2 3 36
P  Ar   r 21. p (atleast 1 game, win by B)
For n = 5:  1  P  DAA or DDD or ADD or AAA 
1 1 1 1 1  19
1   1     
p 5  8 216 24 8  27
1  2 3  4 5 22. (A) There are 4 vowels e, e, o, a and 5
1   2    3    4    5  
5  5 5  5 5 consonants R, S, N, N, C.
1 6 1 If vowels and consonant should came
 2 2 2 2 2
  alternately is that consonants occupy odd and
1 2 3 4 5 5  6 11 55
For n = 6:
SOLUTIONS 159

vowels occupy even places in the ‘9’ letter n  n  n  4 6  n  3


word 25. Pn  n

5! 4!
C3  n  1 n  2 
Hence number of ways is   720 4
2! 2!  p7 
(B) x  y  z  w  20 where 1  x, y, z, w  6 5
26. Way 1: 1 take 1, 2 take 2, 3 take 3,
Let x  6  t1 , y  6  t2 and so on and 4 take 4
Equation becomes, t1  t2  t3  t4  4 1 1
p(way 1) =  1  1  1 
0  t1 , t2 , t3 , t4  4 4 4
Number of solutions is 7 C3 Way 2: 1 take 2, 2 take 1, 3 take 3, 4 take 4
1 1 1
C3 35 7 p(way 2) =   1 1 
Hence p   4 4 3 12
64 6 Way 3: 1 take 2, 2 take 3, 3 take 1, 4 take 4
(C) w1w2 w3 w4 ends in 5 1 1 1 1
p(way 3) =   
1
4 4 3 2 24
Pr (Product is odd) =   Way 4: 1 take 3, 2 take 2, 3 take 1, 4 take 4
2 1 1 1
4 4 p(way 4) =  1   1 
 4  2 4 2 8
Pr (product ends in 1, 3, 7, 9) =     
 10   5  1 1 1 1 1
Required probability =    
 1 16   369  4 12 24 8 2
Hence q     
 16 625   10000  27. Let E1 be the event of both getting the correct
(D) npq  180 p  q  1 answer and E2 be the event of both getting
n is minimum when pq is maximum which is wrong answer.
1 Let E be the event of both obtaining the same
at p = q = answer
2
Hence minimum value of ‘n’ is 720 1 1 1
p  E1    
56 14 8 12 96
(E) x1  x2  x3  x4   A
12 3  1  1  77
p  E2   1   1   
23. x can be 2, 3, 4, 5, 6  8   12  96
The number of ways in which sum of 2, 3, 4,
5, 6 can occur is given by the coefficients of E  E  1
P    1, p   
x 2 , x3 , x 4 , x 5 , x 6 in  E1   E2  1001
 3x  2 x 2
 x 3  x  2 x 2  3 x 3   A  1 P  A  B 1 x
28. P     2 1
 3 x 2  8 x 3  14 x 4  8 x 5  3 x 6 B PB x
This shows that sum that occurs most often is 5 1
4. x
3 3 3 2
2
1 1 1 1 29. For r = 5:
24. P        3C1     
2 2 2 2
3
1 1
 3C 2     
 2  2
1  6  12 19
 
64 64
SOLUTIONS 160

1 1  C1. C1. C1  C1. C1. C1 


1 1 4 2 2 2
P1 P2 P3 P5 P5 
25 6
C3
32. 10 
3
1 7 14
P1 P2 P4 P5 P5  5
2 25
1 33. The scores of ‘n’ can be reached in the
P1 P3 P4 P5 P5  5
2 following two mutually exclusive events
1 (i) by throwing head when the score in n – 1
P2 P3 P4 P5 P5  5 (ii) by throwing tail when score is n – 2
2
1 1 1 1
k  4 5    Pn  Pn 1   Pn  2
2  8 2 2
For r = 6: 1 1
34. Pn  Pn 1  Pn 1  Pn  2
1 2 2
P1 P2 P3 P4 P6 P6  1
26  Pn  2  Pn 3
1 2
P1 P2 P3 P5 P6 P6  6 1
2  P2  P1
1 2
P1 P2 P4 P5 P6 P6  6 1
2 But P1  and a score of 2 can be obtained
1 2
P1 P3 P4 P5 P6 P6  6 by throwing a tail at a single toss or a head at
2
the
1
P2 P3 P4 P5 P6 P6  6 first toss as well as second toss
2 1 1 1 3
 1 5 P2    
k  5 6   2 2 2 4
 2  64 1 3 1 1
30. x  y  z  6  Pn  Pn 1     1
2 4 2 2
x 1 1
Then the number of possible ways = 10 of 35. Pn  1  Pn 1
2
which x = 2 = y = z is one solution
2 1 1 1  2
1 Pn    Pn 1   Pn 1  
 PE  3 3 2 2  3
10
2
31. E1  When three balls are drawn they are  1   2
    Pn  2  
found to be of different colours  2   3
E2  The bag contains 4 red balls  1 
n 1
 2
 E  P  E2  E1 
   P1  
P 2    2   3
 E1  P  E1  n 1
 1   1 
 
1  3. C1  6. C1 . C1 . C1  C1 . C1 . C1   
4 3 2 1 2 2 2

P  E1    2   6 
6
10 C3 n 1 n
 1  1 2  1
7      Pn  
  2   3 3 3.2n
25
2 1 2
1 4 C1 1 4 1  P100   100 
P  E2  E1   .6    3 3.2 3
10 C3 10 20 50
2 1 2
 E  1 25 1 P101   101 
 P 2     3 3.2 3
 E1  50 7 14
SOLUTIONS 161

36. (A) Probability of A throwing a 7 and B 1


n
1 1 1
n 1

1  C0    n C1     
n

throwing a 4 is  2  2   2  10
72 n
Probability that neither A throws a 7 nor B  10  n  1  2
 1  1  55 By trial and error, we set n  7
throws a 4 is  1   1     Least value of n  7
 6  12  72
Probability that one of A or B is declared a 2 2
Hence, 
1 55 2 n 7
winner in a given round  1    37. (A) The required event will occur
72 72 9
Probability that A wins in a given round If last digit in all the chosen number is 1, 3, 7
or 9.
1 11 11
   n
6 12 72 4
Therefore the required probability   
11  10 
11 (B) P  last digit 1, 2, 3, 4, 6, 7, 8, 9 
Probability that A wins  72 
2 16  P  last digit is 1, 3, 7, 9 
9
(B) Suppose A hits the target at the i th shot. 8n  4 n

This means that B will not hit the target till 10n
his i th shot and at whatever shot he hits the 5n  4 n
(C) P 1, 3, 5, 7, 9   P 1, 3, 7, 9  
target is immaterial. 10 n
Thus required probability
i 1 i (D) P  0, 5    5 
10 n
 8n    5n  4 n 

2 3 2 10n
     
i 1  5  5 7 38. (a) Let Ai  i  1, 2, 3, 4  be the event that urn
2
6  4  4  contains 2, 3, 4 or 5W balls and B the event
 1      ... that two white balls is drawn we have to find
35  35  35  
A 
6 P 4 
  B
31 Applying Bayes theorem
(C) Target can have exactly 2 hits in the 1
following ways 1
 A4  4 1
(i) One missile is destroyed and two missiles hit P   
2  B  1  1  1  3  1  6  1 1 2
3 1  2 3 1 4 10 4 10 4 10 4
the target  C2      
2  3 4 4 (b) n, n  3,...m  12
(ii) none of the missiles are destroyed but only 2 n  1,....
3
 2 31 1 n  2,...., n  14
hit their targets     3C2   
 3 44 8 3  5C3  53 31
1 1 3 Required probability = 15

 Required probability    C3 91
4 8 8
3! 2  3
(D) P  x  2   0.9 (c) Required probability = 
4  4  4 16
 1  P  x  2   0.9 (d) For the balls in box there are three
 P  x  2   0.1 possibility
(i) All the 4 balls are red
(ii) 3 of the 4 balls are red
SOLUTIONS 162

(iii) 2 of the 4 balls are red   5  1!   24  5


Let these represented by E1 , E2 and E3 40. P  1    1   
 D5   44  11
respectively
41. Total number of ways of numbering
Assuming that E1 , E2 and E3 are given to be 5  3!  30
equally likely Total favourable ways  1 2  3 2  12 ways
1 12 2 m
P  E1   P  E2   P  E3    
3 30 5 n
Let A be the event two balls drawn are red So, m  n  7
 A  4C  A  3C 1 42. Let p and q be the remainders, when x and y
P    4 2  1, P    4 2  and
 E1  C2  E2  C2 2 are divided by 5.
So,
 A  2C 1 x  5m  p
P   4 2 
 E3  C2 6 y  5n  q
3
 A 0  p  4, 0  q  4
P  A    P  Ei  .P  
i 1  Ei  and m, n, p , q  W
1 1 1 1 1  x2  y2
 1    
3 3 2 3 6  25  m 2  n 2   10  mp  nq    p 2  q 2 
5
 So, x 2  y 2 is divisible by 5 when p 2  q 2 is
9
Thus, divisible by 5
1 1 1 Now, total number of ways of selecting p and
1  q = 5  5  25
 E 3 E
  3
P 1   3  , P 2   3 2  and Now, 0  p 2  q 2  32
 A 5 5  A 5 10
9 9 For favourable ways,
1 1 p 2  q 2 can be 0, 5, 10, 15, 20, 25 or 30

E  1 p 0 1 2 3 4
P 3   3 6 
 A 5 10
9 q 0, 5 2, 3 1, 4 1, 4 2
E E E
Now, denote the events 1 , 2 , 3 by Number of favourable ways = 9
A A A 9
F1 , F2 , F3 respectively, then Hence, P 
25
 25P  9
3 3 1 43. Let line of arrival of A be taken along x-axis
We have P  F1   , P  F2   , P  F3  
5 10 10 and that of B along y-axis meeting between A
Let B is the event that next draw is red ball and B taken place if and only if x  y  15 .
B B 3 B 2 Let x and y be the coordinates in the plane
P    1, P    , P   
 F1   F2  4  F3  4 and scale be taken in minutes.
3 3 3 1 2 7 Therefore all possible out comes will be a
P  B   1      square with side 60 the favourable out come
5 10 4 10 4 8 will be in shaded region to the whole area =
6 2
5 1 5 602  452 7
39. p     111     .
6 6 6 602 16
SOLUTIONS 163

Vectors & 3D Coordinate Geometry (Solutions)


2
1.
k k k
Let Q  , ,  be the orthocentre  p  2x   p  q  r   0
a b c pq r
Then, QA, QB and QC should be coplanar x
2
4. Q is point of intersection of line
k k k x  2 y z 1
a   and plane x  3 y  3 z  0 .
a b c 1 3 2
1
k k k  1 1 1 5. No Solution
b 0 k  2  2  2 
a b c a b c  6. No Solution
k k k 2
c 7.  a  b b c c  a  =  a b c  =
a b c
2 2 2
c .  a  b  = c a  b cos 2 
x y 1 z  2 2
2. Any point on   is The maximum value of  a b c  is
2 3 1
 2 , 1  3 , 2    possible only when c is parallel to a  b
So, vector from A(1, 0, 3) to this point is    0 or 
2 2 2
 2  1, 1  3 ,   1  
Hence, a  b  c  a  b c sin 2   0
Since it is parallel to x  y  z  2 , we have
8. Given OA  OB  OC  OD  r  say 
1
 2  1, 1  3 ,   1 . 1, 1, 1  0    Squaring the given equations, we get
6
So, the point is  x2  y 2  r 2  2 xyOA.OB =
1 1 1   1 3 13 
 , 1 , 2     , , 
x 2
 y 2  r 2  2 xy OB.OC
3 2 6 3 2 6 
  x 2  y 2  r 2  2 xy OC.OA =
Direction ratios of vector are
1 3   2 3 5 x 2
 y 2  r 2  2 xy OD.OA
  1,  3     , ,  
3 2   3 2 6  OA.OB  OB. OC  OC.OA  OD. OA
Equation of line is  cos  AOB = cos  BOC  = cos  COD  =
1    13 
x   y   z   cos  DOA 
 3   2   6 ,
 2 3
  5 Since sum of these four angles is 2 and all
      angles are equal, hence ABCD is a square.
 3 2 6 9. The required ratio is
3x  1 2 y  3 6 z  13
that is   OA  OB  OB  OC  OC  OA
2 3 5
3x  1 2 y  3 6 z  13 OC  OA
or  
2 3 5 Also, OA  OB  3OC  OB  0
3. Without loss of generality, we can assume
that  OA  OB  3 OC  OB
p  p iˆ, q  p ˆj, r  p kˆ Similarly OA  OC  2 OB  OC
The equation becomes
2 2 2 OA  OB OC  OA
3 p x p  p q r  p x 0   OB  OC  
3 2
SOLUTIONS 164

6 x 2  y 2  8 x  10 y  40  0
Therefore required ratio is 3
2 C    4, 5  , r  1
10. p  q   a  b    a  b   2  b  a 
 2
P1  max  x  2    y  3
2

 pq  b
2
 2 2
a  b . a 
2
 P  min  x  2 
2
  y  3 
2
2
2

 p  q  4 625   b . a 
2
  P   2, 3
1


 p . q  2 625   a . b  
2 2
CP  2 2
2 2

   625  
Then P2  2 2  1 , P1  2 2  1  
11. Equation of plane will be 17. r  xiˆ  yjˆ
x y x 2  y 2  8 x  10 y  40  0
 1    z   0
a b C    4,5 , r  1
1 1
Also,  c   0  d    0
a b P1  max x  2   y  3 
2 2

12. The new position vector   2iˆ  ˆj  kˆ   iˆ   P  min  x  2    y  3 


2
2 2

13. No solution
14. Plane P passes through (–1, –1, 2) and  P   2, 3
direction ratios of normal to the plane are (–3, CP  2 2
3, –1). 2 2
15. A  
Then P2  2 2  1 , P1  2 2  1  
 a  b  .b  c   b . c  a .b   a . c P1  P2  18
b  c  . c  a    a . c  b . c   a .b 18. A point on the given line is  2t, t, t  and

 c  a  . a  b    a .b   a .c   b .c this is a point on the given plane iff t  2


3
Given that a  b  c  3 19. Locus of ‘Q’ is the line of intersection of the
plane x  2 y  3 z  4 and
  a  b  c  . a  b  c   3
1 x 1  1 y 1  1 z 1  0  then the
 a .b  b .c  c . a  0
line is x  y  5  z  2
    a . b  b . c  + b .c   c . a  + 1 2 1
 
 c . a   a .b  – a .b  b . c  c . a 20. a  b  3

    a . b  b . c  +  
 1  1  2 a.b  3
b .c   c . a   c . a   a .b   1
  0 (since x y z  0,
 
 a.b 
2
 
xy  yz  xz  0 ) Given, c  a  2b  3  a  b 

 max  0 only when a . b  b . c  a . c  0 Taking dot products with b we get
 
16. r  xiˆ  yjˆ c .b  a.b  2  0
 1 5
 c .b  2  
2 2
SOLUTIONS 165

21. Equation of the plane containing L1 and Taking dot product by a, b , c on


x  2 y 1 z 1 b  2c  ka , we have
parallel to L2 is 1 0 2 0 a .b  2  a . c   k  a . a 
1 1 1 1 1
 a .b   k  a .b  k  ...  2 
2 2
i.e., 2 x  3 y  z  2  0
k2 k
This plane meets the axes at Similarly b . c  8   ...  3 and
2 4
 2 
A 1,0,0 , B  0, ,0  & C  0,0, 2 b . c  2  k  a . c  ….(4)
 3 
 Volume of tetrahedron From (2), (3) and (4) we get
1 2 k2 k k
OABC  abc  8  2  k  
6 9 2 4 4
22. Sum of 3 equations identically zero. They  k  3,  4
intersect in a line. 28. r  b   c = 1    iˆ   2    ˆj  1  2  kˆ
23. Lines lies in the same plane  These are
coplanar r .a 2
Projection of r on a is 
2 3  a 3
3 2 3 0 4 2 1      2     1  2  2
 
1 1 1 6 3
 sin1  sin 4    4    1 2
x  3 y  2 z 1    1,  3
24. Any point on   is
2 3 4  r  2iˆ  3 ˆj  3kˆ,  2iˆ  ˆj  5kˆ
P  3  2t, 2  3t,1  4t  lies on 29. No solution
x2 y 3 z 2
30. Intersection of line with both the planes are
   t 1 the same
3 2 3
3 6
 p   5, 5, 5   2
 2
3  6 1  2   3 6  6 1  2    6
25. Equation of the plane containing the lines is 2 2
x  6 y  5 z  10 if θ is the angle between  2    1  3   2   0
x  6 y  5 z  10 and 4 x  y  2 z  0 then    2,   1
cos  0 31. Solving the equation we get
26. AB  BC  AC  
r  iˆ  ˆj  kˆ   iˆ  ˆj  2 kˆ ,   R
2u u v u v 32. a1 x  b1 y  c1 z  d1  0 ,
AB     
u u v u v
a2 x  b2 y  c2 z  d 2  0 , &
u v u v  a3 x  b3 y  c3 z  d 3  0
AB.BC     .     1  1  0
u v u v  form a triangular prism if
 B  90 a1 b1 c1
 1  cos 2A  cos 2B  cos 2C  0   a2 b2 c2  0
 1 1 a3 b3 c3
27. a . c  a c cos  cos 1   a . c  ... 1
 4 4 And none of 1, 2 , 3 is zero
SOLUTIONS 166

33. Observe that the line L1 , L2 and L3 are and pˆ . q  4 x  6 y  14 …. (4)


parallel to the vector (1, −1, −1) From (3) and (4) we get
Also,  0 1 and bc x 2  y 2  4 x  6 y  13  0
1 2 b2c1  0
2 2
 The three planes intersect in a line  x  2    y  3  0  x  2 and y  3
34. Distance between  2t, 3t 1, t 1 is 11 38. Any plane through the second line is
yields two values of t, gives a, c  3x  y  2 z  4     2 x  y  z  1  0
35. Let d  x  b  c   y  c  a   z  a  b  or  3  2  x +    1 y +    2  z +  4     0
Taking dot product with a , b , c one by one If this plane is parallel to the first ine its
normal must be at right angle to first line.
a . d  x  a b c   0  0   3  2  2     1 4     2   0
a .d   0
 x
 a b c   Equation of plane through the second line
b .d c .d and parallel to the first line is
Similarly y  , z 3x  y  2 z  4  0 1
 a b c   a b c 
Now the required shortest distance s =
d
 a . d  b  c    b . d   c  a    c . d  a  b  perpendicular distance of a point (1, 3, –2) on
 a b c  the first line to the plane.

 a . d  b  c    b . d   c  a    c . d  a  b   1
39. 3iˆ  4 ˆj  2kˆ    4iˆ  2 ˆj  3kˆ  = 8iˆ  ˆj  10kˆ
 is parallel to the required plane
 a b c 
D.R s of the normal to the plane is given by

x2  x1 y2  y1 z2  z1
    
2iˆ  3 ˆj  4kˆ  8iˆ  ˆj  10kˆ  26 iˆ  2 ˆj  kˆ 
Equation of the plane is
36. x3  x1 y3  y1 z3  z1  0
 x  0  2  y  0 1 z  0  0
x4  x1 y4  y1 z4  z1
97
1 1 1 40. Shortest distance =
13 6
 1 0 2  0
2 2  9
 
41. aˆ  cˆ  cˆ  bˆ  aˆ  bˆ  cˆ  0

 9 aˆ  bˆ  t1cˆ for some scalar t1


 1    7 similarly bˆ  aˆ  aˆ  cˆ
2
37.  pˆ  q   pˆ   pˆ . q  q = x 2
 y2  q +  bˆ  cˆ  t2aˆ for some scalar t2
14  4 x  6 y  pˆ Hence cˆ 1  t1    t2  1 aˆ
 pˆ . pˆ  q   q . pˆ  pˆ +  pˆ . q  q =  x 2  y 2  q + either ĉ is collinaer with
aˆ or 1 t1 1 t2  0
14  4 x  6 y  pˆ
ĉ cannot be collinear with â
Since p̂ and q are non-zero non-collinear,
aˆ  bˆ  cˆ  0
 pˆ . pˆ  pˆ . q  q   x 2  y 2  q …. (1) 2
aˆ  bˆ  cˆ  0
  q . pˆ  pˆ  14  4 x  6 y  pˆ …. (2)
 1  pˆ . q  x 2  y 2 …. (3) or 1  1  1  2  cos   cos   cos    0
t  6
SOLUTIONS 169


61. Volume

Vector AM is along AD  MD  MD  2 3 2
2
 
 AD  MD MD  MD AD   AB AC AD   6 4 8  138
  
 245 5iˆ  3 ˆj  8kˆ  98 13iˆ  12 ˆj  18kˆ  0 2 5

 49  25iˆ  15 ˆj  40kˆ  26iˆ  24 ˆj  36kˆ  62. Clearly, b makes an angles of 120 with a ,
 
 49 iˆ  9 ˆj  4kˆ  also as it is coplanar with a and k̂
 it makes 30  with Z axis
Now DR’s of AC & AB
Clearly, b . c  cos 30 and
5iˆ  3 j  8kˆ iˆ  9 ˆj  4kˆ max
MD  AM  
7 2 7 2 b .c  cos150
min

 6,12, 4 or 4, 6,12
1 3 ˆ
57. Since we know that volume of tetrahedron  
63. b   b . aˆ  aˆ  b . kˆ kˆ   a 
2 2
k
1 Now, b  c  a  c  c  b and
 abd sin 
6
1 c a  a.c  0
  3 4  6  sin 30  6 cubic unit
6 a b
 c 
a b
 as c  1
58. No solution
 1 ˆ 1 ˆ
  i j
x7 y6 z 2  2 2 
59. L1    and
3 2 4  16  p 4  q 4  r 4   8
x5 y 3 z 4
L2   
2 1 3
1 3 ˆ
 C  3  7, 2  6, 4  2  and    
64. b  b . aˆ aˆ  b . kˆ kˆ   a 
2 2
k
D  2  5,   3, 3  4 
2  3  2  3  2   2  4  3 65. The vector equation of the plane passing
So,     
2 2 1 through points a , b , c is
        
   2,   1  
r. a  b  b  c  c  a   a , b , c 
So, C 1, 10, 10  and D  7, 4, 7   CD  9 Therefore, the length of the perpendicular
from the origin to this plane is given by
60. Equation of plane parallel to L1 and   
a, b , c 
 
containing L 2 is given by      
a b  b c  c  a
a  x  5   b  y  3  c  z  4   0
 2 a  b  3c  0 and 3a  2b  4c  0 66. Let (t, 0, 0) be a point on the x-axis through
a b c which a straight line L is drawn perpendicular
  
2 17 7 to the x-axis and intersecting both the lines
So, the required plane is 2 x  17 y  7 z  33 L1 , L2 D.R’s of L may be taken as (0, 1, λ)
SOLUTIONS 171

a .a a .c a .r  13 137 
2
or  , 
a c r  c .a c .c c .r  25 5 
r.a r .c r .r
77. Let (t, 0, 0) be a point on the x-axis through
1 0 0 which a straight line L is drawn perpendicular
3 3 9 to the x-axis and intersecting both the lines
0 9  L1 , L2 D.R’s of L may be taken as (0, 1, λ)
2 4
3 3 5  t 3 15
0 1 L and L1 are coplanar  1 5 2  0
2
2 0 1 
3 3 2 27
 a  c  r  a  t 1 6
2 4
L and L1 are coplanar  2 5 3  0
74. Unit vectors along AB, AD, AC are 0 1 
i  j i  j  k i  k  3 
, , Solving we get   , t    , 2 
2 3 2  4 
 13 137 
75. c  a  c  b  c .a  b .a or  , 
Now,  25 5 
b  c  b    a  c   b    b .c  a   b .c  c BN BN 
2 78.   
  a b c    b .c    b .a  BC
 NC 1  
 
 AN  1    b   c
And b .c    a b c   1
OP AP 1  
  a b c     a b c   1   b .a 
2
    AP  1    c
CA PC 
2
   
 2  a b c   1   a .b   1 AN  BP  CM  O
1 79. The number of planes which have three
  a b c  
2 points on one side & the fourth point on the
other side = 4
76. Let (t, 0, 0) be a point on the x-axis through The number of planes which have two points
which a straight line L is drawn perpendicular on each side of the plane = 3
to the x-axis and intersecting both the lines
L1 , L2 D.R’s of L may be taken as (0, 1, λ) 80.
5  t 3 15
L and L1 are coplanar  1 5 2  0
0 1 
t 1 6
L and L1 are coplanar  2 5 3  0
0 1 
 3 
Solving we get   , t    , 2 
 4 
SOLUTIONS 174

Limits, Continuity and Differentiability (LCD)


(Solutions)

1. f  x 
1 5 g 1  5  g 1  5  2  g 1  2
2 x 1
2 x Putting y  , we have
 f  f  x   x
3  2x 1 1
3  2x g  x g    g  x  g  

 f f  f  x    4  3x
x  x
Hence ponts of discontinutity are Hence g  x   x  1 or  x n  1
n

3 4 (by comparing the coefficient of )


x  2, ,
2 3 If g  x   x n  1 , then g 1  1  1  2 and
x2 x2 g  2   2n  1  5  n  2
2.  (Using
sin x tan x  x3   x3  So, g  x   x 2  1 and if g  x    x n  1 , then
 x  6  x  3 
   g 1  0 , which is not true
expansion)
1 So, g  x   x 2  1

1 2  lim g  x   10
1  x  ... x 3
6
5. No Solution
 x2  n
 1
 lim  0
x  0 sin x tan x  6. S  lim  
  n 
r 1 n  r 
3. Using L’Hospital’s rule, we have sin   
 4 4n 
et f  x   e x f   t  1
dx
lim 2
2 
tx
 f  x  0  
sin  x  
ex f  x   ex f   x  4 4
 2
2 1
 f  x  4      
   ln  tan  x    
   8 8  0
d  ex 
  2      
dx  f  x    4 ln1  ln  tan    
   8  
ex
  2x  c
f  x  4 ln  2 1 
ex 7. g  x   sgn  f  x   has discontinuty points, at
 f  x 
2x  c the points where
1 1 1 5
 f  0    c  2  sin x 
c 2 2
ex 1 5
 f  x   sin x  (not possible)
2 1  x  2
4. g  x  g  y   g  x   g  y   g  xy   2 1 5
 sin x 
Putting x  2, y  1, we have 2
SOLUTIONS 176

1 21. No solution
sin
17. lim x  1
x  1 22. Let x  K sin 
x 1  1  4 K cos  
  lim 2  2 
 0 K
 sin  cos  
1 1 2
x  2 x 2 sin 0  is finite K    
18. f   0   lim x 4 k
x0 x0
 1
 lim  1  2 x sin   1 23. Given that f  2 x    e x  1 f  x  and
x0
 x
It is not continuous at x  0 f  0  0
 1    e x  1 f  2 x    e2 x  1 f  x 
But f     0 and
 2n  f  2x  f  x
 
 2  e2 x  1 e x  1
f     0  n 
  4n  1   x  x  x 
f  f  f n
f  x 2 4 2
 x  x   x   ...  x 
19. Observe that f  0   0 and e 1 2 n
e 2 1 e 2 1 e 2 1
f  h  x 
f   0   lim 1 f  x
f n
f  h
2
h  lim x   lim h
h 0
 x
f  x  h  f  x e 1 n 
n
h  0 e 1
f   x   lim e 2 1
h0 h
f h f  h  f  0
f  h   x h  xh2
2  lim  lim  f   0
 lim  x2  1 h 0 h h  0 h
h 0 h f  h
x3  f   0   lim
h0 e h 1
Hence, f  x    x
3

x x
24. Given that f  2 x    e x  1 f  x  and
sin x  ae  be  c ln 1  x 
20. L  lim f  0  0
x0 x3
 x  3
 x x x   2
x x 3
x   x x  2 3 2 3
  e x  1 f  2 x    e x  1 e x  1 f  x 
 x    a 1      b 1      c  x   
 3!   1! 2! 3!   1! 2! 3!   2 3
 lim f 2x f  x
x0 x3  
2x
e 1 ex 1

 a  b   1  a  b  c  x  
a b c 2  1 a b c 3 x x  x 
   x      x f  f  f n
2 2 2  3! 3! 3! 3  f  x 2 4 2
 x   x   ...  x 
 lim
x 0 x 3
 x
a b c e 1 2 n
 a  b  0, 1  a  b  c  0,   0 e 2 1 e2 1 e 2 1
2 2 2  x 
f n
f  x  2   lim f  h 
1 a b c  x  lim
and L      e  1 n 
e2
x
n
h  0 eh  1

3! 3! 3! 3
f  h f  h   f 0
Solving first three equations, we get  lim  lim  f   0
h 0 h h0 h
1 1
c  0, a   , b   f  x   f   0   e x  1
2 2
 abc  0
SOLUTIONS 177

1 1 1
dy
 f  x  x
lim    lim 
 ex 1  x

 ex 1  x  x
 lim 1  
 3  x 2  6 x  8  3  x  4 
x0
 x  x 0
 x  x0
 x  dx
1
 1  For real root of f  x   x3  9 x 2  24 x  a,
 e 2  f    1
2
 f  2 f  4  0
1 1
25. Given f 1  x    f  f  x   2
f  x x g  x    f   x    f   x  f   x 
31. d
1 1   f   x  f   x  
 f 1  x   f    f  x  f   1
x  x dx
2 The minimum number of zeros of g  x  in
 f 1  1  f 1  1 as f 1  0
 0, 6 is one less than the minimum number
26. If f  x  is continuous then being bijective it of zeros of f   x  f   x  in  0, 6 .
will be monotonic. First, let’s count the number of zeros of
 f  f  x   would be increasing but f  x .
1 f  x  f 6  x  f  x   f  6  x
f  f  x  is decreasing.
x Thus, the roots of f  x are
Thus, it contradicts the fact of f  x  being
x  0, 1, 2, 3, 4, 5, 6 .
continuous.
Since f   x  has 7 roots, so minimum
  5  number of roots of f   x  is 6.
f  x   cos 1  cos   x
  2  Hence, minimum number of roots of
27. f   x  f   x  are 13 in  0, 6 .
  3 
 sin 1  sin   x    4  2 x So, the minimum number of roots of
  2 
d
g  x    f   x  f   x   is 12 in  0, 6  .
dx
sin x  tan x tan x  sin x 1
28. lim 3
 lim 3

x  2
4x  2x x0
2x 16 1 1

32. L  lim
1  3x  2 x 2  x  1  3x  2 x2  x
1  2 x0 x
29. Area      1
2 2 4
 lim
1  3x  2 x  2 x
 e3
x 0 x
30. (A) Let f  x   x  cos x  a 1

 f   x   1  sin x  0x 
 lim
1  3x  2 x2  x  e3
Thus, f  x  is increasing in  ,  
x 0 x
 L1  L2
For positive root, f  0   1  a  0 1

(B) f   x    sin x  a  0 2
L1  lim
1  3x  2 x2  x  e3
2 2
x0 x
 a  sin xx   a  1 
ln 1 3 x  2 x 2 
(C) Let y  x 3  9 x 2  24 x  a e x
 e3 5
 lim   e3
 x  x 2  9 x  24   a x 2
x0
SOLUTIONS 178

Similarly, L 2  
13 3
e  f  x  is even function f  x  is not
2 2
differentiable at xn  : n
 L  4e3 2n  1
33. As x  0,  f  x    1 and  f  x   0 38.
So the given limit is in the form 1  x m f  x   h  x   3 
lim g  x   lim  lim 
34. Equal chords subtends equal angles at the x 1 m  x 1
 2 x m  4 x  1 
centre of a circle, if each of sides of length
 h  x  3 
K(K = 1, 2, 3) subtends an angle  k at the  f  x  
 lim  lim xm 
given circle then m  m 1
 4 x  1 
21  2 2  2 3  360 2 m
 x 
1  2 3
 
 90 f 1
2 2 2  g 1   f 1  2 g 1
2
   
 cos  1  2   sin 3 h 1  3
 2 2  2 Similarly, g 1  lim g  x  
x 1 5
1 2 1  
cos cos  sin sin 2  sin 3 ... 1  h 1  5 g 1  3
2 2 2 2 2
  1   1   3 Now, f 1  h 1  3 g 1  3  3 1  g 1 
sin  1   , sin  2   , sin  3  
 2  2r  2  r  2  2r  a  3, b  1, a  b  4
 2 2
4r  1 r  1  3r  1 39. sin x cos y  f  2 x  2 y   f  2 x  2 y  
r  2 r 3  7 r  3  0  cos x sin y  f  2 x  2 y   f  2 x  2 y  
 r  0 we have 2r 3  7 r  3  0 f  2x  2 y  sin  x  y 
 
35. No solution f 2x  2 y sin  x  y 
1  x, 0  x  2 f   f  
36. f  x      k
 x  3, 2  x  4  
sin sin
 2  x,  1  x  0 2 2
g  x   x
 2  x, 0  x  3  f  x   k sin  4 f   x   f  x   0
2
1  g  x  , 0  g  x   2
 fog  x    1 1

1  x
3
2 2 
 
 g  x  3 , 2  g  x   4 
40. I n   x n 1 x 1  x 2 dx   x n 1 

3  dx
1  x, 1  x  0 0 0  
  
  1  x, 0  x  2 1
1 3

37. f  x  is obviously ifferentiable at x  0 and 


   n  1 x n  2 1  x 2 2 dx
30

1
xn  where n  0, 2, 4,...  3I n   n  1  I n  2  I n 
2n  1
In n 1
We get f   xn    and f   xx    and or   1 as n  
In  2 n  2
2
xn  where n  1, 3, 5,... 41. Hence, the continuity must be checked at
2n  1
x  1, 2, e, 3, 10, 11, 12, 3, 5
We get f   xn    and f   xn    and
Further, f 1  0 and
SOLUTIONS 179

lim f  x   lim sgn  x  2    log e x   0 9


1 e   1 1 
x 1  x 1      
Hence, f  x  is continuous at x  1 2     r  r  1  r  1 r  2  
n
lim f  x   lim sgn  x  2    log e x   lim  A r A r 1 A r  2
x2 x2 n 
r 1
  1  0  0 9 9
1 e  1 1 e 
lim f  x   lim sgn  x  2    log e x       
x2  x2 2    1.2 4   
r
  1  0  0 
1  
  r
3 3
 3 
Ar
e e e
 ee   k r
Hence, f  x  is continuous at x  2  
 
lim f  x   lim sgn  x  2    log e x   1 3 1
x3  x 3 n
1 1
 3 
 lim  k  r    k  1   e e  1
 lim f  x   lim  x 2   0 n 
r 1 k 1  
x 3 x 3  
x2 3
e 1
x sin f  x
2
 2
   2 x   sin f  t  dt  2 x 45. A r   
  r
1
42. lim
x 1 2  x  1  A1 A 2 A 3 ... are in HP
Applying L – H rule again. We will get 9
e 1
2 f  1  4  A r A r 1 A r  2   
   r  r  1 r  2 
3
e 1 9
1 e   1 1 
43. Ar        
  r 
2     r  r  1  r  1 r  2  
 A1 A 2 A 3 ... are in HP n

e 1
9  lim  A r A r 1 A r  2
 A r A r 1 A r  2    n 
r 1
   r  r  1 r  2 
9 9 9
1 e 
  
 1

1  1 e  1 1 e 
2 
 
 r  r  1  r  1 r  2  
     
2    1.2 4   
n
3 r
 lim  A r A r 1 A r  2 
1
Ar
 
  r
3
 3 
n 
r 1 e e e
 ee   k r
 
1 e  1 1 e 
9 9  
      3 1
2    1.2 4    n
1 r 1
 3 
r  lim  k    k  1   e e  1

1  
  r
3
 3
3
 n 
r 1 k 1  
Ar  
e e e
 ee   k r
  x
  
1
ln f  x  2 1
n  3 
3
46. lim  lim 
r 1 1 x  0 ln g  x  x0 x 2
 lim  k    k  1   e e  1
n  k 1  
r 1
  47. Domain h  x  is  0, 1
3
e 1   ln l  
44. Ar    48. h  x   tan 1  ln  2   0  x  1
  r   x 
 A1 A 2 A 3 ... are in HP 1 1
1  2    0  ln 2  
9 x x
e 1
 A r A r 1 A r  2   
   r  r  1 r  2 
SOLUTIONS 181

Functions, Inverse Trigonometric Functions(ITF)


n
g  x   x m  x  1
1. cos 1 x   0  x   1,1 and
m
f  x  0  x 
cot 1 x   0  x  R mn
Hence cos 1 x    cot 1 x   0  m  mm nn
f  mn
 m  n  m  n
  cos 1 x    cot 1 x   0
If cos 1 x   0 6.  x   x   2 x  2 x  ...  12 x  12 x  0
 x   cos1,1   x    2 x   ...  12 x   0
1
If cot x   0  0  12 x  1
 x   cot1,   1 1 1
0 x  x
 x   cot1,1 12 25 10
1 1
 x
2. Given, f  x   4 x 4  ax 3  bx 2  cx  5 25 12
 total values of x are 13
a
r  4
1
7. f  f  x    6  f  x   2  or  f  x   1
b c 5
 r1r2  4 ,  r1r2 r3  4 , r1r2r3r4  4 f  x   2, has two roots and f  x   1 has
r r r r four roots
Consider, 1 , 2 , 3 , 4
2 4 5 8
3 2
Clearly, AM = GM 8. f     0, sin  
, cos    gives
r r r r 13 13
 1 2  3 4 maximum value of f    3456
2 4 5 8
1 a/4
 
4 19 10 n  1
9. x 2  bn x  n
 a  19 9
2 10n  9n  1
3. 1  5C2  5C4 .3  x  bn x  0
9
Two cases arise 10n  9n  1
f     Product of roots  
Case 1: 9
1 n
Case 2:  
f     C0 9  C1 9  C2 9n2  ...  nCn 2 92 
n n n 1 n

9
 f     Which is divisible by 9
Hence these equations can have 9 as a
4. α can be selected from where sin x is cut by common root
line parallel to y-axis at more than one point But 9 is not the root for all the equation as αβ
has other divisors also for example n = 4
5. Consider,    4C0 93  4C1 9 2  4C2 9  9  123
On equation has 123 as a root
SOLUTIONS 182

10. Conceptual 8
x ...  iii 
3
1 1 1
11. f   x   sin  cos  0 as x   8 
x x x  x  ,4
3 
1 1
f   x    3 sin  0 for all Hence complete solution is
x x
8 
 1  x   ,   m  n  8  3  11
x  1  0  k  1   3 
 x 2
 f   x  is strictly decreasing on 1,   14.
 x  y  z   x  y    y  z    z  x  
2 2 2
 f   x   0 for all x  1 Applying Lagranges
MVT to f on  x, x  1 for x  1  1 8 Rejected
We observe that f  x  1  f  x   f   c  for  8 1 Selected

8 
same c in  x, x  1 2  4 Rejected
 f  x  1  f  x  which is true for all x in 4  2 Selected
x  1, y  1, z  1
1,  
 x yz 3
12. Possible values of x  y  z  4 or 8
f  0   0  a1 cos 1  a2 cos  2  ...  an cos  n  0 x yz 4
Possible triplets (2, 1, 1)(1, 2, 1)(1, 1, 2) by
f  x1   0   a1 cos 1  a2 cos  2  ...  an cos  n  cos x1
checking these triplet satisfy the above
  a1 sin 1  a2 sin  2  ...  an sin  n  sin x1  0 equation
 a1 sin 1  a2 sin  2  ...  an sin  n  0   x1  n  If x  y  z  8,
2 2 2
 a1 cos 1  a2 cos  2  ...  an cos  n  0  x  y   y  z   z  x 1 is not
and a1 sin 1  a2 sin  2  ...  an sin  n  0 possible as if x, y, z are integers not all equal
then
 f  x  0  x
3/ 2 3/ 2

15. f  n  
 2n  1   2n  1
13. Case-I:
2
4x  0
Hence
x4 3/2 3/ 2

f 1  f  2   ....  f  40  
81  1
 364
x   4,  )
2
Case-II:
4 x  0 x0
x   , 4  ...  i  16.
 f  2   2 f  0   f 1  1
2
x  2x  0 Similarly, x = 1
 x   ,0   2,   ....  ii   f  3  5
SBS  x 2  2 x  16  8x  x 2
SOLUTIONS 183

x2  1  x  2
 f  4   3 and x  3  2  2 x  4
 f  5   13  22  32  2  a  2 x  a  4  a
b   2  a  2
   1 and 4  a  4, i.e. a = 0
a
b can take any value
17.  RHS is always odd
While LHS is always even 22. f1  x   x
 f11  f1
18. Let f  x   p  x   10 x So f1 OF  f 4
 f 1  f  2   f  3  0  F  f11 O f 4  f1 O f 4
 p  x    x  1 x  2  x  3 x     10 x  F  f4
p 12   11 10  9 12     120
23. FOG  f5
p  8    9    10    11 8     80
 f 4OG  f5
 p 12    8   11 10  9 12    8     40
G  f 4 1Of5 , f 4 1  f5
 11 10  9  20  40
G  f5 O f 5  f 4
 40  496
p 12   p  8  24. f3 OHO f 2  f 4
or  1984
10  H  f 31O f 4O f 2 1  f 3 O f 4O f 2  f 4
p 12   p  8 
  1980  4
10 x  x
25. f  x   2 f    f    x2
2 4
19. Let x  3cos  , y  3sin  2
z  2 cos  , and t  2sin  x  x x x
 f  2f   f   
 6 cos  sin   6sin  cos   6 2 4 8 2
.....................
 sin      1
.....................
     90    90   .....................
Therefore, x  3cos  , y  3sin  and 2
 x   x   x   x 
z  2sin  , t  2 cos  f  n   2 f  n 1   f  n  2    n 
P  xz  6sin  cos   3sin 2 2  2  2  2 
Adding and applying n  
Hence, Pmax  3
2
 x  4x
f  x  f   
20. g  f  x   is not defined if (i) 2  a  8 and 2 3
(ii) b  3  8 Repeating same procedure
a  10 and b  5 16 x 2
We get f  x   f  0  
9
21. x   1, 2  f  3  f  0   16

26. Clearly f  x   f  0   x
SOLUTIONS 184

Two solution.  y0

2
 y  y2  1  1
16 x
27. f  x   f  0    10 z  y  1
9
 16  zy
f   x     2x i.e. 0  x  y  z
9
z0
 f  0  0
 z  z2 1  1
28. (A) f  x   x sin x  10 x  z  1
f   x   f  x   f  x  is even xz

At x  0 and  , f  x   0 i.e. 0  x  y  z  x
a contradiction hence not greater than 0
 f  x  is non-invertible If x < 0 we obtain
Range of x sin x is R x  z  y  x  0, a contradiction hence x is
 unbounded not less than 0
(B) Only one solution
f  x   x  tan 1 x
31. For log to defined x  3 and  x  0
f x   0  increasing
  0, x  0
also    f  x    
  0, x  0
x    f  x   
Therefore we will consider two cases
 range is R  f  x  is invertible  i  when   0 and x  0
(C) f  x   cos x is periodic with period 2π
 ii  when   0 and  3  x  0
also range of f(x) is [0,1) 2
 bounded Equation is equivalent to  x  3    x
(D) f  x   tan x When   0 , for only one solution
2
 range [0,1) and periodic with period π Discriminant of  x  3   x must be zero
   12
29. a 2  b 2  c 2  ab  bc  ca When   12
2
 3  a2  b2  c2    a  b  c   d 2 When   0  only one solution
a2  b2  c2 1     , 0   12
 
d2 3
32. Period of f  x  is 2 , but f  x  is not
30. Clearly x = y = z = 0 is a solution   3 
defined for x   ,

If x > 0  x  x 2  1  1  2 2 

 10 y  x  1   
Hence it suffices to consider x    , 
yx  2 2
Further since f  x  is even, we consider
i.e. 0  x  y
 
x  0, 
 2
SOLUTIONS 187

5 5
y  tan   7 and z  tan   3 7  a5
9 26
If  x0 , y0 , z0  is a solution then
Sn  

1

1
  x0 ,  y0 ,  z0  is also the solution and r 1 r  r  1 r  2  r  3 18
 0, 0, 0  also a solution. Matrix Matching :1
 Three solutions are possible.
44.  a  5 x2  5bx   a  b   0 has real roots
42. Let x  tan   25b 2  4  a  5  b  4  a 5  5a   0 b
y  tan 
 2ba 2  135a  25  0

z  tan  and 0   ,  ,    5 
2   a    a  5  0
sin  sin  sin   26 
  
4 5 6 5
4
  a  5, but a  5,
Let the side of  be 4a, 5a, 6a 26
15a  5  b 
 s x , is not true
2 5b
5a 3a for all b as x is not defined when b  0

tan
  s  5a  s  6a   2 2  1 
5
a5
2 s  s  4a  15a 7 a
 7 26

2 2 1 1
Sn   
 1 r 1 r  r  1 r  2  r  3  18
2 tan 2
x  tan   2  7 2 7 7 Matrix Matching :1
2  1 6 3
1  tan 1
2 7
5
45.  a  5 x2  5bx   a  b   0 has real roots
y  tan   7 and z  tan   3 7  25b 2  4  a  5  b  4  a 2  5a   0 b
9
If  x0 , y0 , z0  is a solution then  26a 2  135a  25  0
  x0 ,  y0 ,  z0  is also the solution and  5 
  a    a  5  0
 0, 0, 0  also a solution.  26 
 Three solutions are possible. 5
  a  5, but a  5,
26
 5  b
43.  a  5 x 2  5bx   a  b   0 has real roots x , is not true
5b
 25b 2  4  a  5  b  4  a 5  5a   0 b for all b as x is not defined when b  0
 2ba 2  135a  25  0 5
 a5
 5  26
  a    a  5  0 
 26  1 1

r 1 r  r  1 r  2  r  3 

18
5  5  b 
  a  5, but a  5, x  , is not
26 5b Matrix Matching :1
true
for all b as x is not defined when b  0
SOLUTIONS 188

46. (A) The given equation indicates that any


solution x must be an integer
Let x  6q  r , where r = 0; 1; 2; 3; 4; 5 and q
is an integer.
Then the given equation becomes
 r   2r 
q   r
2  3 
(i) r = 0 gives q = 0, so x = 0 is a solution
(ii) r = 1 gives q = 1, so x = 7 is a solution
(iii) r = 2 gives q = 0, so x = 2 is a solution
(iv) r = 3 gives q = 0, so x = 3 is a solution
(v) r = 4 gives q = 0, so x = 4 is a solution
(vi) r = 5 gives q = 0, so x = 5 is a solution
Thus, there are total of 6 real solutions
a
(B) x  2  2 for x  0
x
a 1/3
x  2 attains minimum value at x   2a 
x
a
 x 2 2
x x  2 a 1/3
32
a
27
a  2
2
(C) a 2  b,  a  4   b  32 when a > 0
 a 2  8a  16  a 2  32
 a  2 i.e. f :  2, 6   4,36
2
or a 2  b  32,  a  4   b when a < 0
 a 2  8a  16  a 2  32
 a  6 i.e. f :  6, 2   4,36
Maximum value of a  6
2
(D)  2   2       2
2
  k  2   2  k 2  3k  5 
 k 2  4k  4  2k 2  6k  10
 k 2  10k  6    k 2  10k  6 
2
   k  5  19
Max  2   2   19
SOLUTIONS 190

 y  1  1 x  2   3  21 
y  x3 ....(2) a  4, 
 2 
Now Equation (1) and (2) meet at a point
Case-2:
 x  3  a2 x2  5ax  4
a1, 
2 2
 a x   5a 1 x 1  0
a4
Let x1, x2 be the two values of x 1
1 a
Sum of roots  x1  x2    1 2 5 a ...(3)
a a  4  1  a  (always true)
Also given, the chord is bisected at x = 2 Thus a1, 
x  x2
 1 2
2  3  21 
5a  1  a  4,   1, 
 2  2 
2a 2
11. Let a  b  c  d  x
 5a  1  4 a 2  e f  g  h  8 x
 4 a 2  5a  1  0 Now, let y  x 2   8  x 
2

1  y  2 x 2  16 x  64
 a  1,
4
y  2  x 2  8 x  32 
 a4  5
10. Given, f  x   1 x  3x  log5 2
y  2   x  4   16 
 1 a   
For decreasing function Minimum value of y is 34 as x can be 4
but it can take value 3 or 5 for given a, b,..., h
f   x  0  x  R
 p2 1  3
 a4  4 12. Given, f  x    2  x  3 x  log 2
 4  1 x  3  0  p 1
 1  a  For decreasing function, f   x   0
a4 3  p2 1  2
1  4  3 2 x 3 0
1 a 4x  p 1 
a4  p2 1  1
 1  0  2
1 a  2
 p 1 x
Here a  4  0
a  4 p2  1
 0
Now p2 1
Case-I: a 4,1  p2 1  0
a4 p2  1
1 1  p  1
1 a
a  4  1  a 
2
p   1,1
a  4  1  a 2  2a 13. Since AM  GM
2
a  3a  3  0
3  21 3  21
a or a 
2 2
Taking common
SOLUTIONS 192

21. P  x   a  x 1 x  2 x  3 x  b 1 2  3 3 2 3


 f   
 P  7   0 2 6 6
 a  20  7  b   24  7  b   30  7  b   120  0 25. 1  sin x  cos x  2
 140  168  210  120  20b  24b  30b   sin x  cos x   1
319 P  2, 1 and Q  2, 1 are the point of
b
37 intersections, angle at p is tan 1 2
22. Given, f  x   1
26. cos x  sin 1  sin x  by graph there exists two
3 x   9  x   1
Let t  3 
x solutions
    
1  t  3x  One in   ,  and the other in  ,  
 f  x     2 2 2 
t 2  t 1  1  t  3 2 2
27. x  4 y  4
Consider, y t2 t 1 x2
  y2  1
Its vertex is at t  1 4
2 Let x  2 cos 
 t 2  t  1  1 at t  1  y  sin 
max
2
Now
& t  t  1  5 at t  3 x 2  y 2  xy  4 cos2   sin 2   2sin  cos 
min

  5  t 2  t  1  1 1
 2 1  cos 2   1  cos 2   sin 2
1  1 2
   ,    1,  
2
t  t  1  5 3 5
 cos 2  sin 2 
1
23. Given g  x  f  x
2 2
5  13
 f  g  x   x Hence maximum value is and
2
Differentiating w.r.t x we get
5  13
f   g  x   g  x   1 minimum value is
2
1 1 x
 g x   6
f   g  x   cos 2  g  x   30. f  x     x   x  t   sin  x  t  dt
0
 g   x   sec 2  g  x   x
  t 6  sin x cos t  cos x sin t  dt
24. Put x  5 and 2  0
6 3 x x
then  sin x  t cos t dt  cos x  t 6 sin t dt
6

1  3 1 0 0
2 f    2 f    ....1 31. g   x   f   tan x  sec2 x  f   cot x  cosec2 x
2  2  3
and  f   x  0  f   x is decreasing
 3 1   3 
2 f    2 f    3 ....  2 tan x  cot x  x   , 
2 4 
 2   2
 f   tan x   f   cot x  &
(1) and (2)
  3 
sec 2 x  cosec 2 x  x   , 
2 4 
SOLUTIONS 193

  3   
 g  x  is increasing in  ,   f   x   0 for all x   0,  and f   x   0
2 4   4
 3    
g(x) is decreasing in  ,   and g(x) has for x   , 
 4  4 2
3  
local maximum at x  Since, f  0   0  f   , applying Rolle’s
4 2
32. dy  3t  
dx theorem to ‘f’ on  0, 
Equation of tangent at ‘t’ is  2
 y  4t   3t  x  2t 2  1
3
... 1 We observe that f   c   0 for atleast one ‘c’
This is a normal at another point ‘t1’  
in  0,

 Slope of normal at  2t1  1, 4t1   3t
2 3
 2
1 Also, 1  sin 5 x  cos 5 x  sin 2 x  cos 2 x  1
i.e.,  3t ....  2   
3t1 for x  0,  holds only if
&  2t12  1, 4t13  lies on (1)  2
sin x  sin 2 x and cos 5 x  cos 2 x
5

t
 t1  ....  3   
2 i.e., only if x  0, 
 2
2
From (2) & (3), t   1
3 36. f   x    1  sin x for x  0
x
 Required straight lines are
 f  is continuous on  0, but f  is not
31 2 31 2 3
2x  y  & 2x  y  differentiable at x  2 n  (‘n’ is a non-
27 27 2
33. Even degree polynomial with leading negative integer) so f  is not differentiable
coefficient positive will have absolute
minimum on  0,
1 1 Both f  x and f  x are positive for all
34. g   g  2 f 
f f x  1 and f   x   2  1 for x  1
 2 1  2 f 
g     3 f 2  2 f   3 f 2  2 x
 f f 
2 2 f 
f f Since, 0   1  sin t dt   
2 1 x for
0
f  2
f  g  f  f 3 f x0
   
f  g f 1
f2
f  f  x    as x  

f   2 f  f   2 f 
 
So, there exists   1 such that

f  f f f x    f  x   2  1 and this implies that
In a similar manner, we can show that the
f   x   2  1  f  x  or f   x   f  x  for x  
2g
same is equal to
g
f  x    as x  0 and
35. f   x    5sin x cos x   sin 3 x  cos 3 x 

f   x    as x  0
 f  x   f   x    as x  0
SOLUTIONS 201

Hence, f  x   ax3  bx 2  cx  d 1 2
r 
From, f  2 x   f   x  f   x  , we have 1
sin 2 2  sin 2
8ax   4bx 2  2cx  d   3ax 2  2bx  c   6ax  2b  2
2
Comparing coefficient of terms, we have Thus, rmax   2
4 2 1
a  , b  0, c  0 and d  0
9 2 2 2
rmin   
4 x3 2 1 3 3
 f  x 
9
1 f  3  12
 2 one  one and onto
4 x3 3
 3  x  x  0, 
9 2
91. Suppose degree of f  x  is n , then the degree
of f  is n  1 and degree of f  is n  2
So, n   n  1   n  2   n  3
Hence, f  x   ax3  bx 2  cx  d
From, f  2 x   f   x  f   x  , we have
8ax   4bx 2  2cx  d   3ax 2  2bx  c   6ax  2b 
Comparing coefficient of terms, we have
4
a  , b  0, c  0 and d  0
9
4 x3
 f  x 
9
1 f  3  12
 2 one  one and onto
4 x3 3
 3  x  x  0, 
9 2
92. Putting x  r cos  and y  r sin  in given
curve, we get
r 2 cos 2   r 2 cos  sin   r 2 sin 2   1
r 2 1  sin  cos    1
1
r2 
1
1  sin 2
2
1
r
1
1  sin 2
2
SOLUTIONS 202

Quadratic Equations
1. Let  be the common root 2
Case I: If  t  2   9  0
  4  2 3  8 2  6  15  0 t 5
 3  4 2    10  0  x  R   5, 5
Given equations are
Then t 2  3t  4  0
 2  3 2  2  5  0 and   t  4  t  1  0
  2   2  2  5   0  t  1 or t  4
2
 For common roots   2  5  0  4  x  4
Product of uncommon real roots is (−3)(
 x   , 5   5,  
−2) = 6
2
2. Let the expression is of the form Case II: If  t  2   9  0
 x    x    x    x    t 5
It can be a perfect square if either both  5  x  5
expression are simultaneously perfect squares
2
Then t 2  3t  4  0
of both roots common, i.e.,  k  2   8k  0   t  4  t  1  0
2k 2k
and k 2  4  2k  4   0 or  t 4
k 2k  4
 4  x  4
Solving we get k can take only value
3. (I) Clearly b 2 r 1  b 2 r  4 govern the condition  x   4, 4 
of exactly one root common between  x   , 5    4, 4    5,  
Qr 1  x   0 and Qr  x   0 1
5. f  0
Now, 2
Q0  x   x 2  b0 x  1  0  6. Let roots are  and 3 , then

Q1  x    x 2  b1 x  1  0  4  4    a and
Let 1 be its common root a2  4a2  f  a  0
Then,  f  a  3a2  a
2 2 2
1  , 2  , 3  ...... f   a   6a 1, f   a   6 , then minimum
b0  b1 b1  b2 b2  b3
and so on value of f   a   6a  1, f   a   6
 br  2  br  k b c
7.   ,   ,  ,   1, 2 
 1 ,  2 ,  3 are in H.P. a a
ar . f 1  ar  ar  br  1   1,  1, 2   , 2     0,1
r = even then ar f 1  0 Apply AM  GM
r = odd then ar f 1  0
 1   2     1 2 
  
2
4. Let x  t
  1   2   

t 2  2t  3
 1
And    1 2   
2 2
t  2 9
SOLUTIONS 204

 (c) is not possible  3  1 10  1


So,   11
Also if c  4  2 3, a  0 ,  2 3 1
 10
Hence (d) is not possible 10
2 (B) x  0  x 2  3  3
15. a 2  b 2  c 2  d 2  ab  bc  cd  d 
5  log 2  x 2  3  log 2 3
2 2 2 
 b 3 2c  2  3d   ve
 a    b    c  
 2 4 3  3 4   log 2  x  3  is always positive
2

2
5 4 1
 d   Also, x  0  x  2
8 5 x
4 3 2 1  1
d  , c ,b ,   log 1  x    log 1 2
5 5 5 5 3  x 3
c  a  2b 1  1 1
16. The roots of the equation are 1,  log 1  x    log 1 2
a  b  2c 2 x 2
3 
they are rational  
3
 ve
f  1 f  0   0   2a  b  c  c  a  2b  0  1  1
 log 1  x   is always negative
 c  a  2b  0  2a  b  c  0  2 3  x
For ax 2  2bx  c  0 Hence, there is no value of x for which
2b 1  1
Sum of the roots = 0 log 2  x 2  3  log 1  x  
a 2 3  x
For ax 2  2bx  c  0 (C)
2b 4  x2 4
Sum of the roots = 0   x
a x x
c
Product of the roots =  0 4  x2 4  x2
a  
Both roots are negative x x
2 2
  4b 2  4ac   c  a   4ac   c  a   0  4  x2  4  x2 ;x0
17. (A) Let log10 x  y  4  x2  0 ;x0
2
1 8y   x  2  x  2   0 ; x  0
 1
y  2 y2  x   2, 2  0
 1 8 y2  y  2 y2 So, integer values of x are 2,  1, 1, 2
 6 y2  y 1  0 So, number of integral values of x is 4.
 6 y2  3y  2 y 1  0 (D) log c 2 log b 125  log10 8 log c 10
  3 y  1 2 y  1  0 log c 10
 log b 125  log10 8  log10 8 log 2 10
log c 2
1 1
 y ,  log b 125  log 2 8  3
3 2
1 1  125  b 3
 log10 x  , 
3 2  b 3  53
1 1

 x  10 3 , 10 2  b5
1 1 18. a  1, b  1, a  b

   10 2 ,   10 3
SOLUTIONS 206

5  C) It a  0, then we can take any non- zero


 a   , 5  a  5 value of K and then the inequality will be
 26 
So, integral values of a are 1, 2, 3, 4. valid for all values of x .
p If a  0, then there is no value of k such that
Now, Tr    r  am 
m 1 ax  k 2 is positive for all value of x
 Tr   r  1 r  2  r  3 r  4  D) For any a  R , there exists values of x
n and k for which ax  k 2  0
Sn   Tr 34.     2 p1 ;   1
r 1
n
Sn    r  1 r  2  r  3 r  4      2 p2 ;   2
r 1
    2 p3 ;   3
Clearly Tn vanishes for n  1, 2, 3, 4
From here we get two sets of  ,  , 
Hence, required sum  1  2  3  4  10
i.e.  P1 , P2 , P3 
1
31. Putting x = 1, and
2 a1  b1  c1
2  a  b  0 and  8  a  2b  0 35. 1   1    
a1
4  2a  2b  0 and 0   a  2b  8
By solving we get 1

4  a  8 and  8  b  2 2  3 a  b  c m
1   1   1      i i i   n
ai
1 3
2  i 1 
2 
32. x  x  1   x     0  x  R
 2 4  m  2 and n  1
x 1
x 2
 x  1  1   x  1 log  x  x  1  0
2
36. Two cases arise
Case 1: Case 1: one common root
x  0,1  nosolution Case 2: d = 0
Case 2: 37. f  x   0 has real roots 1 ,  2 ,  3 , where

x  1  0  log x 2  x  1  0  1   2, 1 ,  2   1,1 and  3  1, 2 

x   ,1  x   , 0   1,   Clearly f  x   1 for three values of x say

x   , 0  x1 , x2 , x3 and hence f  f  x    0 for

33. ax  k 2  0 x  x1 , x2 , x3
A) There is no value of a so that the Similarly for  2 &  3
inequality is valid for all values of x and k
B) It a  0 , then there is no x for which
inequality is valid for k  0 . For a  0, we
can find a value of x such that inequality is
valid for any value of : k
SOLUTIONS 208

 
1   1 1  1
     S   m
....  i 
3 r 1  r  r  1 r  2   r  1 r  2  r  3  m 1 n 1  3 3n  3m
  
m nm
1 1 1 1 1 1 1  Also,
        ...   
1
3 1.2.3 2.3.4 2.3.4 3.4.5 3.4.5 4.5.6  S   n
n 1 m 1  3 3m  3n
  
1 1  1
  n mn

3  1.2.3  18  
1
 S   n ....  ii 
 a b  m 1 n 1  3 3m  3n
9. x  y  z  3    
 2  n mn
 ab (i) + (ii)
 15  3  
 2    
  
 a  b  10 .... 1  
 1  1 1 
2 S   
1 1  m 3n    3m   3n  
m 1 n 1  3
3           

1 1 1  
   
a b   m n    m   n  
x y z 2   3n 3m  
5 3  a  b  3 10     n  m 
   1  
   m
3 2ab 2ab m 1 n 1   3 3  3 3  
n m n

 ab  9 ...  2       
 m n   m n  
From (1) and (2), a = 9, b = 1 or a = 1 and  
m n
b=9    m  n 
m 1 n 1  3 3 
Hence GM  ab  3, a + 2b = 11 or 19 
m  n
10. Let g, l be the G.C.D and L.C.M then  2S    m  n 
m 1  3 n 1 3 
g  l g  l 169
g . l  192 and   Let,
2 2 gl 48 
n 1 2 3 4
2
  g  l   132.4 2 k   n   2  3  4  ... ....  iii 
n 1 3 3 3 3 3
 g  4, l  48
 a  4, b  48 or a  12, b  16 1 1 2 3
 k     .... ....  iv 
11. D1 : b 2  4ac  0 3 32 33 34
D2 : c 2  4ab  0 (iii) – (iv)
1
D3 : a 2  4bc  0 2 1 1 1 1 1
k   2  3  4  ...  3 
D1  D2  D3 : a 2  b 2  c 2  4  ab  bc  ac  3 3 3 3 3 1 2
1
a2  b2  c 2 3
1 4 3
ab  bc  ac k
 
m2 n 4
12. Let S   
 m 3 3   m  3 3 9
m 1 n 1  n.3  m.3  3
m n m
So, 2S    m      m    
m 1  3 4  4 m 1  3  4 4 16
SOLUTIONS 209

9 1 n  1 1 
S     
32 2 r 1   r  1 r  2   r  2  r  3 
 p  9, q  32
2 11 1 
13. a1  1, an12  1      2  an 2 ,    
  2  6  n  2  n  3 
2 n n n
1 5 5 5
  , an 12  an 2    an 12    . a12 17.  n.Tr   r  r  1 r  2  r  3
3 9 9 9 r 1 r 1
n 1 n
5 1    r  r  1 r  2  r  3 r  4    r  1 r  r  1 r  2  r  3 
 an 12    An  an 2  5 r 1
9 10 1
n 1   n  n  1 n  2  n  3 n  4   0 
5 1 5
   n n
9 10 t 1
18.  r  
n 1 r 1 r r 1 r  r  1 r  2  r  3 
9
    10
5 1 n  1 1 
    
3 r 1  r  r  1 r  2   r  1 r  2  r  3 
n5
2
14. a1  1, an12  1      2  an 2 , 1 1 1 
     
n 3  6  n  1 n  2  n  3 
an 12   2 2  2  1 .an 2   2 2  2  1
19. (a) a  b  c
  n 1 1 8  a 2  b  a   b2  c  b   c 2  a  c   0
 A    2
n
2
 2  1 
2  2 2

3
n 1 n 1 1
2 (b) f   0  
16  16  3  0 3
1 3 Apply L-Hospital rule
 , 2
4 4 2  1
2
(c) 4 sin x   a   is true if
15. a1  1, an 12  1      2  an 2 ,  a
 
sin x  1  a  1
an  2an 1 (d)
an 2  2an 12 s  3n  s  3n 
s  2n   s  n    f n  3
 an 2  2  2 2  2  1 an 2 3 s  2n   s  n 

1
  1 2n  2n 
2  lim  f  r   lim 3    6
n  n
n r 1
n 
 n 
1 n
16.    Tr  f  n   given  20. log 6  abc   6
r 1 tr r 1

1   abc   66
 Tn   f  n   f  n  1
tn b
Let a  and c  br
  n  1 n  2  n  3 r
36
n
1 n
1  b  36 and a 
 T    r  1 r  2  r  3
r 1 r 1
r
r
 r  2,3, 4, 6,9,12,18
 1
Also, 36  1   is a perfect cube
 r
SOLUTIONS 210

50
 1
 36 1   is a perfect cube for r = 4   tan 1 1  r  r 2   tan 1 1  r  r 2 
 r r 1
 a  b  c  36  9  144  189  2550 
21. Number of terms in each group 1, 3, 7, 15 ….  tan 1 1  50  502   tan 1 1  tan 1  
 2552 
Last term in each group 1, 4, 11, 26 ….an
Now S  1  4  11  26.....  an k 6
24. S3n  2  S4 n  S3n 
S  1  4  .......an
3S3n  2S4 n
 an  1  3  7  15...... n term
3n
an  1  3  7  15...... n term 3  2a   3n  1 d 
2 
an  1  3  7......... n th term
4n
nth term = 1  2  4  8....   2n  1  2   2a   4n  1 d 
2
n
 18a   27n  9  d  16a   32n  8  d
so an    2n  1  2  2n  1  n
n 1  2a   5n  1 d
so last term nth group = 2  2n  1  n Sum of first 2n terms
th
Now, k 
last term of  n  1 group = Sum of next 2n terms
2  2n 1  1   n  1 2n
 2a   2n  1 d 
 2 
Sum of the numbers in the nth group 2n
 2  a  2nd    2n  1 d 

 1  2  3.....2  2n  1  n  2 
2a   2n  1 d

 1  2  3....2  2n 1  1   n  1  
2a  4nd   2n  1 d


2 n 1
 n  2  2 n 1
 n  1  2   n  1 2n  n 
n


 5n  1 d   2n  1 d
2  5n  1 d  4nd   2n  1 d
2 n 1
 3.2   2n  5  7nd 7
n 1
k 
2   n  1 11nd 11
a  b  0.8 11k  7 7  7
So,  1
14 14
k 2 .k ! 2010 k .k ! 2010 k ! k  k 
2010 2

22. 
k 1 2
k
 k 
k 1 2 k 1 2k
1 1
25. H n  1    ... 
1
2 3 n

2010
 k  1!k 1 1 1 1
H n 1  1    ...  
k 1 2k 2 3 n n 1
2010
  k  2  !  k  1 !  2012! 2! 1
   k 1   2010   H n1  H n 
k 1  2k 2  2 1 n 1
Sum = 2 1 H  Hn
Now, Tn   n 1
50  1  r  r 2  1  r  r 2    n  1 H n H n 1 H n H n 1
1
23. S   tan   1 1
r 1  1  1  r  r 2 1  r  r 2    Tn  
  H n H n 1
n
Sn   Tn
n 1
SOLUTIONS 211

 1 1   1 1   1 1   1 1  2 1

H

H

H

H

H

H
  ...  
H

H
 
   2

 from 1 and 
 1 2   2 3   3 4   n n 1 

1 1 1 2 1
   1 
  3

 from  2 
H1 H n1  1 1 1 
1    ...   2
 2 3 n 1 But  satisfies t2 x  6x 1  0  t2  8
2
S  Lim Sn And  satisfies t1x  4x 1  0  t1  3
n 
1 1
  28. 
  an 1 an  an  1
1
 Lim 1   1 1 1
n 
  1  1  1  ...  1     
  2 3  an 1 an an  1
n  1  
1 1 1
1   
 1 n 1 an  1 an an 1
1
Lim  1 1 1 1 1
r 1 r
n 
   ...   
1 a1  1 a2  1 a2012  1 a1 a2013
 1 n 1 1
1 1  2
Lim   a2013
n 
r 1
r n
n 1
& 0 1
1 a2013
 1 1
1 29. 81  144a 4  16b 4  9c 4  144abc
0 x dx A.M  G.M.
1 1 1 81  144a 4  16b 4  9c 4 1
 1 1
 1  1  1 0  1   81 144a 4 16b 4  9c 4  4
log x  1      4
0
an / 2 an  81  144 a 4  16b 4  9c 4  4  3  2 3a  2b  3c
26. An  2  2 an x dx  2  2 an an  x dx  81  144a 4  16b 4  9c 4  144abc
0 an /2
Here equality holds
2 an /2 2 3/2 an  81  144a 4  16b 4  9c 4
An  4 an   x 2/3   4 an   an  x  
3 0 3   an /2
 3  2 3a  2b  3c
3 3
8 a a  8  a a  a ,b ,c 3
An  an  n n   an  0  n n  2 2
3  2 2  3  2 2  Here, c  b  a
  
4 2 2  CBA
An  an sq. units 3 9
3 Now, a 2  b2    3  c 2
4 1 4 4
27.     ;   ... 1  Triangle ABC is right angled
t1 t1
1 3 3
6 1  Area of ABC  ab 
And     ;   ... 2  2 8
t2 t2 30. x, y , z are in A.P.
  ,  ,  ,  are in HP Let x  y   and z  y  
SOLUTIONS 212

cos  y     cos y  cos  y     1 


3a  12kd
3 3a  3  4k  1 d
sin
 2  cos  y  4d  12kd

 4d  3  4k  1 d
sin
2
4 1  3k 
1 
sin  y     sin y  sin  y     1  12k 
2
3 a4k +1 4 1  3k 
sin Let, 4  4
2  sin  y  a4k 1  12k

  1  3k  1  12k  9k  0  k  0
sin
2 which is not possible
th
 cot y  2 Hence, ratio of  4k  1 and  4k  term can’t
3 be 4
sin
2  3  3  4sin 2  n

 32. an   br
sin 2 2 r 1
2 where, br is number of positive integral
3 2 solutions of the equation,
 cos  
2 2 x1  x2  ........  xr  n
2
31. a9  a5  a16  br   n  r   r 1 C r 1  n 1C r 1
2
  a  8d    a  4d  a  15d  n

2 2 2 2 So, an   n 1 C r 1  2n 1
 a  64d  16ad  a  60d  19ad r 1

 4d 2  3ad Now, an 1  2an  2  2n  2  2  2 n 3 


 4d  3a
 2  2n 2   2n 1  an
(A) If a1  4k ; k  I
Now, an 1  an  2  2 n  2  2 n 3
then 4d  3  4k   d  3k
 3  2n 3   an
 every term of A.P. is an integer
(B)  85, 149, 261 is an increasing triad 33. tn 
 n  2  n  1
2
if a149 n  n  3
 a85  a261
2 t3t4 .....tn
if  a  148d    a  84 d  a  260d 
 1 4     2  5     3 6  
if    
a 2  296ad  21904d 2  a 2  344ad  21840d 2  3  6    4  7    5  8 

...
 n  4  n  1 n  3 n 
if 64d 2  48ad  n  1 n  2  n  n  3
if 4d  3a
which is true 
1 4 2 5
Hence,  85, 149, 261 is an increasing triad.  n 1 n  2 n  n  3
1
(C) If d  , then a 
1
n  50 
1 4 2 5  1
4 3  49 52 50 53 72.13.5.53
a4 k 1 a  4kd
(D) 
a4 k a   4k  1 d
SOLUTIONS 213

n  1 1   1 1 
34. tn   1  2  2  ....   2  2  2  .... 
n  n 1  n  2 2 3 2 4
x   
n  1 1 
 1  2  2  .... 
n  2  n  n  1  n  1  1  2 3 
1 n 1 1 1 2
     x  1 2  x
n  2  n n n 1 n 2
2009 x2
1 1 1 1 1 1
t
n 8
n      ... 
2 3 3 4

2008 2009 38. Let a 
x y
, b  xr , c  xr 2
2
1 1 Then y  xr 3
 
2 2009 sin 3 B  sin 3 C b3  c3
Now, 
100
k 1  100 1 1  sin A sin B sin C abc
35.  4
k 1 k  4
    
4  k 1  k  12  1  k 12  1 
3 3
x r x r 3 6
 2
x  xr 3 2
1  10000 5100   xr  xr
    2
4  10001 10202 
39. n1  L.C.M .  2,3, 4,...9   1
36. (A) On rationalizing,
n2  2  L.C.M .  2, 3, 4,...9   1
n
 k  1 k  k k  1 1

k 1 k  k  1
 1
n 1 n2008  2008  L.C.M  2,3, 4,....9   1
 2008  2520  1
3k
n
3k 1 3 3n
(B)  k k
    Remainder when n2008 is divided by 11
k 1 3  2 3k 1  2k 1 3  2 3n  2n
1 1 is 5
n k k
(C)  2  2
1 1 40. Where S  1.2 n  2.2 n 1  3.2 n  2  ...n.2
k 1 2 2
k k  k k 1
2 2 S  2n 1  2.2 n  2  ...   n  1 .2  n
1 1 2
n k  k  1  Subtracting,
 2  2
1
k 1 2
1 2 1 S   2n  2 n 1  ...  2   n  2  2 n  1  n
k k   k  1   k  1  2
2 2
2n  1  S  2  2 n 1  2   2n  2  2 n 1  n  2 
 1 2
2n  2n  1 n 1
Hence,  2 (i.e.) n = 7
1 1 1 1 4
(D)   
xk 1 xk  xk  1 xk xk  1
1 1 1
  
xk  1 xk xk 1
1 1
1  2  2  ....
37. Let x  2 3
1 1
1  2  2  ....
2 3
SOLUTIONS 214

Matrices and Determinants


1. Total number of matrices that can be formed 7. det  A0  .det  I  A0 B02  A02  A03 B04  .... upto
is 39
Let A   aij  33 where aij  1, 0,1 
10 terms) = 0  det  A0   0 
8. B1  adj  B0   B0
If ‘A’ is symmetric then aij  aij  i, j
Hence, B1  B2  B3  .....  B49  B0
If ‘A’ is skew symmetric then aij   aij  i, j
 B1  B2  ....  B49  49 B0
 a 2  b 2  m 0 
2m   9. A0  0 and adj  A0  .C1  0
2. A 
 2 m  No solution
 0  a  b  
2
10. Conceptual
m 1 0  1 n n... n 
  a 2  b2     
0 1  n 2 n... n 
11. An 
MA  A2 m  n n 3... n 
 
MAA1  A2m A1  n n n... n 
M  A2 m1  An is nonsingular for all ‘n’
n 1
M  A2 m 2 . A An   1 . n!
2 m 1 1 0   a b  12. (A) a11  a22  a33  0 & a12  a21  0 &
  a2  b    
0 1  b a  a31  a13  0 & a23  a32  0
m 1  a b (B) iˆ  ˆj  kˆ  a11  a22  a33  0
  a2  b2   
 b a  (1) iˆ  1, ˆj  2, kˆ  1 then a12  a21
3. det  1  x 3  y 3  z 3   11 (2) iˆ  1, ˆj  3, kˆ  1 then a13  a31
 2,1,11, 2,11,1, 2  (3) i  2, ˆj  3, kˆ  2 then a23  a32
4. By the property of inverse matrices a is true (C) & (D) are by similar argument
A1  A1 I  A1  AB  13. Conceptual
14. We have five entries 1 and the remaining four
  A 1 A  B
entries are 0
 IB  B Since the matrix is symmetric, we must have
 b is true an even number of zeros for i  j
Similarly B 1  A Hence, we have two cases:
Option c is true (i) Two entries in the diagonal are zero
We can select two places from three (in
5. AB  A, BA  B,
diagonal) in 3 C2 ways
A2 B 2  A  AB  B  A  AB 
Now we have to select elements for the upper
 A2  A  BA   AB  A triangle
6. Observation gives the conclusion by For the upper triangle, we have three places
induction of which one entry is 0 and two entries are 1
 3n  1 n  One place from three can be selected in 3 C1
An    ways
 9n 3n  1
SOLUTIONS 215

Hence, the number of matrices is 12   2 1  k .... i 


3
C2  3C1  9
1 2   2  2  0 .... ii 
(ii) If all the entries in the principal diagonal
is 1, we have two 0 and one 1 in the upper 11  1 2  0 ....  iii 
triangle  2 1   2 2  k ....  iv 
Hence, the number of matrices is 3
Thus, total matrices = 12  i    iv 
15. Dc  D 2  9 12   2 2  0
x3  1 0 x  x4 1    2 .....  A 
2
 Dc  0 x  x4 x3  1    2 1  k
1

x  x4 x3  1 0  2 1   2   0
16. Observe that 1, 2, 3, 4 are roots of 1 1   2   0
n 4  wn3  zn 2  yn  x  0  2 1   2 2  k
w  10, z  35, y  50, x  24 If 1   2  0 then  2  1  0
17. AT  A So, 1   2  0
BT   B
1 1 T det  A   1 2   2 1
Now  A  B   A  B   A  B   A  B  
      2 2   2 1 
1
 A  B  A  B    A  B  
1 T T
  A  B k
1 1 1      2 2  0
2

  A  B  A  B   AT  BT  AT  BT  1  2  0   2  1  0
1 1
  A  B  A  B  A  B  A  B   0 2 
1 1 A  also satisfies A2  kI  0
  A  B   A  B  A  B  A  B   I  1 0 
1 1
  A  B   A  B  is orthogonal matrix 19. 1  AB  exists
1
Consider the equation 1  BA  X  1,
 A  B   A  B    A  B   A  B 
1 1
and
i.e., X  1  BAX
if X is an orthogonal matrix than so is X 1  A  X  1  1  AB  1 AX
For orthogonal matrix X, det X  1 1
But As  X  1  BAX  B 1  AB  A
1 1
det  A  B   A  B  A  B   A  B   X  1  B 1  AB  A
1
 
 det I  1 20. A  B  AB
or,  I   A  B  AB   I
1 1
det  A  B   A  B   .det  A  B   A  B    1   I  A  I  B   I
   
Either both will be 1 or −1  I  A . I  B 1
18. A2  kI  0
1  2  1  2   k 0 
Also,  I  B  I  A   I
          0 k   I  B  A  BA  I
 1 2 1 2  
as A  B  B  A & A  B  AB
 AB  BA
 (a) and (c) are correct
SOLUTIONS 216

4
21. tan 2   tan 2  4
3
d11  d 22  3cos 2  4sin 2  5
n

Use of S T AS   S T An S ,
S 1  S T , gives conclusion
1  4.350  250 2.350  251 
A50   50 51 
5  2.3  2 350  252 
22.    111
  .3.37     
Clearly,  is equal to the last digit of  2 .
Now 37 must divide one of the  or 
Let it be  then   37 or 74 (if   8 ,
then  .3.37  12.74 )
  74 is not possible because
     74.14  1036
 for   8
But  111  12.74 
   37    27
   7,   3,   2 &   9
         21
 a b c   a c b  1 0 0 
23. PP   c a b  b a c   0 1 0 
T

 b c a   c b a  0 0 1 

 a 2  b 2  c 2 ab  bc  ca ab  bc  ca  1 0 0 
 
  ab  bc  ca a 2  b2  c 2 ab  bc  ca   0 1 0 
 ab  bc  ca ab  bc  ca a 2  b 2  c 2  0 0 1 
 

24. Conceptual
SOLUTIONS 217

Permutations and Combinations(PnC), Binomial


Theorem
5. Length of the rectangles may be 1, 3, 5, 7
1. Ax4  108 units breadth of the rectangle can be 1, 3, 5, 7
units
108 4 4  4
 A Number of required rectangle = 6
x4 2
 x1 x2 x3 x4  108  2 2.33 6. Let ‘x’ be the number of one rupee coins then
the number of 50 paise coins may be even or
xi  2 i 3 i  i  1, 2,3, 4 odd.
1   2   3   4  2 If number of 50 paise coins is 2y then number
1   2   3   4  3 of 25 paise coins must be a multiple of 4
2 y 4z
Required = 2  4 1
C2  3 4 1C3  10  20  200  x   20
2 4
2. x1  x2  x3  x4  x5  x6  10 where  x  y  z  20 where x  0, y  0, z  0
22
0  xi  10  Number of ways = C2  231
n  n  1 If number of 50 paise coins is 2 y  1 then 25
n
3. C2   number of possible pairs of paise coins must be of the form 4 z  2
2
2 y  1 4z  2
n objects  x   20
2 4
n  n  1  x  y  z  19, x  0, y  0, z  0
Total weight of pairs are
2  Number of ways = 21 C2  210
n  n  1  Number of selections = 441
 2  w  n  n  1 w units
2 3x  4 y  2 
2

 x  1   y  1  Cn 
2 2 m
Similarly total weight of all triplets = 480 7. 
 5 
……(1) m
Cn  1   2,1 3,1 4,1 4, 2  5,1 5, 2 
n  n  1 n  2  w
  480 ....  2   Number of hyperbolas = 6
2 8. A  2, 3, 5, 7,11,13,17,19, 23, 29
n2
Dividing (1) and (2), we get 4 Two different numbers for numbers and
2 denominator from these can be obtained in
 n  10 p q
10
x y P2  10.9  90 ways and if   1
4. The line   1 where ‘p’ is a prime and q q
p q So, number of ways (if numerator and
’q’ is a positive integer passes through (4, 3) denominator are same) = 90 + 1 = 91
3p 12 9. On solving the given four lines, we get to
only if q   3 .
p4 p4 know that lines y  x  3, y  2 x  3, y  x  3
Since, ‘q’ is a positive integer, p > 4 and (p – are concurrent at (0, 3) and the line
4) divides 12, there are only two such primes, y  3 x  2 intersects these lines at points
p = 5 and p = 7.
1 7  1 11 
 ,  , 1, 5  ,  ,  , respectively.
2 2 4 4 
SOLUTIONS 219

Hence option (a) is correct 26. (A) Sum of digits must be divisible by 3 and
23
19. 23  23  530  1
11 the digit at units place must be an even
number
 2353011  11C1 53010  ...  11C10 530  11C11
4! 4!
(B) 8 P4  3C1. 7 C2 .  3C2 .
 2323 is divided by 53, the remainder is 30 2! 2!2!
Statement (2) is true (C) 10  1 8  1 6  1  1
20. 5 J m  3 J m 1
(D) 120  60  24  12  6  2  236
m3 m2
 5  m Cr mCr 3  3  m 1 Cr m 1
Cr  3 27. (a) standard
r 0 r 0 12 12! a b c
(b)  x  y  z    x y z
 5  2 m Cm  3  3  2 m  2 Cm  2 a !b ! c !
 7m 2  8m  46  0 Where a  b  c  12
12!
m
21. 1  y  1  y 
n   312  k  4
a !b !c !
 m  m  1 n  n  1  (c) 1323000  23.33.53.72 which is divisible by
 1   n  m y     mn  y 2  ... 105; 2, 3, 5, 7 must occur atleast one time
 2 2   Total number of divisors =
100  100  100  100  3  3  3  2  54
22. (a)   2  3  4
 3   3   3   3  Hence, 2k  10  54  k  6
 33  11  3  1  48 (d) From 1 to 9, there are 9 × 1 = 9
 50   50   50  From 10 to 99, there are 90 × 2 = 180 digits
(b)     2    3   16  5  122 From 100 to 599, there are 500 × 3 = 1500
 3  3  3 
digits
100!25!
(c)  max. value of P such that From 600 to 659, there are 60 × 3 = 180 digits
50!25.50! From 660 to 666, there are 7 × 3 = 21 digits
3P divides expression is 14. 28. (a) One Indian wife and one American wife
 25   25  can be selected in 2 C1  2C1 ways and keeping
(d)     2   8  2  10
 3  3  an unmarried person in between these two
23. For 1st event there are 7 ways wives the total number of linear arrangements
2nd event there are 7 ways are 2 C1  2C1  7! 2!  40320
2nd event there are 7 ways (b) Required number of ways = 8!  40320
2nd event there are 7 ways (c) Required number of ways =
8th event there are 7 ways  7  1! 2! 2C1  2C1  5760
 Total no. of ways = 78 (d) Number of ways in which interviews can
24. 1  1  1  1  1  1  2  8 be arranged
Total number of ways =  9  8C2  6C2  4C2  2C2  22680
8! 18  17
29. (A) 
16  3  1
 7!  28  5040 C3  1  18 C2   153
2! 6! 2
n 6
25. 1  1  1  1  1  3  8 (B)  1   1  4
2 2
11 1 1 2  2  8
100
Total number of ways = (C)  4  (T1, T25, T73 are rationals)
24
 8! 8!  Total number of terms rational terms =
    7!  266  7!
2
 3!5! 4! 2! 2! Irrational terms  101  4  97
SOLUTIONS 221

From 2 to 2000 number of multiples of 3 are n n n 1

 2000 
 52   nC0  5  nC1  5 21  ....
 3   666 n n n 1

From 2 to 2000 number of multiples of 6 are


 5  2  n
C  5 0 n C  5
1 21  ....

 2000  where n is an odd prime


 6   333 n n
  52    52 
 Number of possible ‘n’ are = n 1 n 3
1999  1000  666  333  2  nC1  5 21  nC3  5 23  ....

10
C7  6 C3 2
40.  nCn  2  5 2n  2  nCn 2n  ....  ii 
9  9 P6 
41. The exponent of 2 in n! is
n  n   n   Let,  52 
n
I f,  5 2 
n
 f1 
 2    22    23   ....  I  f  f1  integer
n n n  f  f1  some integer
    ....  n
2 2 2 23
Now, 0  f  1,  1   f1  0
The exponent of 2 in n! is less than n
  1  f  f1  1
there is no positive integer for which 2n
divides n!  f  f1  0  f  f1 is an integer 
n
Hence, form equation (ii)
 2
42. f  n     r  1 n Cr  r 2  nCr 1  n 3
r 1
I  2  2  5  nC1
 5   ....  C 2   2 n n 3 n 1
n n 2  n2
  n  1  Cn 1   n  1  1
n 3
43. 2n
Cn  2  C0 2  C12  C2 2  ...  Cn 2  2  I  2  20  C  5   ....  C 2 
n 1 n n n 3
 1 n2

 C12  C2 2  C2 2  ...  Cn 12 n


 Integral part of  5  2   2  20n  n 1
n n n n
(as C1 , C2 , C3 ,... Cn 1 are divisible by n)
(an integer) (using equation (i))
 2 n Cn  2 is divisible by n2 n

2n
Cn  2, n 2
 Integral part of  
5  2  2n1 is
divisible by 20n
44. 24 n  2n  7 n  1  7 2  2n  n C2  n C3 73  ...
 Statement 1 is true
 142  a a positive integer 46. (a) Number of onto functions from a set of
n  n  1 n  2  .... r factors containing 6 elements to a set containing 5
45. nCr   an integer elements  6C2  5
r  r  1 r  2  .... 3.2.1
In above expression, in denominator, there is (b) Number of function from a set containing
5 elements to a set containing 6 elements =
no factor of n as n is prime and 1  r  n  1 0
So, nCr  r   some integer  ...  i  (c) Number of onto function from a set of
containg 6 elements to a set containing 6
 nCr is divisible by n, if n is prime and
elements  6  720
1  r  n 1
 Statement 2 is true.
Now,
SOLUTIONS 222

47. Let there are m-n seats in a row, select n/2   2n  1 p  2   1
n

gaps out of m – n + 1 gaps (as in this case


 2n  where p  3, n is even number 
persons will sit in pairs and pair will adjacent)
Now in these n/2 gaps make 2 people sit in 52. (A)  a  b  c  d   81
each gap Applying AM  GM
Hence required number of ways =  a  b    c  d   a  b c  d 12
m  n 1
2
    
Cn / 2  n !
48. Let about middle, seats are numbered as 1, 2, Minimum value of a  b  c  d is 18
14 14
3,…., m/2 and 1′, 2′, 3′,…, m′/2 (B)
 x  y  1  xy 
14
Out of these m/2 numbers selected n numbers  xy 
now corresponding to each say k and k′ Both factors in the numerator have 15
One should be filled and one should be empty independent terms
hence seat can be selected in 2 ways. Total number of terms 15 15  225
86 43 43
Hence required number of ways = m / 2 Cn 2 n. n ! (C)  23   529    530  1
n 43 42 41
49. 1  px  x 2   1  a1 x  a2 x 2  ...  a2 n x 2 n   530   43C1  530   43C2  530   ...

2
Differentiable on both sides 143 C41  530     43  530   1
n 1 
n 1  px  x 2   p  2 x   a1  2a2 x  ...  2na2 n x 2 n 1 86
Hence the last two digit of  23  are last two
Multiplying 1  px  x 2  on both sides
digits of  43  530   1 that is 89
n  p  2 x  1  a1 x  a2 x 2  ... 1 log10 x 2
(D) T3  5C 2
x3
x   1000 
 1  px  x 2  a1  2a2  ...  2na2 n x 2 n 1 
Taking log on the base 10 both side
Equate coefficient of x r on both sides 2
npar  2nar  r   r  1 ar 1  p.r.ar   r  1 ar 1 3log10 x  2  log10 x   2  0
Put log10 x  t
 np  pr  ar   r  1 ar 1   r  1  2n  ar 1
2t 2  3t  2  0
50. a1  5a2  9a3  ...   8n  3 a2 n
1
2n t , 2
   4r  3 ar 2
r 1
x  1, log10 x  2  x  100
n
n
 4 rar  3  p  2   1 53. There are 302 points (m, n) with 1  m, n 
60
r 1
 
2
 n
  n
 4 n  p  2   3  p  2   1

We must exclude those with
2m < n & 2n > m
n
  p  2   4n  3  3 The number of points satisfying 2m < n is 0
2n for n = 1, 2
51. a1  3a2  5a3  ...   4n  1 a2 n    2r  1 ar The number of points satisfying 2m < n is 1
r 1 for n = 3, 4
2n 2n
The number of points satisfying 2m < n is 2
 2 rar   ar
r 1 r 1
for n = 5, 6
The number of points satisfying 2m < n is 14

 2 n  p  2 
n
  p  2 1n
for n = 29, 30
SOLUTIONS 223

 Required value = For S to be divisible by 4, last four terms


900  2.2 1  2  ...  14   480 must be equal to 4 or zero.
For this a2 and a4 must be both even or both
54. 3375  53.33 , 1125  53.32
odd and a3 a5 can be any digit
Clearly, 33 is a factor of ‘x’ and 32 is a factor
of atleast one of y & z This can be done in 2   8  5  10  5 10  =
This can be done in 5 ways 40,000 ways
Also, 53 is a factor of atleast two of the  Largest possible number of values of
numbers x, y, z which can be done in
3
 a1 , a2 , a3 , a4 , a5  is  9000  5 
C2  4  2  10
 k 5
 k  50
55. a  b  c  11 or  1
d  e 1
or a  b  c  1 or  1
and d  e  11
take a  4  a, b  4  b, c  3  c, d   d  2
and e  e  2
it becomes
a  b  c  0 or 22
d   e  5
or a  b  c  10 or 12 and d   e  15
Answer =
1  24
C2  17C2   6   14 C2  7 C2  12C2  5C2   16
Compute unit digit
56. Let S  2 a1  3a2  5a3  7 a4  9 a5
a a a a
 2a1   4  1 2   4  1 3   8  1 4   8  1 5
a a a a
 2a1  4   1 2  1 3   1 4  1 5  
Case I: When a1  0 in this case
S will either of  4n  1 or  4n  3 form
which is obviously not divisible by 4
Case II: a1  1,
a a a a
S  2  4   1 2  1 3   1 4  1 5
If S is divisible by 4, last four terms must be
equal to 2.
For this, one of a2 and a4 must be odd and
other to be even
This can be done in 2 1 5 10  5 10  ways
= 5000 ways
Case III: a1  2,
a a a a5
4n     1 2  1 3   1 4  1
SOLUTIONS 224

Solution of Triangles (SOT)


1. In an acute angles triangle, 1
3  r   distance from I  0, 0  to BC 
cos A  cos B  cos C  and 2
2 5. R  8r
tan A tan B tan C  3 3 A B C
 R  8  4R  sin sin sin
2 2 2
So, no solution is possible A B C 1
In an obtuse angled triangle,  2sin sin sin 
2 2 2 16
cos A  cos B  cos C  0 but
 AB AB C 1
tan A tan Btan C  0   cos  cos  sin 
So, no solution is possible  2 2  2 16
2. b 2  c 2  1999a 2 1 C C 1
   sin  sin 
 b 2  c 2  a 2  1998a 2 2 2 2 16
b 2  c 2  1999a 2 C 1 C 1
 sin 2  sin   0
 b 2  c 2  a 2  1998a 2 2 2 2 16
2
cot B  cot C sin  B  C   C 1
   sin    0
cot A cos A  2 4
sin Csin B 
sin A C 1
 sin 
2
sin A 2 4
 C
sin Bsin Ccos A cos C  1  2sin 2
 a2   2
2bc 
   2 2 2  1
 bc   b  c  a   1 2
16
2a 2 7
 
1998a 2 8
1  
 6. B  as 4sin A cos B  1 and A  as
999 2 2
3. 2sin A cos B  sin C tan A is real
sin  A  B   sin  A  B   sin C 
 C 
Consider sin  A  B   0, that is A  B and 2
Therefore angles are in the ratio 1: 2 : 3
C    2A
R 1
A
1  tan 2 4 24
C 2 7. sin A  , tan C  , that is
 tan  cot A  5 7
2 A
2 tan 24 1000
2 sin C  , c  100, 2R   25
4. Let image of A about y  x, y  2 x be P 25 200
and Q. sin B  sin  A  C 
 11 2  4 7 24 3 100 4
 P   2, 1 , Q    ,       
 5 5 5 25 25 5 125 5
 Equation of BC: x  7 y  5  0 106
  2R 2 sin A sin Bsin C 
3
SOLUTIONS 225

A sin A sin B
8. Let cot x  
2 2 3 1
B 2
cot y  3  4sin 2 B   3 1
2 3 1
C
cot  z 2 3
2  sin 2 B 
4
A sa 2
x  cot 
2 r
 sin 2 B
 3 1
B s b 8
y  cot 
2 r 3 1
C sc  sin B  
z  cot  2 2
2 r 3 1
3s   a  b  c  s  sin B  (negative value rejected)
x y z   2 2
r r  B  15, A  45 and C  120
2
2 2 2 6  B C
 x   2 y    3z      x  y  z  
11. BD  CD  r  cot  cot 
7   2 2
or,
 BC
13 x 2  160 y 2  405 z 2  72  xy  yz  zx   0 sin  
2 2 2 r  2   4R sin A sin B  C
  3x  12 y    4 y  9 z   18 z  2 x   0 B C 2 2
sin sin
 3x  12 y, 4 y  9 z , 18 z  2 x 2 2
9 BC BC
 x  4 y, y  z, x  9 z  4R cos sin
4 2 2
9 9  2R  sin B  sin C   b  c
x : y : z  9 z : z : z  9 : :1  36 : 9 : 4
4 4
12. As a  b  c  a  b  c
x y z
   2R  sin A  sin B  sin C 
36 9 4
s a s b z c  2R  sin D  sin E  sin F 
   (R is same for both the circles)
36 9 4
a b c  sin A  sin B  sin C  sin D  sin E  sin F
   Or,
13 40 45
9. In any ABC A B C D E F
1  cos cos cos  1  cos cos cos
A B C A B C 2 2 2 2 2 2
cot  cot  cot  cot  cot  cot 
2 2 2 2 2 2 13. r1  s   A  90
 A B C sa
 loge  cot  cot  cot  1 1 1 1
 2 2 2    , use of
r1 r2 r3 r
 A  B  C
 loge  cot   loge  cot   log e  cot    s  s  a  s  b  s  c  gives
 2  2  2
a b conclusions
10.  
3  1 a  2b  
2 3 1
14.
SOLUTIONS 228

But 1 3 C
2    cot
3 9 2
tan 2 A  tan 2 B  tan 2 C  3 tan A tan B tan C 3
C
 cot  1
 using AM  GM  2
Also, in an acute angled  ABC C
1
  45
2
tan A  tan B  tan C  3 tan A tan B tan C 3
 C  90
 using AM  GM  So, the area of
But 1 1
tan A  tan B  tan C  tan A  tan B  tan C  ABC  a sin C   6  3 1
2 2
Therefore, tan 2 A  tan 2 B  tan 2 C  9 23. Since angle bisector divides opposite sides in
a2 b2 c2 the ratio of sides containing the angle, hence
So, 2  2  3  144
r1 r2 r2 ac ab
BA= , CA =
bc bc
1  a 2 b2 c 2 
The minimum value of     Now BI is also an angle bisector of B for
18  r12 r22 r23  triangle ABA
is 8 AI b  c AI bc
   
4 A I a AA a  b  c
(D) cos  A  B 
5 BI ac
Similarly,  and
 AB BB a  b  c
1  tan 2  
  2 4 CI ab

 AB 5 CC a  b  c
1  tan 2  
 2  AI  BI  CI  a  b  b  c  c  a 
  3
 AB 1 AA  BB  CC a  b  c
tan  
 2  3 Using AM  GM, we get
But a  b  A  B 1

AB 1 2  a  b  c    a  b  b  c  c  a   3
So, tan    
 2  3 3 a  b  c   a  b  c 
 A B a b C  a  b  b  c  c  a   8
Again, tan    cot 
 2  ab 2 a  b  c 27
1 3 C
   cot 27  AI  BI  CI 
3 9 2  Maximum value 8
AA  BB  CC
C
 cot  1 24. R cos B  2R cos A cos C
2  cos  A  C   2 cos A cos C  0
C
  45  tan A tan C  3
2
25. In  AIF, AIE
 C  90
So, the area of IF IE
 AI 
1 1 A A
 ABC  ab sin C   6  3 1 sin sin
2 2
2 2
IE  IF
 AI 2 
A
sin 2
2
SOLUTIONS 230

 S2  36 cos  B  C  p  1
 
 S6 cos  B  C  1  p
1 36 p 1
  1  r  1 but r1  r2  r3  r  4R  cos  B  C  
r 36 2  p  1
5 3 1 p 1
 11  1  4R  R   0  BC     1
2 4 2 2  p  1
5
 Area  2   6  30 p 1 2p
2  1  and
A B C p 1 p 1
30. cos2  cos2  cos2  p  1  2  p  1  0
2 2 2
 A B C  p  1
 2  1  sin sin sin 
 2 2 2  p  p  1  0 and
 1 
 y  x2  2   2 y
 x 

   
2 1 
 
2  1 p   p  1  0
2
A B C
 y  1  sin sin sin  1  
1 9  p   0, 1 and  2  1  p   p  1  0
 
2 2 2 8 8  
31.  A   a constant  p   0, 1 and p  1, 3  2 2 
If I is centre of  ABC

 p  0, 3  2 2 
BIC  90  which is also fixed chord
2  p  0 or p  3  2 2
Hence I lies on a fixed circle of which BC is a 34. Let AB  x
fixed chord 1
  A   If H is orthocentre Then BC2  x 2  2  1 and
x
 BHC  180   which is fixed 2 x2  1 3
Hence, H lies on a circle of which BC is fixed cos B  that is, B  2
4
2 x  x 12 2 x 1
chord
 A   , HGK   where H, K are AD x
Also,  gives
points of trisection of base BC which are sin B sin  B  60 
fixed x x
 The fixed line segment H, K subtends a AD   2
1 3 x 1
constant angle  at a variable point G  cot B
2 2
Hence, locus of centroid is also lies on circle.
1 1
32. Let ABC be the triangle in which AB  AC  AD   , with equality iff
Let I, P respectively be the incentre and the 1 2
x
orthocentre of the triangle x
A AB  AC  1
AI  r cosec , AP  2R cos A 35. AF.AB  AG.AD
2
c2 2
A   AD2
r cosec  2R cos A  r 2 3
2
33. tan B tan C  p
SOLUTIONS 232

 Area bounded by y  1, y  0, x  y  2 42. c  a  c sin A,


is 3 sq. units sin C  sin A  sin C sin A
I n  n  1 CA B
(B) Let n  2 4sin sin  cos  A  C   cos B
In2 n 1 2 2
CA B
(C) Let x  y  z  w, x, y, z, w  1  2  2sin 2  2sin 2
2 2
x  x1  2
 CA B
y  x  x2   sin  sin   1
  4 x1  3 x2  2 x3  x4  20  2 2
z  y  x3  CA CA
w  z  x4  sin  cos 1
2 2
Number of solutions = 23
2x
(D) y 2  5 y  3  2
x  x 1

f  x 
 x  1 2  2 x  2 x  1
x 2

2
 x2  x  1
2 x 2  2 2  x 2  1
 2
 2
x 2
 x  1 x 2
 x  1
f  x  is increasing on  1, 1 and decreasing
on  ,  1  1,  
2x  2
 The range of is  2, 
2
x  x 1  3
2
 y  5 y  3  2
 y2  5 y  5  0
 5 5 5 5 
 y   , 
 2 2 
SOLUTIONS 234

1  tan1 1  tan 2 1  tan 3  ... 1  tan 25   tan 1 3  tan 1 1   tan 1 5  tan 1 3 
 
 1  tan1 1  tan 44 1  tan 2 1  tan 43   lim    tan 1 7  tan 1 5  ... 
n   
...1  tan 22 1  tan 23  1  tan 45 
   tan 1  2n  1  tan 1  2n  1  
 223  
 n  23  lim  tan 1  2n  1  tan 1 1
n 

8. x yz    
  
 tan x  tan y  tan z  tan x tan y tan z 2 4 4
 tan x  tan z  tan y 
11. 1 2cos 2 x  1  0  x  n 
  tan x  tan z  tan z  tan y tan z  18 6
n
 tan 2 z  18  tan x tan z  18  2  16 (2) tan x  0  x  
7
2
 K 
9.  tan 2   
K 1  16   3 cos 2 x  3sin 2 x  0  x  n 
6
2 2
       Also,
  tan  cot    tan  cot 
 16 16   8 8
2
  
2cos 2 x  1  2 cos2 x  sin 2 x  cos2 x  sin 2 x 
 3 3  2
 cos x  3sin x 2
  tan  cot  1 6
 16 16  Now, the given equation reduces to
  3 b sin x  b  sin x
 cosec 2  4cosec2  4cosec 2 1 6
8 4 8 b
 sin x 
   b 1
 3  4  cosce 2  sec2 
 8 8   1  sin x  1
   b
 3  4  2  cot 2  tan 2   1  1
 8 8 b 1
 11  4   
2 1 
2
   35
2 1
2

b
b 1
 1  0 and
b
b 1
1  0

1
2k 2 2b  1 1
10.  cot   0 and 0
k 1 b 1 b 1
 
1 2  1
  tan 1 2
  tan 1 2  b    ,   1,   and b    , 1
k 1 2k k 1 4k  2

2 1
  tan 1 When b   sin x  1 which is not
k 1 1   4k 2  1 2
possible

  tan 1
 2k  1   2k  1 1

k 1 1   2k  1 2k  1  b    , 
 2

   tan 1  2k  1  tan 1  2k  1  12.  tan x  8 tan x  15 tan 2 x  8 tan x  7   33
2

k 1
n
Let tan 2 x  8 tan x  p
1 1
 lim
n 
  tan  2k  1  tan  2k  1
k 1
  p  15  p  7   33
 p 2  22 p  72  0
 p  18 (rejected) or p   4
 tan 2 x  8 tan x  4  0
SOLUTIONS 236

1  2 cos  cos  2  b   2a cos  cos  2  b 


2 tan 2   tan 
19. tan   3  2 cos   b   2a cos 2 cos   b 
2 3
1  tan   cos  3  b   cos   b 
3
2 tan  
1  a cos   b   cos   b 
 tan   0 or 3 1
2  sin  sin  2  b   a sin b  0
1  tan 3 
3 Now, sin  2  b   a sin b is meaningful only
3
That is, tan   3tan   2  0 when a sin b  1
 tan   1 or  2
A B  
So,   0,   1,   2 25. In given  ABC, both and   0, 
2 2  2
20.  cot   cot       cot   3cot  2    
 
 cos  cos      cos  3cos  2     sin x is increasing in  0,  and cos x is
    2
 sin  sin       sin  
sin  2    
   
decreasing in  0, 
 sin  2     cos  3cos  2      2
 
 sin  sin       sin  3sin 

A B A B
    If   sin  sin  x1  x2
 3sin    cos   cos  2     2 2 2 2
     A B
 sin  sin       sin   cos  cos  x3  x4
2 2
3  2sin     sin   x 
2010
x 
2009
 6  1  3 
sin  sin    
 x2   x4 
21. sin x cos 2 y  or  cos x sin 2 y  1 A B
2
Similarly, for 
2 2
a 2
 1  1  1 and a  1  1
A B 1 1
2    x4  x2 and 3  4
a 2
 1  0 and a  0  a  1 2 2 x x
22.  OAB   OPA   OPB   PAB  BC  AC  1
3   
 4 26. 2 cos  cos  sin   1  sin 
4 2 2 2
Since, the triangle is an equilateral triangle     cos    sin   1  sin 
1   cos     1 sin     1  0
 3   2  d  OA   d  P, OB  d  P, AB  
2 2
 For all positions of P It has real solution if   1   2    1
d  P,OA   d  P, OB   d  P, AB   d  P, AB   3   
27. f  x, y  f   x,  y 
 k  3  3k  3 2 2 
23. 1  a  cos cos  2  b   1  a cos 2  cos   b   1  sin x  1  cos 2 x 
2
 2
 1 2
 1
 1  sin y  1  cos y 


 sin 2
x  sin 2 y  cos2 x  cos2 y 
1  sin y 1  cos y 
2 2
SOLUTIONS 237

2
  sin 2 x  sin 2 y   2  2
    sin x  cos x   1   7
  0  x, y   0,   sin 2 x  sin 2 x
 2 2
1  sin y 1  cos y 
 2 2
 2  4 28
 exactly one of  1  sin 2 x  1    49  2 
 sin 2 x  sin 2 x sin 2 x
      
 f  x, y  , f    x  ,   y    is greater 2
 t  44t  36  0, where t  sin 2 x
  2  2  
than zero Solving, we get sin 2 x  22  8 7
 Statement 1 is true  a  22, b  8
28. Limiting cases 31. (a) 1  x  2  1  1  x  3 is true only for
Statement 1 x 1
(i) cos x  1  0 when x  1, so, 1   cos1  5  3
(d) tan 1 2  tan 1 3    tan 1   
(ii) cos x  2   at x  1, so 2    cos1  5  4
So,     cos1,   cos1 (b),(c) are obviously tue
Statement 2 32. (A) xi2  8  1  xi is odd natural number
 x1  x2  x3  21
Let line 1: y   x 
2 Let x1  2a  1, x2  2b  1, x3  2c  1
d a  b  c  9  coefficient of x 9 in
(i)
dx
 cos1 x   1  1
9 3
min
(ii) Equation of line passing through (1, 0) 1  x  ...  x   11C9  55
  (B) D1  1, D3  9, D5  D6  ...  D21  3
and  1,   is y   x  .
2 2  D1  D3  ...  D 21  37
 (C)  tan 2 x  and a are integers  tan x is
So, 2   .
2
also integer
  
Hence,      ,  1 , the equation 1  1  4a
 2  Roots are
exhibits 3 solutions 2
2
    1  4a   2k  1
So,       ,  1 , the equation
 2   a  k2  k
exhibits exactly one solution  
2sin  x  
29. 2  3  90  2  90  3    6
(D) 2sin  x   4
2  3r 1  90 3r 1  3r   6
  
cos  2  3r 1    sin  3r   sin  x    1
 6 
cos  2  3r 1   1, r is odd  2
  x , 
sin  3  
r
1, r is even 3 3
Similarly, 3
33. (A) 1  cos A  cos B  cos C 
 cos  3  2r 1   , r  1 2
 A B C 1
cos  2    
r
0  sin sin sin 
 cos  3  2   , r  2
r 1
2 2 2 8
1 sin x  cos x  Area bounded by y  1, y  0, x  y  2
30. sin x  cos x   7 is 3 sq. units.
sin x cos x sin x cos x
SOLUTIONS 242

1b c  1a c  p
    . sin A    . sin B  OA  OB  1  y  px 
23 3  2 3 3  p 1
1a b  p
  . sin C 
2 3 3  d 
 p  1 
p
1 2 1  p2  p  1 1  p2
1     3  
9 3
1 2 dd
  for mind d ,  0  p  1
 3 dp
20. The quadrilateral formed by angular bisectors  d min 
  1 
1
is a rectangle whose sides are  2  2 2 2
 
 a  b  sin ,  a  b  cos
2 2 23. OAOB
.  4  y  px  2  p
S  ab sin 
2 p
1 2 d
Q   a  b  sin  1  p2
2
S 2ab a S  Q  Q 2  2QS
dd
   for dmin   0  p  1
Q  a  b 2 b S dp
2   1
21. Sine AB subtends right angle at P and Q on  dmin   2
variable line 1 12
So AB is a diameter of circle whose chord is a
variable line p
24. AB  1  y  px 
Equation of circle is 1  p2
x2  y 2  6 x  0 ..... i  p
Equation of line through (2, 4) is 1 p2 p
d  
y  4  m  x  2 .... ii  1 p 2 1 p2
Line (ii) is chord if
3m  4  2m dd
3 for dmin  0 p  1
2
1 m dp
2
8m  8m  7  0 1
 d min 
2
 23 2   23 2 
 m   ,  ,  
 4   4  25. Let
L1 : x  y  2  0
22. Equation of tangent at (x, y) is L2 : 2 x  y  3  0
Y  y  p  X  x sign given by origin
dy y L1  0  0  0  2  0
Where p then OA  x  and
dx p L2  0  0  0  3  0
O B  y  px Sign given by points in acute angle:
SOLUTIONS 243

  a1a2  bb
1 2     2 1  1  0

So  a , a 2  should give

  ve sign with L1 and   ve sign with L2.


So, a  a 2  2  0
 a 2  2a  a  2  0
  a  2  a  1  0
 a    , 2   1,  
and
2a  a 2  3  0
a 2  2a  3  0
 a  3 a  1  0
 , 1   3,  
Their intersection gives   ,  2   3, 

26. For the equation to be pair of lines Δ = 0 then


a=2
If the three lines are concurrent, no circle
exist then b = 5

4
27. sin A  , AB  AC  5 2
5
So, point B will be (0, 10), (10, 0) and point C
will be (4, -2), (6, 12)
 13 
Centroid will be  3,  ,...
 3
 5 5 
Circum centre will be  , 
 2 2
SOLUTIONS 253

1 Equation of the tangent is y  ax  1


 r 
5 Intercepts are -1/a and 1
Equation of the tangent at (0, 1) to the  Area of the Δ bounded by tangent and the
parabola y  x2  ax 1 is 1 1 1
axes =  .1 
y 1 a 2 a 2a
  x  0  1
2 2 It is minimum when a = 2
i.e. ax  y  1  0
 minimum area = 1
1 4
r  55. Since no point of the parabola is below x-axis
2
a 1
radius is maximum when a = 0  a2  4  0
 equation of the tangent is y = 1  maximum value of ‘a’ is = 2
 slope of the tangent is 0 Equation of parabola, when a = 2 is
Equation of the tangent is y  ax  1 y  x2  2x 1
Intercepts are -1/a and 1 It intersects y-axis at (0, 1)
 Area of the Δ bounded by tangent and the Equation of tangent at (0, 1) is y  2 x  1
1 1 1 Since y  2 x  1 touches the circle
axes =  .1 
2 a 2a
x2  y2  r2
It is minimum when a = 2 1
 r 
 minimum area = 1 5
4
Equation of the tangent at (0, 1) to the
54. Since no point of the parabola is below x-axis
2 parabola y  x2  ax 1 is
 a 4  0
y 1 a
 maximum value of ‘a’ is = 2   x  0  1
2 2
Equation of parabola, when a = 2 is i.e. ax  y  1  0
y  x2  2x 1 1
It intersects y-axis at (0, 1) r 
a2  1
Equation of tangent at (0, 1) is y  2 x  1
radius is maximum when a = 0
Since y  2 x  1 touches the circle  equation of the tangent is y = 1
x2  y2  r2  slope of the tangent is 0
1 Equation of the tangent is y  ax  1
 r 
5 Intercepts are -1/a and 1
Equation of the tangent at (0, 1) to the  Area of the Δ bounded by tangent and the
parabola y  x2  ax 1 is 1 1 1
axes =  .1 
y 1 a 2 a 2a
  x  0  1
2 2 It is minimum when a = 2
i.e. ax  y  1  0
 minimum area = 1
1 4
r 
2
a 1
radius is maximum when a = 0
 equation of the tangent is y = 1
 slope of the tangent is 0
SOLUTIONS 256

2 2
  As i z1 z2  z2 z1  z1  iz2
e
i  2 B 
e
i  2 B

z2 z2 or, iz1 z1 z2  z2 z2 z1  z1  iz2
z1 z z or, z1 z2  iz1  z2   i  z2  iz1   As z1 z2   z2 z1 
 1 ei 2 B  1  ei  1
z3 z3 z 2 z3 or,  z1 z2  i  iz1  z2   0
z z  either z1 z2  i or iz1  z2  0
   2 3
z1
z1 z2  1 or z1  z2
6
13. z 6   z  1
z12 z2 z2
6 16.  2  3 1  0
 z 6   z  1 z2 z3 z1 z3 z1 z2
 z  z 1  z13  z23  z33  z1 z2 z3  0
1   z1  z2  z3   z12  z2 2  z32  z1 z2  z2 z3  z3 z1 
 z lies on the line x  
2  4 z1 z2 z3
6 6
 Roots of z   z  1 are collinear
b c
 2
  z1  z2  z3   z1  z2  z3   3 z1 z2 z3  z1  z2  z3  
14. z1  z2  ; z1 z2   4 z1 z2 z3
a a
 z1  z2  1
2
3

  z1  z2  z3   z1 z2 z3 4  3 z1  z2  z3
2

  z1  z2  z1  z2   1 3
 z1  z2  z3  3 z1  z2  z3  4
2

 2  z1 z2  z2 z1  1
 z1  z2  z3  1 or 2
2
z  z 
 1 2 1 z z z
17. 1 2 3  3
z1 z2 3
b2 c Cleary centroid coincides with circumcentre,
   b 2  ac hence ABC is equilateral.
a2 a i 2
Now, z2  z1ei then z1  z2  z1 1  ei 18.   e 28
 1 1  
 2 cos 1 x      2sin  sin
2 2 2 14 14
2  
  If    3  4 
3 14 2
PQ  z2  z1  3 sin 3  cos 4
2

15. iz2 1  z1
2
  z 1  z 
1 2
2
3 x  4 x3  2 cos 2 2  1  2 1  2sin 2    1
2
z1 3 x  4 x3  2 1  2 x 2   1
or, is purely imaginary
z2
 2  4 x 4  4 x 2  1  1
z 
 Re  1   0  8x4  8x2  1
 z2 
z z 8 x 4  4 x3  8 x 2  3x  1  0
Also, 1  1 i.e. z1 z2  z2 z1  0 19. C1  C2
z2 z2
SOLUTIONS 257

1 3  2i i 23. (A) Let z1 , z2 , z3 be the roots


z   3  2i 2 1 i Then,
i 1 i 3 z1  z2  z3  a ... 1

Clearly  z   z z1 z2  z2 z3  z3 z1  b ...  2 
zz z1 z2 z3  c
 zz 2 z k z Also, z1  z2  z3  1
From (2), we have z1 z2 z3  z1  z2  z3   b
Thus, k  2 or 3k  6
 From (1) and (2), a  b
2 2 2
20. z  z  z
1 2 3 (B) Let a  b  t
2
  z1  z2  z3   2  z1 z2  z2 z3  z2 z1  So, we have
1 1 1 f  z   z 3  tz 2  tz  c  0 and
 2 z1 z2 z3    
 z1 z2 z3  t  z1  z2  z3  3
 2 z1 z2 z3  z1  z2  z3   0 f   z   3z 2  2tz  t
21. 1    2 2  3 3  ...  2012 2012  a  b 2 D  4t 2  12t  0 for t  3
 1    2 2    3 3  4 4  5 5   f  z   0 has exactly one root
 ...  671 brackets 24. C1  3x 2  3 y 2  8 x  2 y  3  0 is a circle
 a  b 2 2 2
with radius
  2   670   2 2   a  b 2 3
 671 2  670  a  b 2 C2  x 2  y 2  6 x  8 y  9  0 is a circle with
radius 4
 a b 1
4 1
 1 i 3  1 , 1    ,  ,  2 ,  2    3, 4 
 2   z2  z1   3 3
22. z3  z1  2 
  Z  Z3
25. (A) 1  ei /2
or, z3  z1  2    z2  z1  Z 2  Z3
 2  z2  z1  e  i /3 Z
(B) 1  ei /6
Let z2  z3   Z2
Using cosine rule in ΔABC Z  Z3
(C) 2  ei /3
1 x2  4x2   2 Z1  Z3

2 2.x.2 x Z  Z2
(D) 1  ei 2 /3
or, 3x   2
2
Z3  Z 2
1  26. z1  z2  z1  z2 is possible only when
Area = Δ = .2 x.x.sin
2 3 arg  z1   arg  z2 
2
3  2
 . or , 3 z2  z3  6 So, statement-2 is not always true in all
2 3
k 6 conditions.
SOLUTIONS 260

z1  z2  z3 Re  z1 z2  z2 z1  ...
Now, 1  Re  P   n 
3 r2
 z1  z2  z3  9
2

 n
 nr 2 
0
  z1  z2  z3  z1  z2  z3   9 r2
38. a  b  c 2  a  b 2  c
 4 1 1  4 1 1 
         9
 z1 z2 z3  z1 z2 z3   2 a  b  c 2
And  QOP  120  a  b  c 2
 a  b 2  c   2
36.  15  1 and  is the root of 10

x14  x13  ...  x 2  x  1  0 39.  r 


r 1
r  11 r 
Now, f  x   x13  2 x12  3x11  ...  13x  14
  1 2  10   2  2   9   3 3  8 
f  x 14
 x12  2 x11  ...  12 x  13  4  4   7   5  5   6   6  6   5 
x x
By subtracting 7  7   4   8  8   3   9  9   2 
 1 13 12 11 14 10 10  1 
 1   f  x   x  x  x  ...  x  1 
 x x     2   3   4  5   6 
  1  14 15  11 1 
  7  8   9  10
14
Put x   ,   f       
    
15
  1
 f   
1  11
1514   11
Hence, N   0
11
1    1   2  ... 1   14 
And, we know that
z
1    1   2  ... 1   14   15 40. Chord AP 
z
 1  arc  AP   
 N  1513  313  513
2 z
37. z1  z 2  ...  zn  0   1  arg z
z
2 2 2
 z1  z2  ...  zn  z1 z2  z2 z1  ...  0
n 
n
z 
Also P    i 
 
j 1 i 1  z j 

z z  zz z z
  1  2   ...  n  1 22  2 21  ...
 z2 z1  z2 z2

P  n
 z1 z2  z2 z1  ... ,
r2
where r  z1  z2  ...  zn
SOLUTIONS 262

 2  t1  t1  t  12. The curve is xy  c 2


 2  2 2 
 1
 4  t1  t1  t  (2, 4) satisfies it.
1 1  2  4  c2
  1  Required curve is xy  8
 2  t1   t  t1 
So, eccentricity of rectangular hyperbola is
 t12   2  t1  t1  2t  1  0 2.
For real t1 , 13. The curve is xy  c 2
2
2  t   4  2t  1  0  t 2  4t  0 (2, 4) satisfies it
 t   , 0   4,    2  4  c2
8. Point S(1, 2) lies on x  y  3
 Required curve is xy  8
y1 y2  b 2
Using reflection property of ellipse, we have  focus of rectangular hyperbola is
y y
 1 2
x1 x2
  
2c, 2c and  2c,  2c 
Eliminating  x2 , y2  , we get  foci are  4, 4  and  4,  4 
 Locus of focus is a pair of straight lines
9. Transverse and Conjugate axes are angle 14. The curve is xy  c 2
bisectors of asymptotes (2, 4) satisfies it
And as asymptotes are perpendicular here, so
asymptotes are also angle bisectors of  2  4  c2
Transverse and Conjugate axes.
Hence, observe (2, 4) will lie closer to TA in  Required curve is xy  8
comparison to CA.
10. Equation of normal to the curve y 2  4ax and  Equation of directrix of rectangular
x 2  4by in terms of m are given by hyperbola is x  y   2c
b
y  mx  2am  am3 and y  mx  2b  2  Equation is x  y   4
m
For common normal both values of y are 15. PS  PS   SS   2a  2ae
equal
2ae  a cos    a  ae cos   ae 
b   a  ae cos   ae 
 2b   2am  am 3  0 h
m2 2a  2ae
 am5  2am3  2bm 2  b  0  ae cos 

Which can attain maximum 5 values of m 2ae  b sin  be sin 


k 
2a  2ae 1 e
So maximum normal can be 5
2 2

11. By using condition for tangency we get  h   k 1  e  


     1
4h 2  3k 2  2  ae   be 
 Locus of  h, k  is 4 x 2  3 y 2  2 Which is an ellipse
4 7
 e2  1   e
3 3
SOLUTIONS 264

 dy  8m 4  2m 2  1  0
 Coordinates of P are  x, x 
 dx  2  6 1 1
m2   , m
16 4 2
dy a 1 1
  y  x  2, y   x  2
dx x 2 2
  2 y   x  4
 locus of P is y  ax or y 2  ax ,
P   4, 0 
Which is a parabola 22. CC1  r  r1 and CC2  r  r2
20. Let M   x, y   CC2  CC1  r2  r1
2
 dy 
Then equation of L is Y    X 
 x  1 y2
  1, which is a hyperbola
 dx  1 3
4  
4
 dy  with eccentricity 2.
 Coordinates of P are  x, x 
 dx  23. Equation of this rectangular hyperbola is
x  y 1
dy x  x  x2  y2  2
  a  2
dx 2
a x 2 2 2
a  a  x   2 xy  2 x  2 y  1  0
  x2 Pair of asymptotes is
 2
 a x
2 
 a  a 2  x2   2 xy  2 x  2 y  1  c  0
 x2    0  c 1
 
   xy  x  y  1  0
  x  1 y  1  0 represents asymptotes
dy x
 24. Let x1  t12 , y1  2t1 and x2  t22 , y2  2t2
dx a2  x2
 x3  t1t2 , y3  t1  t2



 a a  a 2  x2
   a  a
2
a2  x2 

Obviously, x1 , x3 , x2 are in GP and y1 , y3 , y2
 x  x2 a 2  x 2  are in AP
   Let any tangent of given parabola be
ty  x  t 2
 x4  a2  x2   a2  x2
   t12  t 2  2t1t
 x 3 a 2  x 2  x So, l1  and so, on solving, we
1 t2
 locus of P is get l1l2  l32
y  a 2  x 2 or x 2  y 2  a 2 , 25. Let A   2t12 , 4t1  , B   2t22 , 4t2  and
C   2t32 , 4t3 
Which is a circle
Slope of AB  2  t1  t2  1 and
1 1
21. y  mx  ,  8m 2  2 t1  t2  t3  0
m m
So, t3  1
SOLUTIONS 265

2  t12  t22  t32  4  ab  4


Also,   t22  t22  1 2 2
3 3 30.   1
 t1  1, t2  0 t  2
t  
 t
 A   2, 4  , B   0, 0  and C   2,  4 
 2
Hence P   6, 0   4  t t  
 t
26. The equation of the locus is y  2  x 2  1  t2  2
1  2  1   2  1  2
Thus, parabola is x 2   y  2 31.  
2 1 1 2
 vertex is (0, 2) 1 3
  ,  
 1   17  2 2
 focus is  0, 2     0, 
 8  8  1 3 2  1  3  5 
   
2 2  2  3
1 15 2 1 5
 Directrix is  y  2   or y 
8 8  11 3 
 S   ,     , 
27. M  x0 , y0  is a point on the locus  2 2
11 3
y0 2 y0  x0  1  2
   2 2 9
x0  2  x0  12  y02 Semi latus rectum  
2 2
 3 x02  y02  3 2 2 2
32. Let the circle x  y  r touch the curve
 Locu of M is the hyperbola 3 x 2  y 2  3 xy  8
y2 64
Thus, hyperbola is x 2  1  x 2  2  r 2 should have equal roots
3 x
 x 4  r 2 x 2  640  0 should have equal
 e  1 3  2 roots
 LLR  2  3  6  r4
It means for r  1, 2, 3 the circle x 2  y 2  r 2
1
 Directrix  x   neither touches nor intersects the curve
2
xy  8
28. Required points will lie on the intersection 33. Any point on the line x  y  1 can be taken
points of the hyperbola and director circle of  t, 1  t 
the given circle.
Equation of the chord, with this as mid point
29. Equation of tangent is y  2 x  4a 2  b 2 2
is y 1  t   2a  x  t   1  t   4at ,
This is normal to the circle
x  y2  4x  1  0
2 It passes through  a, 2a 
This tangent passes through  2, 0  So, t 2  2t  2a 2  2a  1  0, this should have
2 distinct real roots
So, 0  4  4a 2  b 2 So a 2  a  0, 0  a  1,
 4a 2  b 2  16 So length of latusrectum < 4
4a 2  b 2 34.
Using AM  GM, we get  4a 2 b 2
2
SOLUTIONS 267

2 2 2
Again slope of OP is  M OP  x  4x  p
t p
2 Hence b  4
Also, slope of OQ is  M OQ
t1 (b) If h1 , h2 , h3 are the altitudes drawn to the
4 sides a, b and c, then
Since M OP M OQ  1 
tt1 1 1 a 1 1 b
  and  
 2 4 h1 2 12 h2 2
 4t  t    4
 t 1 c a b 1
2
So, a  b and   
t 2 p 2 2 6
(b) P 1, 2  , Q  4, 4  , R 16, 8 1 ab 1
Also,    3 p 6
Now, ar   PQR   6 sq. units p 2 3
(c) Z lies on the circle with centre at (4, 8)
(c) Equation of normal form any point
and radius 10
P  am 2 ,  2m  is y  mx  2am  am3
Z also lies on the ellipse with foci at (3, 5)
 11 1  Since, the length of the minor axis is also
It passes through  , 
 4 4 10
2
 4m  8m  11m  1  0 Z can be only one of the extremities of the
 4m 2  3m  1  0 minor axis.
(d) If the area of  PAB is 3, the point P and
Now, f  m   4m2  3m
origin cannot lie on either side of AB.
 f   m   12m2  3  0 Since the distance if P from AB is less than
1 6
 m 5
2
Therefore, P must lie on the straight line draw
1  1
Since f   f     0 has 3 normals are 6
2  2 parallel to AB at a distance of towards the
5
possible
origin side of AB and this intersects the
(d) Since, normal at P  t1  it meets the curve ellipse at exactly two points.
2 (One in quadrant 2 and the other in quadrant
at  t2  , then t2  t1  4).
t1
39. Conceptual
Such that here normal at P 1 meets the 2
2 2 4  5 x  12 y  6 
curve again at Q  t  40.  x  2    y  3   
9 13 
1 2
 t  1   3 e  , focus (2, 3), directrix  5 x  12 y  6  0
2 3
 t 1  4 a
2
Distance between focus, directrix   ae
38. (a) If y  y  p   a  x  p  is the parabola, e
 3 2  5
  52
2  a    a   4
then a    2 3   6  13
p
24 48
2 2 a , 2a 
Thus, y    x  p  p 5 5
p

You might also like